Đến nội dung

Ban Biên Tập

Ban Biên Tập

Đăng ký: 01-01-2012
Offline Đăng nhập: 01-08-2013 - 22:35
***--

#325346 Diễn đàn Heidelberg của những người được Giải thưởng

Gửi bởi Ban Biên Tập trong 15-06-2012 - 09:09

LIÊN ĐOÀN TOÁN HỌC THẾ GIỚI (IMU)

Hình đã gửi
Ingrid Daubechies (Chủ tịch), Martin Grotschel (Tổng thư kí)
Ngày 22 tháng 6 năm 2012
Thông cáo báo chí




Diễn đàn Heidelberg của những người được giải.
Những người đoạt các giải thưởng danh giá: Abel, Fields và Turing sẽ gặp các nhà khoa học trẻ đầy hoài bão. Đó là nội dung của Diễn đàn được thành lập bởi Quỹ Klaus Tschira.

Quỹ Klaus Tschira sẽ thành lập Diễn đàn Heidelberg của những người được giải, dưới hình thức một hội nghị thường niên, để những nhà khoa học đạt các giải thưởng cao quý của ngành Toán (Abel và Fields) và Khoa học máy tính (Turing) gặp gỡ với những nhà nghiên cứu trẻ tài năng. Diễn đàn này được đề xướng bởi Học viện Nghiên cứu lý thuyết Heidelberg (HITS), một viện nghiên cứu của Quỹ Klaus Tschira ở Đức nhằm thúc đẩy các ngành Khoa học tự nhiên, Toán học và Khoa học máy tính. Diễn đàn Heidelberg của những người dược giải (gọi tắt Diễn đàn Heidelberg) được xây dựng theo mô hình Hội nghị thường niên của Các cuộc hội nghộ thường niên Lindau của những người được giải thưởng Nobel Laureate - tiến hành hơn 60 năm qua với mục đích tìm kiếm những ý tưởng mới. Klaus Tschira, người sáng lập và đồng quản lý Quỹ nói: “Gặp gỡ với các nhà khoa học hàng đầu của Toán học và Khoa học máy tính sẽ truyền cảm hứng và tiếp thêm động lực mạnh mẽ cho các nhà khoa học trẻ tuổi.”


Hình đã gửi



Thỏa thuận hợp tác về Diễn đàn Heidelberg giữa những người tổ chức và các tổ chức trao giải (Viện Hàn lâm Khoa học và Văn học Nauy, Liên đoàn Toán học thế giới và Hiệp hội Máy tính ) đã được ký kết vào ngày 22 tháng 5 năm 2012 tại Oslo nhân kỷ niệm 10 năm giải thưởng Abel.

Việc nghiên cứu trong tất cả các ngành đều cần đến các phương pháp toán học và công cụ máy tính, và thành quả của chúng được ghi nhận trong mọi khía cạnh cuộc sống hàng ngày của chúng ta. Toán học và Khoa học Máy tính là nền tảng không thể thiếu của thế giới công nghệ. Giải thưởng Turing từ lâu đã được ghi nhận là giải thưởng khoa học cao quý nhất thế giới cho ngành Khoa học Máy tính, cũng như giải thưởng Abel và Fields đối với ngành Toán. Tuy nhiên, trong khi các nhà nghiên cứu trẻ của ngành Vật lý, Hóa học, Dược, và Kinh tế có cơ hội giao lưu gần gũi với những người đoạt giải Nobel tại Lindau hàng năm thì giới nghiên cứu trẻ của Toán học và Khoa học Máy tính lại chưa có những cơ hội như vậy.

Bắt đầu từ tháng 9/2013, Diễn đàn Heidelberg sẽ tạo cơ hội để những người đoạt giải thưởng Abel, Fields và Turing có thể gặp gỡ những nhà khoa học trẻ từ khắp nơi trên thế giới. Cuộc gặp mặt sẽ được tổ chức tại thành phố Heidelberg, nơi có viện nghiên cứu của Quỹ Klaus Tschira là Học viện Nghiên cứu lý thuyết Heidelberg (HITS). Diễn đàn được tổ chức với sự cộng tác của Hiệp hội máy tính (ACM; Giải thưởng Turing), Liên đoàn Toán học thế giới (Giải thưởng Fields) và Viện Hàn lâm Khoa học và Văn học Nauy (Giải thưởng Abel). Cuộc gặp đầu tiên của Diễn đàn Heidelberg sẽ diễn ra trong các ngày 23-27 tháng 9 năm 2013.

Đại diện cho Liên đoàn Toán học thế giới, Chủ tịch Ingrid Daubechies, thể hiện sự vui mừng trước sáng kiến của Quỹ Klaus Tschira, và hi vọng diễn đàn mới này sẽ góp phần đáng kể trong việc thúc đẩy nhiệt huyết toán học cho thế hệ sau.Việc gặp gỡ những người “anh hùng khoa học” đối với các nhà nghiên cứu mới vào nghề là điều rất quan trọng bởi nó có thể cho họ thấy những những nhà khoa học lẫy lừng này không phải là những thần tượng không thể vươn tới được. Chẳng hạn như Toán học, cùng với tất cả những gì trừu tượng của nó, vẫn được chinh phục bởi con người, trong đó mọi khía cạnh của sự trao đổi giữa người với người cũng đóng một phần quan trọng. Chúng ta cần thu hút và giữ vững tinh thần nghiên cứu toán học ở những trí tuệ trẻ sáng lạn có ham muốn làm việc và tích cực đẩy mạnh sự phát triển của ngành Toán học.

Quỹ Klaus Tschira Stiftung là một quỹ của Đức thúc đẩy Khoa học tự nhiên, Toán học và Khoa học máy tính. Học viện Nghiên cứu lý thuyết Heidelberg là một viện nghiên cứu của Quỹ.

Bắt đầu từ năm 2003, Viện Hàn lâm Khoa học và Văn học Nauy trao Giải thường Abel hàng năm cho các nghiên cứu khoa học xuất sắc của ngành Toán. Giải thưởng trị giái 800,000 Euro. Thành lập năm 1857, Viện Hàn lâm Khoa học và Văn học Nauy là một cơ quan phi chính phủ, liên quốc gia và liên ngành quan tâm tới tất cả các lĩnh vực kiến thức. Viện có 895 thành viên, trong và ngoài Nauy.

Liên đoàn Toán học thế giới bao gồm hơn 70 quốc gia thành viên. Liên đoàn thúc đẩy hợp tác quốc tế về Toán học, tổ chức các Đại hội Toán học thế giới, khuyến khích và ủng hộ các hoạt động Toán học trên khắp thế giới nhằm đóng góp vào sự phát triển của Toán học ở bất kỳ phương diện nào: toán học đơn thuần, toán học ứng dụng hay giáo dục Toán học.

Hiệp hội Máy tính là cộng đồng giáo dục và khoa học về máy tính lớn nhất thế giới, quy tụ các nhà giáo dục, nghiên cứu và chuyên gia về máy tính để truyền cảm hứng vào các cuộc đối thoại, chia sẻ các nguồn nghiên cứu và những thách thức trong ngành.

Để biết thêm thông tin về sự kiện quan trọng này, vui lòng xem thêm các tin tức liên quan tại website: http://www.mathunion.org/general/news và các thông cáo báo chí đính kèm



Xin vui lòng thông báo những thông tin này đến cộng đồng của bạn
Thân mến,
Martin Grotschel
Tổng thư kí Liên đoàn Toán học thế giới
Theo http://viasm.edu.vn/?p=4750
Mời bạn cùng thảo luận tại:



#325345 Gs Vũ Hà Văn

Gửi bởi Ban Biên Tập trong 15-06-2012 - 08:53

BBT: Đây là bài viết của Giáo sư Nguyễn Duy Tiến viết và gửi cho Diễn đàn toán học nhân ngày sinh nhật GS Vũ Hà Văn 12/06


Viết một bài Toán lạ thành quen
Viết trăm bài Văn quen thành lạ

Hình đã gửi


Chúc mừng sinh nhật GS Vũ Hà Văn

Vũ Hà Văn, con người Tài Hoa
Có thể nói rằng Vũ Hà Văn là nhà toán học Việt Nam xuất sắc nhất vể Toán Học Rời Rạc gồm lý thuyết Tổ Hợp, Xác Suất và Khoa Học Máy Tính, và là nhà toán học hàng đầu của thế giới trong lĩnh vực nói trên. Thật vậy, tính tới tháng 8/2010 (xem Lý lịch Khoa học dưới đây) GS. Vũ Hà Văn đã công bố 104 công trình trên các tạp chí uy tín nhất của Toán học (như Ann. Math; Adv. Math.), hoặc trên các tạp chí chuyên ngành (như Ann. Probab.), trong số đó có tới 74 bài ISI, và chỉ số H của anh là 13. Thêm vào đó, anh được trao tặng nhiều giải thưởng danh giá (trong đó có Giải thưởng Polya); được mời báo cáo ở rất nhiều hội nghị quốc tế; anh làm việc ở nhiều đại học và trung tâm khoa học lớn (3 lần làm việc ở Princeton institute for advanced studies; tham gia tổ chức nhiều hội nghị quốc tế. Năm 2006 GS. Vũ Hà Văn cùng với Terencer Tao (giải thưởng Fields) xuất bản cuốn sách nổi tiếng Additive Combinatorics (Tổ Hợp Cộng Tính), một đóng góp mới cho Toán Học Rời Rạc. Ngoài ra, anh còn đào tạo được nhiều học trò giỏi trong lĩnh vực này.

Hình đã gửi



Nếu Ngô Bảo Châu là bom tấn (đánh điểm) thì Vũ Hà Văn là bom rải thảm (đánh diện). Đó là hai nhà toán học (quốc tịch) Việt Nam mở đầu cho thời kỳ Thăng hoa của Toán Học nước ta.
Tôi là người nghiên cứu Xác Suất trong nhiều năm (gần 40 năm), khi đọc Lý lịch Khoa học của GS, Vũ Hà Văn, tự thấy cần rút lui để thế hệ trẻ (tài năng) tiến bước. Và giờ đây tôi xin đóng vai người dẫn chuyện (MC) thì có lẽ phù hợp hơn.

Tóm tắt tiểu sử khoa học của GS Vũ Hà Văn
Ngày sinh: 12/06/1970 (tuổi Canh Tuất).
Nơi sinh: Hà Nội.
Quê quán: Nam Đinh.
Họ và tên bố: Vũ Quần Phương (nhà thơ).
Họ và tên mẹ: Đào Thị Hường (dược sĩ).


Vũ Hà Văn là cựu học sinh chuyên toán trường trung học danh tiếng Chu Văn An, tốt nghiệp phổ thông năm 1986. Anh sinh ra trong một gia đình có truyền thống hiếu học (bố anh tên thật là Vũ Ngọc Chúc, sinh năm 1940, tốt nghiệp đại học Y, làm bác sĩ 2 năm, rồi trở thành nhà thơ Vũ Quần Phưng, với bài thơ nổi tiếng: Đợi, 100 bài thơ hay của thế kỉ XX, NXBGD, 2007), thi đỗ đại học Bách Khoa Hà Nội với số điểm rất cao (á thủ khoa). Sau đó anh dược nhà nước Việt Nam gửi sang Hungary học.

Vũ Hà Văn tốt nghiệp cử nhân tại Đại học Etvos, Budapest, Hungary năm 1994. Trong thư gửi cho tôi (ngày 12/07/2011) anh tâm sự với tôi: "Thật ra tiểu sử khoa học của Văn có một điểm khác, so với phẩn lớn những người làm toán khác. Đó là Văn khi mới vào đại học chưa theo học ngành toán, mà là hoe điện tử tại Đại Học Bách Khoa Budapest (chứ không phải Đại Học Etvos là trường Tổng Hợp). Học ở đó 1,5 năm. Văn vẫn thích học toán và làm nghiên cứu với bà vợ của GS. Lovasi, rồi GS Lovasi khuyên nên chuyển sang học Toán Lý thuyết. Văn cũng đi gặp anh Trần Văn Nhung hỏi ý kiến và anh Nhung cũng khuyên đây là một cơ hội tốt. Sau đó Văn mới chuyển sang trường Etvos. Thành ra việc trở thành người làm toán cũng một phẩn là do say mê, một phẩn như có số mệnh sắp đặt vậy, chứ con đường không được thẳng băng như mọt số người làm toán khác''.

Bảo vệ luận án tiến sĩ tại Đại học Yale, Mỹ, năm 1998 dưới sự hướng dẫn của GS Laszlo Lovasz.
Là người được tặng Giải thưởng Polya năm 2008.
Sau thời gian làm hậu tiến sĩ tại Viện Nghiên cứu cấp cao (IAS) Princeton và tại Ban Nghiên cứu của Microsoft, từ năm 2001 đến 2005, anh làm việc tại Đại học Caliíbmia ở San Diego, với tư cách trợ lý giáo sư, phó giáo sư và giáo sư (full professor). Từ mùa thu năm 2005, anh trở thành giáo sư Khoa Toán Đại học Rutgers, và hiện tại anh là GS. Đại Học Yale (nơi anh bảo vệ tiến sĩ, năm 1998).
Anh là giáo sư thỉnh giảng của Đại học Paris 6 năm 2006.


Lĩnh vực nghiên cứu của Vũ Hà Văn gồm: toán học tổ hợp, xác suất rời rạc, lý thuyết sô cộng tính và Khoa học Máy tính Lý thuyết.
Anh đã hai lần nhận được Giải thưởng Sloan dành cho các tài năng trẻ ở Mỹ khi viết luận án tiến sĩ (1997), và khi làm nghiên cứu viên (2002), rồi Giải thưởng NSF Career (2003).

Anh là thành viên Viện Nghiên cứu cấp cao Princeton trong những năm 1998-1999, 2005-2006, và 2007 (năm 2007 là người lãnh đạo nhóm dự án Số học tổ hợp tại viện này).
Tính đến nay, số nhà toán học được tặng Giải thưởng Polya vẫn còn rất ít, và họ
đều là những nhà toán học xuất sắc.

Có lần tôi hỏi Vũ Hà Văn: "Trong những giải thưởng anh đã nhận, theo anh giải thưởng nào danh giá hơn cả?"
Anh trả lời ngay: "Giải thưởng Polya năm 2008."

Giải thưởng Polya.
Đây là giải thưởng do Hội Toán công nghiệp và ứng dụng Society for Industrial and Applied Mathematics/ SIAM) của Mỹ lập ra từ năm 1969, trao 2 năm một lẩn, lẩn lượt cho những ứng dụng nổi bật về lý thuyết tổ hợp hoặc những đóng góp nổi bật trong các lĩnh vực khác mà George Polya từng yêu thích như: lý thuyết xấp xỉ, giải tích phức, lý thuyết số, đa thức trực giao, lý thuyết xác suất, v.v. Giải thưởng chủ yếu dành cho những công trình mới, hiếm khi cho thành tựu trong quá khứ.

SIAM được thành lập năm 1952, đặt trụ sở chính tại Philadelphia (Mỹ), có 12.000 thành viên cá nhân và 500 thành viên tập thể (gồm các trường đại học, viện nghiên cứu, xí nghiệp công nghiệp, công ty dịch vụ, tư vấn dân sự và quân sự khắp thế giới).
Quá trình xét chọn người trúng giải được tiến hành nghiêm ngặt. Uỷ ban Giải thưởng được lập ra ít nhất 18 tháng trước ngày tặng giải; phải tham khảo rộng khi xét chọn; có nhận xét bằng văn bản trước 10 tháng, v.v.
Người trúng giải được tặng một tấm huy chương và 20.000 USD. Lần trao Giải thưởng Polya năm 2008 dành cho những ứng dụng của lý thuyết tổ hợp. Người duy nhất được tặng giải là nhà toán học Việt Nam Van H. Vu (tức Vũ Hà Văn).


Đây là danh sách những người được giải Polya (tính đến 2010):
* 1971 R. L. Graham, K. Leeb, B. L. Rothschild, A. w. Hales, and R. I. Jewett
* 1975 R. p. Stanley, E. Sz emeredi, and R. M. Wilson
* 1979 L. Lovasz (thầy hướng dẫn luận án tiến sĩ của Vũ Hà Văn)
* 1983 A. Bjomer and p. Seymour
* 1987 A. c. Yao
* 1992 G. Kalai and s. Shelah
* 1994 Gregory Chudnovsky and Harry Kesten
* 1996 Jeffry Ned Kahn and David Reimer
* 1998 Percy Deift, Xin Zhou, and Peter Samak
* 2000 Noga Alon (cùng với Spencer cuốn sách nổi tiếng: Probabilistic Method)
* 2002 Craig A. Tracy and Harold Widom
* 2004 Neil Robertson and Paul Seymour
* 2006 Gregory Lawler, Oded Schramm, and Wendelin Wemer
* 2008 Van H. Vu
* 2010 Emmanuel Candốs and Terence Tao.

Như vậy, Vũ Hà Văn được giải Polya trước Terence Tao. Năm 2012 sẽ công bố người nhận giải Polya tiếo theo.
Từ ngày 16 đến 22/12/2009, tại Seoul, diễn ra cuộc gặp làm việc giữa các nhà toán học Mỹ và Hàn Quốc. Terence Tao, Van H. Vu (tức Vũ Hà Văn), James T. McKeman, Frank MorganHee Oh là những người được Hội Toán học Mỹ cử sang Seoul giới thiệu những công trình mới. Theo dõi qua Internet, tôi thấy giới toán học Hàn Quốc đón tiếp rất trọng thị đoàn đại biểu giới toán học Mỹ mà Vũ Hà Văn là một thành viên.


Năm 2009, Nhà nước ta đã công nhận Vũ Hà Văn là giáo sư kiêm chức tại Viện Toán học Việt Nam, khi anh 39 tuổi. Ngô Bảo ChâuVũ Hà Văn là hai giáo sư trẻ nhất Việt Nam. Dù sống và làm việc ở nước ngoài nhiều năm, cả hai anh vẫn giữ quốc tịch CHXHCN Việt Nam.

Mới đây nhất, GS Vũ Hà Văn đã phát biểu cảm tưởng về “sự kiện Ngô Bảo Châu”:
GS Châu là bạn tôi. Tôi rất phấn khởi khi nghe tin này. Mặc dù “tin đồn ” đã có từ ỉ âu trong giới toán học, nhưng khi biết chắc chắn vẫn vui hơn. Cũng như anh Châu, tôi và vợ con về Việt Nam gần như thường xuyên, và mỗi lẩn về, tôi đều kết hợp giảng dạy và nghiên cứu tại Viện Toán học và Trường đại học Khoa học tự nhiên (thuộc Đại học Quốc gia Hà Nội).


Hướng Nghiên cứu Toán học của Vũ Hà Văn
Tôi đã nghe GS. Vũ Hà Văn giảng bài hai lần. Lần đầu tiên ở xeminar Xác Suất trường đại học KHTN (DHQGHN) anh nói về Phương Pháp Xác Suất ứng dụng trong Lý Thuyết Số. Đây là lĩnh vực do trường phái Toán Học Hungary (với những nhà khoa học lừng danh như Renyi, Erdos) khai sinh từ những năm 60 của thế kỷ trước và hiện tại đang là mốt của Xác Suất rời rạc vì có nhiều ứng dụng trong lý thuyết số, đồ thị ngẫu nhiên và khoa học máy tính, Trong bài giảng, điều gây ấn tưạng nhất đối với tôi là anh ứng dụng bất đẳng thức Talagrand (một nhà toán học hàng đầu của nước Pháp trong lĩnh vực Xác Suất trên các không gian Banach) vào công trình của anh (cùng với Kim).

Quãng 5 năm sau (2009), tôi cùng với sinh viên nghe anh giảng về Matrận Ngẫu Nhiên. Lần này anh đã truyền đạt một trong những vấn đề về Tổ Hợp Cộng Tính hết sức sâu sắc và đang được cả thế giới toán, vật lý quan tâm. Anh giảng bài mà không cần một giáo án nào, tất cả kiến thức của anh rất chắc chắn, lần lượt tuôn ra từ bộ não sáng sủa của anh (không nhầm lẫn). Nghe anh giảng về toán giống như nghe bố anh, nhà thơ Vũ Quần Phương bình thơ vậy, tôi hết sức thích những ý tưởng hình học của anh và những ví dụ cụ thể. Anh là người có năng khiếu sư phạm bẩm sinh.

Bây giờ ta hãy tìm hiểu sơ qua những vấn đề mà GS. Vũ Hà Văn quan tâm.
1) Phương Pháp xác suất.
Từ lâu người ta đã dùng các kết quả của Xác Suất để chứng minh một số kết quả của Giải Tích, Đại Số hoặc Lý Thuyết số (ví dụ như dùng luật số lớn chứng minh Đinh lý Weierstras Xấp xỉ hàm liên tục bằng đa thức; Bất đẳng thức Khinchin v.v...). Để giới thiệu về phưang pháp quan trọng và mới này, tôi dẫn ra đây lời mở đầu của cuốn sách "The Probabilistic Method" của hai nhà toán học Noga Alon, Joel H. Spencer (2008, xuất bản lần thứ 3, của John Wiley & Sons. INC): Phương pháp xác suất là một phương pháp hữu hiệu được ứng dụng rộng rãi trong tổ hợp. Một trong những nguyên nhân chính giải thích cho sự phát triển mau lẹ của phương pháp này là tẩm quan trọng của tính ngẫu nhiên trong lý thuyết khoa học máy tính và vật lý thống kê. Tương tác qua lại giữa toán học rời rạc và khoa học máy tính đã gợi ý cho một cái nhìn thuật toán trong việc nghiên cứu phương pháp xác suất trong tổ hợp. Đây cũng là cách tiếp cận được sử dụng trong cuốn sách này. Cũng vì thế mà trong sách sẽ bao gồm những thảo luận về các kỹ thuật thuật toán cùng với sự nghiên cứu các phương pháp cổ điển cũng như các công cụ hiện đại được ứng dụng trong đó. Phẩn thứ nhất của cuốn sách đưa ra các công cụ được ứng dụng trong các lập luận xác suất, bao gồm các kỹ thuật cơ bản sử dụng kỳ vọng và phương sai, cũng như các ứng dụng gần đây của martingale và bất đẳng thức tương quan ịcorrelation inequaỉity). Phần thứ hai nghiên cứu một lớp rộng rãi các chủ đề trong đó các kỹ thuật xác suất đã được ứng dụng thành công. Phẩn này bao gồm một số chương về đồ thị ngẫu nhiên và đổ thị rời rạc (discrepancy graph), cũng như một vài lĩnh vực trong lý thuyết khoa học máy tính: mạch phức hợp ịcircuit complexity), hình học tính toán, tái ngẫu nhiên hóa các thuật toán ngẫu nhiên. Giữa các chương là các các đoạn ngắn được đặt dưới tiêu đề chung Lăng kính xác suất. Đây là những lời giải đẹp, chúng không nhất thiết có liên quan tới các chương trước đó và có thể đọc một cách độc lập. Cơ sở của phương pháp xác suất có thề mô tả như sau: đề chứng minh sự tồn tại của một cấu trúc tổ hợp với các tính chất xác định, chúng ta xây dựng một không gian xác suất thích hợp và chỉ ra rằng một phần tử được chọn một cách ngẫu nhiên trong không gian này có tính chất mong muốn với một xác suất dương. Phương pháp này được khởi đầu bởi Paul Erdos, người đã đóng góp rất nhiều cho sự phát triển của nó trong suốt hơn 50 năm, phương pháp này vì thê có thề được gọi là "Phương pháp Erdos". Đóng góp của Erdos có thể được đánh giá không chỉ bởi rất nhiều kết quả sâu sắc của ông, mà còn dựa trên nhiều bài toán và giả thuyết mang tính dẫn đường mà từ đó đã kích thích nghiên cứu rộng lớn trong lĩnh vực này. Dường như là không thể có một cuốn từ điển về phương pháp xác suất; có quá nhiều kết quả thú vị gần đây có sử dụng những lập luận xác suất, và chúng tơ sẽ không cố gắng đề cập tới tất cả những kết quả đó.

2)Đồ thị ngẫu nhiên.

Để hiểu lý thuyết này, ta xét ví dụ sau.Số Ramsey $R(k, l)$ là số nguyên dương bé nhất sao cho mỗi đồ thi đầy đủ với $n$ đỉnh $K_n$ và các cung được tô hai màu xanh và đỏ tồn tại hoặc đồ thi con đầy đủ với $k$ đỉnh $K_k$ với tất cả các cung màu đỏ, hoặc $K_l$ màu xanh. Ramsey (1929) đã chứng minh rằng $R(k, l)$ là hữu hạn với hai số nguyên bất kỳ $k$ và $l$. Ta sẽ nhận được cận dưới của các số đường chéo Ramsey $R(k, l).$

Mệnh đề: Nếu
$$\binom{n}{k}2^{1-\binom{k}{2}}<1$$
thì $R(k,k)>n$. Do đó $R(k,k) > \begin{bmatrix} 2^{k/2} \end{bmatrix}$ đối với tất cả $k \geq 3.$

Chứng minh. Xét một đồ thi ngẫu nhiên hai màu có các cung của $K_n$ nhận được bằng cách tô màu mỗi cung màu xanh hay màu đỏ độc lập với xác suất như nhau. Với tập cố đinh $R$ của $k$ đỉnh, ký hiệu $A_R$ là biến cố: đồ thi con cảm sinh của $K_n$ trên $R$ là đơn màu (tức là tất cả các cung của nó là đỏ, hoặc tất cả xanh). Rõ ràng là,
$$Pr\begin{bmatrix} A_R \end{bmatrix}=2^{1-\binom{k}{2}}.$$
Vì có $\binom{k}{2}$ khả năng chọn $R$, nên xác suất để có ít nhất một trong các biến cố $A_R$ xảy ra nhiều nhất bảng
$$\binom{n}{k} 2^{1-\binom{k}{2}}<1$$
với xác xuất dương không có biến cố $A_R$ nào xảy ra và tồn tại một đồ thị hai màu của $K_n$ mà không đơn màu $K_k$ nào; tức là, $R(k,k)>n$. Chú ý rằng nếu $k \geq 3$ và ta lấy $n=\begin{bmatrix} 2^{k/2}\end{bmatrix}$ với tất cả $k \geq 3$

3) Ma trận ngẫu nhiên
Ma trân ngẫu nhiên là ma trận
$$A=(a_{ij})$$
Trong đó $a_{ij}$ là các biến ngẫu nhiên.
Định lý Terence Tao, Van Vu (2006). Nếu $a_{ij}$ là các biến Bernoulli, tức là,
$$P_{(a_{ij}=1)}=P_{(a_{ij}=1)}=1/2$$
độc lập, thì
$$P_(det(A)=0)<(3/4+o(1))^n.$$


Một trong những kết quả hay nhất (theo tôi) của Vũ Hà Văn

Định lý giới hạn trung tâm đối với đa diện ngẫu nhiên
Đầu những năm 60, RenyiSulanke nghiên cứu mô hình sau. Cho $K$ là một hình lồi trong $R^d$ ($d$ cố định) có thể tích 1. Lấy mẫu $n$ điểm trong $K$ một cách ngẫu nhiên và gọi $K_n$ là bao lồi của những điểm đó. Lý thuyết do $R-S$ đề ra là nghiên cứu các biến ngẫu nhiên được xác định bởi $K_n$, chẳng hạn số đỉnh của $K_n$ hoặc thể tích $K_n$.

Một bài toán mở cơ bản của lý thuyết là:Các biến ngẫu nhiên này thỏa mãn đinh lý giới hạn trung tâm.

Cho$K$là một tập lồi trơn với thể tích 1 trong $R^d$. Chọn ngẫu nhiên $n$ điểm một cách độc lập theo phân bố đều. Ký hiệu $K_n$ là bao lồi của các điểm này và gọi nó là đa diện ngẫu nhiên. Việc nghiên cứu các phiếm hàm then chốt (như thể tích, số đỉnh,...) của $K_n$, khởi đầu từ Efron, R.LenyiSulanke là một hướng cổ điển của hình học lồi. Ký hiệu $Vol(K_n)$ là thể tích của $K_n$. Một giả thuyết nổi tiếng trong lĩnh vực này là biến ngẫu nhiên này thỏa mãn định lý giới hạn trung tâm khi n tiến tới vô cùng.
Hình đã gửi
Giả thuyết.
Tồn tại hàm $\varepsilon (n)$ tiến đến 0 khi $n$ tiến đến vô cùng sao cho với mọi $x \in R$ ta có:
$$\begin{vmatrix} P\left(\dfrac{Vol(K_n)-E(Vol(K_n))}{\sqrt{Var(Vol(Kn))}}<n\right)-\Phi(x)\end{vmatrix}<\varepsilon(n)$$
Trong đó $\Phi$ là hàm phân phối chuẩn. Giả thuyết này mới chỉ được xét trong trường hợp $K$ là hình cầu trong $R^2$. Trong việc nghiên cứ đa diện ngẫu nhiên trong thể lồi $K$, mặt của $K$ đống vai trò cốt lõi. Một phần có ý nghĩa của tài liệu hiện có tập trung hai trường hợp sau:

1) $K$ trơn, nghĩa là biên của $K$ hai lần khả vi với độ cong dương hữu hạn.
2) $K$ là một đa diện.

Trong bài báo rất hay của Vũ Hà Văn đăng trong tạp chí hàng đầu Advances of Math, Văn đã chứng minh giả thiết cho trường hợp $K$ là tập lồi trơn

Định lý 1 của Vũ Hà Văn 2006 Cho $K$ là thể lồi trơn với thể tích 1 trong $R^d$. Tồn tại hàm $\varepsilon (n)$ tiến đến $0$ khi $n$ tiến đến vô cùng sao cho với mọi $x \in R$ ta có:
$$\begin{vmatrix} P\left(\dfrac{Vol(K_n)-E(Vol(K_n))}{\sqrt{Var(Vol(Kn))}}<n\right)-\Phi(x)\end{vmatrix}<\varepsilon(n)$$


Kí hiệu $f_i(K_n)$ là số các mặt bên $i$-chiều của $K_n,0 \leq i \leq d-1$ $(f_0(K_n)$ là số đỉnh của $K_n)$ Văn đã có kết quả sau:

Định lý 2 của Vũ Hà Văn, 2006. Cho $K$ là thể lồi trơn với thể tích 1 trong $R^d$. Tồn tại hàm $\varepsilon (n)$ tiến đến 0 khi $n$ tiến đến vô cùng sao cho với mọi $x \in R$ và

$$\begin{vmatrix} P(\frac{f_i(K_n)-E(f_i(K_n))}{\sqrt{Var(f_i(Kn))}}<n)-\Phi(x))\end{vmatrix}<\varepsilon(n).$$

$0 \leq i \leq d - 1$ ta có:

Trong cả hai định lý trên, ta có thể lấy
$$\varepsilon(n)=n^{-1/(d+1)+o(1)}.$$

Môi trường Làm viêc của Vũ Hà Văn
Toán học Hungary PAUL ERDOS

Vũ Hà Văn là hạt giống toán học tốt và được gieo trồng ở Hungary, mảnh đất rất màu mỡ, là quê hưang toán học của nhiều học giả xuất chúng như G. Polya, Paul Erdos, A. Rényi, von Neumann, Halmos vân vân. Hầu hết các học giả này gốc Do Thái, làm việc ở Mỹ. Hướng nghiên cứu của Vũ Hà Văn được khởi nguồn từ các ý tưởng của ErdosRényi (xem bài "Toán học Hungary").

Bạn làm Toán của Vũ Hà Văn. Vũ Hà Văn có khá nhiều bạn làm toán ở nhiều nơi khác nhau trên thế giới. Một trong số đó là Terence Tao: “Mozart của toán học".

T. Tao sinh năm 1975 (kém Văn 5 tuổi) tại Adelaide, Australia trong một gia đình người Hoa, bố là bác sĩ, mẹ là giáo viên dạy toán. Mới han 2 tuổi, nhờ “học mót” toán và tiếng Anh qua ti-vi, Tao đã dạy lại hai môn này cho một cậu bé 5 tuổi! Đến 9 tuổi, Tao được nhận vào chương trình nghiên cứu tài năng đặc biệt của Đại học Johns Hopkins ở Mỹ. Mới 10 tuổi, Tao lọt vào đội tuyển quốc gia Australia đi dự Olympic Toán quốc tế và đoạt huy chương đồng; năm sau, đoạt huy chương bạc; rồi đến năm 13 tuổi, đoạt huy chương vàng. T. Tao là người đoạt huy chương vàng ít tuổi nhất trong lịch sử các Olympic Toán quốc tế.

17 tuổi, Tao được tặng bằng thạc sĩ tại Australia, và nhận được học bổng sang Mỹ học tiếp. 20 tuổi, Tao bảo vệ thành công luận án tiến sĩ tại Đại học Princeton danh tiếng; 25 tuổi, trở thành giáo sư. Năm 2006, mới 31 tuổi, T. Tao được tặng Huy chương Fields (được coi như Giải thưởng Nobel trong toán học), trở thành một trong mấy người trẻ tuổi nhất được nhận vinh dự cao quý ấy. Terence Tao được coi là “Mozart trong toán học”.

Vũ Hà Văn kể lại:
Năm 2003, được ông Chủ tịch Hội Toán học Mỹ giới thiệu, tôi bắt đầu làm quen với Terence Tao. Năm ấy, Tao mới 28 tuổi, chưa được tặng Huy chương Fields. Anh sống với người vợ trẻ gốc Hàn Quốc trong một cản hộ hẹp tại quận Cam, bò ra sàn nhà làm toán. Cùng mang dòng máu châu A, nên chúng tôi dễ đồng cảm. về sau, qua trao đổi email, chúng tôi cảm thấy rất dễ hiểu những ý tưởng của nhau. Từ đấy, Tao và tôi cộng tác công bố được 15 bài báo khoa học và 1 cuốn sách chuyên khảo dày 500 trang. Riêng cuốn sách chúng tôi viết mất ba năm.

Đó là cuốn Additive Combinatorics (Tổ hợp cộng tính) của hai tác giả Terence TaoVan H. Vu, do Viện Nghiên cứu toán học cao cấp Đại học Cambridge (Anh) xuất bản năm 2006. Ben Green, một nhà toán học Anh rất nổi tiếng, làm việc tại Đai học Cambridge, viết. Ben Green cho biết thuật ngữ toán học tổ hợp cộng tính chỉ mới xuất hiện gần đây, do Terence Tao đặt ra, và đã trở thành một chuyên ngành toán học phát triển nhanh, mang lại nhiều hứng thú. Còn ít nhà toán học quen với thuật ngữ này, mặc dù họ rất quen những thành tựu cột mốc của nó. Sau khi phân tích ý nghĩa của cuốn sách, Ben Green kết luận:

Tóm lại, cuốn sách là một đóng góp quan trọng cho vãn liệu toán học và đã trở thành cuốn sách mà thế hệ sinh viên mới cần đọc cũng như những chuyên gia trong các lĩnh vực gần gũi cần học hỏi thêm về toán học tổ hợp cộng tính (chẳng hạn, chương 4 có thể coi là rất hấp dẫn đối với các nhà lý thuyết về khoa học tính toán). Đây là cuốn sách viết rất đúng lúc và hai tác giả của nó rất đáng được ngợi ca vì đã thể hiện một cách đầy thuyết phục. Riêng tôi, tôi có tới ba bản in: một bản để ở nhà, một để ở nơi làm việc, và bản thứ ba dự phòng trường hợp hai bản kia bị cũ nát.

Văn có nhiều quan hệ với người Hung và người Việt. Thày hướng dẫn luận án tiến sĩ của Văn là GS. L. Lovasz, người Hungary. Ông là nhà toán học lừng danh (sinh ngày 09/03, năm 1948; ba lần liên tiếp, 1964, 1965, 1966, đoạt huy chương Vàng Olympiad thế giới; con ông năm 2008 cũng đoạt được huy chương Vàng Olympiad). Ông được trao nhiều giải danh giá:

Kyoto Prize (2010)
Hungary's Szộchenyi Grand Prize (2008)
Bolyai prize (2007)
Gordel Prize (2001)
Wolf Prize (1999)
Fulkerson Prize (1982)
Best Information Theory Paper Award (IEEE) (1981)
Polya Prize (SIAM) (1979)

Ông đã từng giữ chức Chủ Tịch Hội Toán Học Quốc Tế (2007-2010), là viện sĩ của viện Hàn Lâm Thụy Điển, và viện Hàn Lâm Hoàng Gia Anh.

Ngoài ra, Vũ Hà Văn còn cộng tác nghiên cứu và đào tạo nhiều tiến sĩ người Việt và các nước khác (xem lý lịch khoa học của Vũ Hà Văn dưới đây).


Hình đã gửi

Từ trái qua phải GS Vũ Hà Văn, GS Đặng Hùng Thắng, GS Nguyễn Hữu Việt Hưng và GS Nguyễn Duy Tiến


Blog của Vũ Hà Văn
Ngày nay các nhà toán học thường dùng blog để trao đổi toán học với nhau và những người khác. Điều này làm cho các kết quả của toán học trở nên gần gũi hơn, và thường được phổ cập nhanh chóng. NgôbảoChâu có Blog "thích học toán" rất được nhiều ban trẻ quan tâm. Vũ Hà Văn có "Van's Blog" cũng thế. Xin được trích dẫn bài số đề của GS. Vũ Hà Văn:

Số Đỏ.
Dân ta thích đỏ đen. Không biết có tự bao giờ, nhưng số đề đang là trò “đỏ đen’’ được nhiều người ưu ái nhất, chơi nhiều nhất. Đêm nằm mơ số đề đẹp, sáng ra chợ nghe bàn, trưa đi đặt số, chiều đợi radio nghe xô’ số... đề. Luật chơi đề đợi loại như sau: Sáng bạn đặt một số tiền, nói đơn giản là 1 đô cho chủ đề, vào một số từ 00 đến 99. Mục đích của người chơi đề là làm sao số này trùng vào 2 chữ số cuối cùng của gidi sổ xổ do nhà nước phát hành trong ngày đó. Khi xổ số quay, hai chữ số này được xác định gọi là “đề về”), chủ đề so số và thanh toán tiền nong. Nếu sổ của bạn trùng, bạn sẽ được 70 đô (tức 70 lẩn số tiền đẩu tư). Nếu không trúng, bạn sẽ mất 1 đô đặt cược lúc đẩu. “Ai ơi yêu lấy số đề Khi đi một chỉ, khi về bảy cây !” Đánh đề thông dụng có lẻ bởi nó đơn giản dễ hiểu, và khả năng trúng xố, trong mắt người chơi là tương đối cao (1/100). Khả năng này cao hơn nhiều so với các giải xổ sốchính thức của nhà nước, và có tác dụng tâm lý rất mạnh. Những người chơi đề lâu ngày, thường ai cũng thắng, hoặc quen biết những người đã thắng một vài lần. Tâm lý chơi đề để có một cơ hội “đổi đời” rất phổ biến, nhất là đối với những người nghèo. Chuyện về “nuôi đề”, nằm mơ thấy “đề về”, đi thăm mộ thấy số đề, vv là những chuyện nghe thấy hàng ngày. Vậy thực chất Đánh Đề có phải là một trò chơi đem lại nhiều hy vọng? Nhìn về khía cạnh toán học mà nói, thì luật chơi đề rất thiệt cho ngươi chơi, vì kỳ vọng của nó là một số âm to đùng. Giả sử ông A chơi đề ngày một lẩn, mỗi lẩn đều đặn 1 triệu đồng. Như vậy sau 6 năm, tính là 2000 ngày cho chẵn, ông A bỏ ra 2 tỷ. Mỗi lẩn chơi, xác xuất trúng là 1%. Như thế, trung bình ông A sẽ trúng 20 lấn. Mỗi lẩn được 70 triệu, 20 lấn là 1,4 tỷ, như vậy, trung bình ông A lổ 600 triệu.

Tất nhiên, ông A sẽ nói “ờ thì trung bình là vậy, nhưng nhỡ tôi may thi sao ? Xác suất may của ông A hoàn toàn có thể tính được. Nó được biểu diễn qua một định lý rất nổi tiếng và cơ bán: Định lý giới hạn trung tâm: Nếu $X_1,X_2,...X_n$ là các biến độc lập ngẫu nhiên, có cùng kỳ vọng là $E$ và phương sai là $V=\sigma ^2$. Khi đó nếu $n$ tiến đến vô cùng
$$P(\frac{\sum^n_{i=1}X_i-nE}{\sqrt{n}\sigma } \geq x) \to \Phi(x),$$
ở đây
$$\Phi(x)=\frac{1}{\sqrt{2 \pi}} \int_{ x}^{\infty} e^{-t^2/2}dt$$
là hàm phân bố Gauss.


Điều quan trọng ở đây là hàm $e^{-t^2/2}$ tiến đến $0$ rất nhanh với $t$, do đó $\Phi (x)$ cũng tiến đến 0 rất nhanh với $x$. Chẳng hạn $\Phi (1) \leq 0,16$, $\Phi (2) \leq 0,03$, $\Phi(3) \leq 0,003.$
Định lý trên có thể viết dưới dạng
$$P(\sum _{i=1}^n X_i \geq nE+x \sqrt{n} \sigma ) \approx \Phi(x).$$
Quay trở lại với ông A. Muốn ứng dụng định lý trên, ta cho $X_i$ là số tiền ông $A$ thu hoạch trong lần chơi thứ $i$ có phân bố như sau: $P(X_i=-1)=0,99$ (thua) và $P(X_i=09)=0,01$ (thắng). Ký vọng của là $-0,3$ (triệu đồng) và phương sai xấp xỉ $49=7^2$. Nếu ông $A$ không lỗ sau 2000 lần chơi thì $\sum ^n_i=1 X_i \geq 0$, tức ta phải lấy

$$x \geq \frac{n \begin{vmatrix}E \end{vmatrix}}{\sqrt{2007}} \approx 1,9$$

Vậy xác suất để ông A "may" (không lổ vốn) là độ $\Phi(1, 9)$. Xác suât này cỡ ba phẩn trăm. Nói một cách nôm na, nếu có 100 người chơi như ông, trung bình chỉ có 3 người không lỗ. (Tất nhiên bạn được nghe ba ông này tuyên truyền về "tài năng" của mình bao nhiêu lẩn lại là chuyện khác. Đây là khía cạnh tâm lý của trò chơi.) Định lý giới hạn trung tâm phản ánh một hiện tượng quan trọng và có tính ứng dụng cao: Hiện tượng (Lơrge deviation): Một biến ngẫu nhiên được xác định bởi nhiều biến ngẩu nhiên độc lập thường lấy một giá trị gần với kỳ vọng của nó. Trong trường hợp vừa rồi, biến này là , tổng số thu hoạch của ông A. Bạn có thể dùng định lý giới hạn trung tâm để so sánh sổ đề với rullet, một trò chơi thông dụng ở casino. ở cả hai trò, kỳ vọng của người chơi đều âm, nhưng kỳ vọng của rullet là một số âm nhỏ.


Lý lịch khoa học của Vũ Hà Văn

Curriculum vitae V. H. Vu Name: Van H. Vu Contact information.
Department of Mathematics, 110 Frelinghuysen Road, Piscataway, NJ 08854, USA. Email: [email protected]">[email protected] 1 Referee. Editor: SIAM Joumal of Discrete Mathematics; Joumal of Combinatorial Theory A; Communications in Contemporary Mathematics; Acta Mathematica Viet- namica.
Referee: Combinatorica, Random Structures and Algorithms, Joumal of Combinato- rial Theory, Israel Math. Joumal, Duke Math. Joumal, CiAHA. J.A.M.S, Annals of Probability, Annals of Mathematics, Acta Mathematica etc.
Panelist: NSF panels (several times). Outside referee: NSA, Israel national research foundation, ERC Advanced Research Grants (EU).
Organizing. Random matrices, workshop, American Institute of Mathematics, Dec
2010 (with T. Tao).
Additive combinatorics, mini-symposium, SIAM Discrete Math, June 2010.
Random matrices, Dimacs workshop, Spring 2008, Rutgers.
Additive combinatorics, Focus Program, Fall 2007, 1AS (Princeton), (with J. Bour- gain).
Conference in Arithmetic Combinatorics, IAS, Dec 2007 (with J. Bourgain).
Special section in Probability and Combinatorics, AMS meeting, Rutgers, Oct 2007 (with J. Kahn).
Conference in Probabilistic Combinatorics, Bank Center, Nov. 2005 (with N. Alon, B. Reed and B. Sudakov).
Additive Combinatorics, section at AMS meeting, Santa Barbara, April 2005 (with M.c. Chang).
Conference in Additive Combinatorics, AIM, Palo Alto, September 2004 (with T. Tao). Probabilistic Combinatorics, section meeting at SIAM Discrete Math. conference, Au- gust 2004 (with B. Sudakov).
Probability in combinatorics and the Internet, section at the annual meeting of the AMS, San Diego, Jan 2002 (with F. Chung).
Publications. Books:
Additive Combinatorics (with T. Tao), 512 pages, Cambridge Univ. Press, 2006. Articles:
2011 (with Tao) Random matrices: Universality of local statistics (Acta Mathematica). 2010
(with Erdos et. al.) Bulk universality for Wigner hermitian matrices with subexponen- tial decay (Mathematics Research Letters).
(with T. Tao) Random matrices: The distribution of the smallest singular values (Uni- versality at the hard Edge) (To appear in CiAHAi
(with T. Tao) Random matrices: Universality of ESD and the Circular Law (with an appendix by M. Krishnapur) (Annals of Probabiliy)
(with H. Nguyen) Squares in Sumsets (Szemeredi is 70 proceeding).
(with K. Costello) Concentration of random determinants and permanent estimator (Siam Discrete mathematics).
2009
(with L. Tran and p. Wood) On a conjecture of Alon, J. Number Theory 129 (2009), no. 11,2801-2807.
(with p. Wood) The inverse Erd?s-Heilbronn problem, Electron. J. Combin. 16 (2009), no. 1,8 pages.
(with T. Tao) From the Littlewood-Oord problem to the cừcular law: universality of the spectral distribution of random matrices, Bull. Amer. Math. Soc. 46 (2009), no. 3, 377-396.
(with H. Nguyen) Classication theorems for sumsets modulo a prime, J. Combin. The- ory Ser. A 116 (2009), no. 4, 936-959.
(with T. Tao) Inverse Littlewood-Oord theorems and the condition number of random discrete matrices, Ann. of Math. (2) 169 (2009), no. 2, 595-632. 2 (with T. Tao) On the permanent of random Bemoulli matrices, Adv. Math. 220 (2009), no. 3, 657-669.
2008.
A structural approach to subset-sum problems. Building bridges, 525-545, Bolyai Soc. Math. Stud., 19, Springer, Berlin, 2008.
(with B. Sudakov) Local resilience of graphs, Random Structures Algorithms 33 (2008), no. 4, 409-433.
Random discrete matrices. Horizons of combinatorics, 257-280, Bolyai Soc. Math. Stud., 17, Springer, Berlin, 2008.
(with K Costello) The rank of random graphs, Random Structures Algorithms 33 (2008), no. 3, 269-285.
(with T. Tao) John-type theorems for generalized arithmetic progressions and iterated sumsets, Adv. Math. 219 (2008), no. 2, 428-449.
(with A. Johansson and J. Kahn) Factors in random graphs Random Structures and Algorithms, 33 (2008) no 1, 1-28.
(with T. Tao) Random matrices: The circular law, Communications in Contemporary Mathematics, 10 (2008), 261-307.
(with J. Solymosi) Near optimal bounds for the Erdos distinct distances problem in high dimensions, Combinatorica 28 (2008) 113-125. Sum-product estimates via di- rected expanders, Math. Res. Lett. 15 (2008), no. 2, 375-388.
(with R. Richardson and L. Wu) An inscribing model for random polytopes, Discrete Comput. Geom. 39 (2008), no. 1-3, 469-499.
(with E. Szemeredi and H. Nguyen) Sumset modulo p, Acta Arithmetica (2008), no. 4, 303-316.
2007.
(with I. Barany) Central limit theorems for Gaussian polytopes, Ann. Probab. 35 (2007), no. 4, 1593-1621.
Spectral norm of random matrices, Combinatorica 27 (2007), no. 6, 721-736.
(with T. Tao) On the singularity probability of random Bemoulli matrices, Joumal of the A. M. s, 20 (2007), 603-673.
Some new results 011 subset sums, J. Number Theory 124 (2007), no. 1, 229-233. (with J H. Kim and B. Sudakov) Small subgraphs of random regular graphs, Discrete Math. 307 (2007), no. 15, 1961-1967.
2006.
Central limit theorems for random polytopes in a smooth convex set, Advances in Mathematics 207 (2006) 221-243.
(with T. Tao) Qn random (-1,1) matrices: Singularity and Determinant, Random Struc- tures and Algorithms 28 (2006), no 1, 1-23.
(with Szemeredi) Long Arithmetic Progressions in Sumsets: Thresholds and Bounds, Joumal of the A.M.S, 19 (2006), no 1, 119-169.
(with J. H. Kim) Generating random regular graphs, Combinatorica 26 (2006), no. 6, 683-708.
(with K. Costello and T. Tao) Random symmetric matrices are almost surely nonsin- gular, Duke Math. J. 135 (2006), no. 2, 395-413.
(with E. Szemeredi) Finite and Infinite Arithmetic Progressions in Sumsets, Annals of Mathematics, 163 (2006), no 1, 1-35.
2005
(with L. Wu) Improving the Gilbert-Varshamov bounds for Sumsets: q-ary codes, IEEE Transaction in lnformation Theory 51 (2005), no 9, 3200-3208.
(with B. Sudakov and E. Szemeredi) On a problem of Erdos and Moser, Duke Math. Joumal, 129 (2005), no 1, 129-155.
(with J.H. Kim and J. Matousek) Discrepancy after adding a single set, Combinatorica
25 (2005), no 4, 499-501. 3
(with T. Szabo) k-wise intersecting theorems , Graphs and Combinatorics, 21 (2005), 147-161.
(with B. Sudakov and T. Szabo) A generalization of Turan theorem, Joumal of Graph Theory 49 (2005), 187-195.
Covering codes of arbitrary alphabets, Advances in Applied Math. 34 (2005), 65-70. (with E. Szemeredi) Long arithmetic progressions in sumsets and the number of x-free sets, Proceed- ing of London Math. Society, 90 (2005), 273-296.
Sharp concentration of random polytopes, CiAHA (2005), no 6, 1284-1328.
2004.
(with J. Solymosi) Distinct distances in high dimensional homogeneous sets: Towards a theory of geometric graphs,Contemp. Math. 342, AMS 259-263.
(with J.H. Kim) Sandwiching random graphs, Advances in Mathematics 188 (2004) 444-469.
(with J.H. Kim) Devide and Conquer Martingales and the number of triangles in a random graph, Random Structures and Algorithms 24 (2004), no. 2, 166-174.
2003.
(with H. Q Ngo) Clos networks and a generalized edge-coloring problem 011 bipartite graphs, S1AM J. Comput. 32 (2003), no. 4, 1040-1049.
(with J.H. Kim) Small complete arcs in projective planes, Combinatorica 23 (2003), no. 2,311-363.
(with T. Szabo) Turan's theorem for sparse random graphs, Random Structures and Algorithms, 23 (2003), no. 3, 225-234.
(with F. Chung and L. Lu) Eigenvalues of Power Law Graphs, Annals of Combinatoris 7 (2003), 21-33.
(with F. Chung and L. Lu) The spectra of random graphs with expected degrees, Pro- ceedings of National Academy of Sciences, 100, no. 11, (2003).
(with J.H. Kim) Generating random regular graphs (2003), STOC 2003 213-222. (with M. Krivelevich and B. Sudakov) Covering codes with improved density, IEEE Transaction 011 lnformation Theory, 49 (2003) 1812-1815.
(with M. Krivelevich, B. Sudakov and N.Wormald) Qn the probability of independent sets in random graphs, Random Structures and Algorithms (2003) Nol, 1-14.
(with N. Alon, B. Bollobas and J.H. Kim) Economical covers and geometric applica- tions, Proc. London Math. Soc. (3) 86 (2003) 273-301.
2002.
On sum of dependent random variables and applications in additive number theory, Number theory for the millennium, III (Urbana, IL, 2000),341-356, A K Peters, Nat- ick, MA, 2002.
On a problem of Gowers, Annals of Combinatorics (2002) 229-233.
An upper bound 011 the list chromatic number of locally sparse graphs, Combinatorics, Probability and Computing 11 (2002), 103-111.
(with M. Krivelevich and B. Sudakov) Sharp threshold of reliability, Combinatorics, Probability and Computing 11 (2002), 465-474.
(with J. H. Kim and B. Sudakov) On the asymmetry of random graphs and random regular graphs, Random Structures and Algorithms 21(2002), 216-224.
Concentration of non-Lipschitz functions and applications, Random Structures and Al- gorithms, 20 (3) (2002), 262- 316.
(with N. Alon and M. Krivelevich) Concentration of eigenvalue of random matrices, Israel Math. Joumal, 131 (2002), 259-267. High order complementary bases of primes, Integer 2 (2002).
( with M. Krivelevich) Approximating the independent number and the chromatic num- ber in expected polynomial time, Joumal of Comb. Optimization (2002) 143-155. 2001.
A large deviation result 011 the number of small subgraphs of a random graph, Com- binatorics, Probability and Computing, 10 (2001), no. 1, 79-94.
(with M. Kirivelevich, B. Sudakov and N. Wormald) Random regular graphs of high degree, Random Structures and Algorithms 18(2001), 346-363.
(with M. Krivelevich) The weak choice number of random hypergraphs, Joumal of Combinatorial Theory, Series B, 83 (2001), no. 2, 241-257.
(with I. Pak) On mixing of certain random walks, cuto phenomenon and Sharp thresh- old of random matroid processes, Discrete Applied Mathematics, 110 (2001), 251-272. 2000.
New bounds 011 nearly perfect matchings in hypergraphs:higher codegrees do help,Random Structures and Algorithms, 17 (1)(2000), 29-63.
(with J.H. Kim) Concentration of multi-variate polynomials and its applications, Com- binatorica, 20 (3) (2000), 417-434.
On the concentration of multi- variate polynomials with small expectation, Random Structures and Algorithms, 16 (4) (2000), 344-363.
On the choice number of random hypergraphs, Combinatorics, Probability and Com- puting 9, (2000), 79-95.
(with J. Kahn, J.H. Kim and L. Lovasz) The cover time, the blanket time, and the Matthews bound, FOCS (2000), 467-476.
On arenement of Waring's problem, Duke Math. Joumal,105, (1 )(2000), 107-134.
1999.
On some degree conditions which guarantee the upper bound of chromatic (choice) number of random graphs, Joumal of Graph Theory, 31, (1999), no. 3, 201-226.
( with c. Borg, J. Chayes, A.Frieze, J.H. Kim, p. Tetali and E. Vigoda, Torpid mixing of some MCMCalgorithms in statistical physics, FOCS (1999), 218-229.
Set systems with weakly restricted intersections,Combinatorica 19, (1999), no. 4, 567¬587.
1998 and earlier (prior to mv graduation).
On the infeasibility of training neural networks with small mean squared error, IEEE Transaction 011 lnformation Theory 44, (1998), no. 7, 2892-2900.
On a theorem of Ganter, Combinatorics,Probability and Computing 6, (1997), no. 2, 247-254.
(with D.Kozlov) Coins and Cones, J. of Combinatorial Theory, series A, 78 (1997), no. 1, 1-14.
(with Noga Alon)Anti- Hadamard matrices, coin weighing, threshold gates and in- decomposable hypergraphs, J. of Combinatorial Theory, series A, 79, (1997), no. 1, 133-160.
Extremal set systems with upper bounded odd intersections, Combinatorics, Probabil- Ếứy and Com- puting 13, (1997), no. 2, 197-208.
Small strongly regular r-full graphs, Combinatorica 16,(1996), no. 2, 295-299.
On the embedding of graphs into graphs with few eigenvalues, J. of Graph Theory, 22 (1996), no. 2, 137-149.
List of invited presentations
Szemeredi 70 workshop, Budapest, August 2010.
Workshop in Pseudorandomness, 1AS, June 2010 (P. Samak, A. Wigderson, J. Bour- gain).
Seminar in Probability, Stanford, April 2010 (A. Dembo).
Colloquium, UCSD, April 2010 (J. Balogh).
Colloquium, usc, April 2010 (S. Friedlander).
Seminar in Analysis, UCLA, April 2010 (T. Tao).
Seminar in Probability, NYU, Feb 2010.
AMS-KMS joint meeting, Plenary speaker, Seoul, Dec 2009. 5 Colloquium, Princeton, Dec 2009 (S. Klainerman).
Colloquium, Yale, Oct 2009 (P. Jones). Colloquium, Lehigh, Oct 2009 (J. Yukich). Seminar (Probability), Harvard, Oct 2009 (H-T Yau). Brandeis-Harvard-MIT-Northwestem Colloquium, Oct 2009 (M. Adler).
Workshop at IPAM (Combinatorics), October 2009 (Sudakov el. al.)
Colloquium, usc, October 2009 (S. H. Teng).
Workshop in Probabilistic Combinatorics, Bank, August 2009 (Sudakov et. al.) Colloquium, NTU (Singapore), July 2009 (Opening of the School of Sciences lecture). Colloquium, Hanoi University, June 2009 (H. M. Le).
Applied Math. Colloquium, MIT, May 2009 (A. Edelman).
Workshop 011 Probabilistics Combinatorics, Montreal , May 2009 (B. Reed et. al.). Columbia-Princeton Probability Day, Plenary Speaker, May 2009.
Dimacs workshop 011 property testing, Plenary Speaker, Spring 2009.
Seminar (Combinatorics), Princeton, Spring 2009 (J. Fox).
Seminar (Combinatorics), Columbia, Spring 2009 (M. Chudnovsky).
Seminar (Geometry), NYU, Spring 2009 (R. Pollack).
Colloquium, Emory, Oct 2008 (V. Rodl).
Colloquium, CMU, Oct 2008 (A. Frieze).

Bulding Bridges (Lovasz birthday conference), Renyi Institute, Budapest, August 2008. Seminar (Probability), Microsoữ Research, June 2008 (D. Wilson). Workshop 011 Com- binatorial Number Theory, CUNY, May 2008 (M. Nathanson).
Seminar (Probability), NYU, April 2008.
Workshop 011 Additive combinatorics, Fields Institute (Toronto), April 2008 (I. Laba et. al.).
AMS meeting in Bloomington, Probability Section, April 2008 (R. Lyons et. al.). Workshop 011 expanders, IPAM (UCLA), Feb 2008 (A. Wigderson et. al.). Oberwolfach meeting 011 Combinatorics, Plenary Speaker, Jan 2008 (J. Kahn et. al.) Colloquium, NYU, Nov 2007 (A. Venkatesh).
Seminar (Discrete Math), Princeton, Nov 2007 (P. Seymour). Plenary talk, 3rd Integer Conference, West Georgia, Oct 2007. Minicourse (3 lectures) 011 additive combina- torics and random matrices, IAS, Oct 2007.
Colloquium, Hanoi Institute of Mathematics, Hanoi, Vietnam, May 2007 (T. Ngo). Erdos lecture series (3 talks), Hebrew Univ., May 2007 (G. Kalai et. al.). Plenary talk, 13rd Conference 011 Random structures and Algorithms, Tel Aviv, June 2007 (Alon et. al.)
Workshop 011 Random matrices, Utah, June 2007 (B. Rider).
Seminar(Discrete Math/TCS), IAS, Spring 2007 (A. Razborov).
Seminar (Discrete Mathematics), Princeton, Nov 2006 (B. Sudakov).
Seminar (Computer Science), Yale, Nov 2006 (D. Spielman).
Seminar (Probability), CUNY, Oct 2006 (J. Rosen) Workshop 011 Large scale Graphs, Dimacs, Oct 2006 (Lovasz-Sudakov).
Planery talk, Horizons in Combinatorics, Balaton Lake, July 2006 (Lovasz et. al.)
6 Minicourse (2 lectures) 011 Littlewood-Oord problem, Horizons in Combinatorics, Budapest, July 2006 (Lovasz et. al.) Minicourse (3 lectures) 011 random polytopes, Phenomena in High Dimensions, Paris, June 2006 (Milman et. al.). Annual confer- ence 011 Phenomena in High Dimension, Paris, June 2006 (Milman et. al.).
Colloquium, Rutgers, Spring 2006. Minicourse (4 lectures) 011 sumsets and arithmetic progressions,
Workshop 011 Additive Combinatorics, Montreal, April 2006 (A. Granville, J. Soly- mosi).
Seminar (Discrete Mathematics), IAS, Spring 2006 (A. Razborov).
Workshop 011 Probalistic combinatorics, Dimacs , April 2006 (Nesetril et. al.)
Seminar (Probability), NYU, March 2006 (Dubedat).
Colloquium, Tech. Univ. Berlin, Berlin, Jan 2006 (G. Ziegler). Planery speaker, Bian- nual meeting 011 combinatorics, Oberwolfach, Jan 2006 (L. Lovasz and H.J. Prommel). Workshop 011 Algebraic combinatorics, Dimacs, Nov 2005 (J. Nesetril, F. Roberts). Seminar (Discrete Mathematics), Princeton, Oct 2005 (M. Chudnovsky).
Seminar (Discrete Mathematics), Microsoít Research, Summer 2005 (J. H. Kim). Seminar (Discrete Mathematics), Ohio State, January 2005 (N. Robertson).
Seminar (Discrete Mathematics), IAS, December 2004 (A. Razborov).
Seminar, Theory Group, Microsoữ Research, November 2004 (A. Naor).
Seminar (Discrete mathematics), Yale, September 2004 (G. Kalai).
Colloquium, Rutgers, September 2004 (J. Kahn).
Seminar (Discrete Mathematics), Hanoi Institute of Mathematics, August 2004 (T. Ngo).
Workshop in Convex Analysis, Bank Institute, July 2004 (organizer V. Milman et. al.). 2nd South American congress of Mathematicians, Cancun, June 2004) (thematic speaker, Combinatorics Section).
Conference Siam Discrete Mathematics, Nashville, June 2004, mini-symposium in Probabilistic Combinatorics (T. Bohman and B. Sudakov).
Conference Siam Discrete Mathematics, Nashville, June 2004, mini-symposium in Ex- tremal Combinatorics (D. Mubayi).
Seminar (Discrete mathematics), Microsoữ Research, June 2004 (L. Lovasz). Conference in Additive Number Theory, May 2004, CUNY (M. Nathanson). Seminar (Geometry), NYU, May 2004 (G. Pollack).
Colloquium, Indiana, April 2004 (R. Lyons).
Colloquium, Simon Hraser. March 2004 (L. Goddyns).
Seminar (Discrete Mathematics), UBC, March 2004 (J. Solymosi).
Joint Cal Tech-UCLA Analysis seminar, Feb 2004 (T. Tao)
Seminar (Discrete Mathematics), Hanoi Insitute of Mathematics, Jan 2004 (T. Ngo). Planery speaker, Biannual conference in Combinatorics, Oberwolfach, Jan 2004 (or- ganizer L. Lovasz and H. Promel).
Colloquium, Claremont College Community, October 2003 (M. 0'Neil).
Planery Speaker, Structural and Probabilistic Methods in Coloring, Bank, September
2003 (B. Reed and p. Seymours).
Seminar (Discrete mathematics), Microsoữ Research, August 2003 (J.H. Kim).
7 Seminar (Analysis), UCLA, May 2003 (T. Tao).
Colloquium, UCR, April 2003 (M-C. Chang). Planery Speaker, CombiTexas, Texas, April 2003 (C. Yan).
Seminar (Discrete mathematics and Theoretical Computer Science), Rutgers Univ, March 2003 (M. Saks).
Seminar (Discrete mathematics and Theoretical Computer Science), IAS, March 2003 (A. Razborov).
Seminar (Discrete mathematics and Theoretical Computer Science), Princeton Univ., March 2003 (B. Sudakov).
Seminar (Probability), UCLA, February 2003 (M. Biskup).
Workshop 011 Discrete Geometry, Dimacs, Rutgers, October 2002 (J. Pach).
S0IAM Discrete Mathematics Biennial meeting, mini-symposia 011 Probabilistic Com- binatorics, San Diego, August 2002.
Workshop 011 Geometric Phenomena of Large Dimension, Vancouver, July 2003 (M. Krivelevich, V. Milman, L. Lovasz).
Workshop 011 Measure "từ cấm", Vancouver, July 2003.
Seminar (Discrete Mathematics and Theoretical Computer Science), Princeton Univ., March 2002 (B. Sudakov).
Seminar (Discrete Mathematics and Theoretical Computer Science), IAS, March 2002
B. Sudakov).
Seminar (Discrete Mathematics), March 2002 (I. Pak).
Special Section 011 Probability and Combinatorics, AMS meeting, Atlanta, March 2002 (R. Lyons).
Seminar (Geometry), Hanoi Institute of Mathematics, January 2002 (T. Ngo).
Seminar (Discrete Mathematics), Microsoữ Research, Redmond, January 2002 (J.H. Kim).
Special Section 011 Probability and the Internet Graph, AMS annual meeting, San Diego, January 2002.
Oberwolfach workshop 011 combinatorics, Germany, January 2002 (H. Prommel and L. Lovasz).
Oberwolfach workshop 011 Lnite geometry, Germany, December 2001 (A. Blockhuis). Colloquium talk, University of Illinois at Urbana-Champaign, February 2001 (D. West). Colloquium talk, University of Washington, February 2001 (V. Klee).
Colloquium talk, Camegie Melon University, November 2000 (A. Frieze).
Colloquium talk, Georgia Technical Institute, November 2000 (D. Randall). Colloquium talk, University of Michigan at Ann Arbor, October, 2000 (A. Barvinok). Probabilistic Methods in Combinatorial Optimization, BRICS, Aarhus, Denmark, Au- gust 2000 (A. Panconesi).
Research program 011 complexity, 1AS, Princeton, August 2000.
Search and Communication Complexity, Budapest, Hungary, July 2000 (G. Katona). lOth SIAM
Conference in Discrete Mathematics, Mini-symposia in coloring, Minnesota, June
2000 (M. Albertson).
Millennial Conference 011 Number Theory, Illinois, May 2000 (H. Halberstam). Probabilistic Method, Fields Institute, Toronto, Canada, February 2000 (M. MolloyB. Reed).
Combinatorics, Oberwolfach, Germany, January 2000 (H. Prommel and L. Lovasz). Probabilistic Analysis of Hard Problems, DIMACS, New Jersey, November 1999. Special section 011 the Probabilistic Method, Annual AMS meeting, Texas, January
1999 (B. Bollobas J.H. Kim).
Workshop 011 Randomized and Derandomized algorithms for Discrete Structures, In- stitute for Advance Study, November 1998.
Combinatorists of New England, Smith College, 1998 (M. Albertson).
Workshop 011 analysis and combinatorics, Stockholm, Sweden, May 1995 (A. Bjoner). Workshop 011 combinatorics and optimization, Budapest, Hungary, 1994 (R. Andras). Workshop 011 combinatorics and optimization, Comell 1992 (R. Andras).
Contributed Talks RANDOM 2009, Berkeley, August 2009.
STOC 2007, San Diego, June 2007.
STOC 2005 (2 lectures 011 random matrices), Baltimore, May 2005.
STOC, San Diego, June 2003.
FOCS, Redondo Beach, November 2001. SIAM Discrete Mathematics, Toronto 1998. FOCS, Burlington, 1997.


Tài liệu tham khảo
1) Hàm Châu, Sau Ngô Bảo Châu là nhà toán học Vũ Hà Văn?.
2) Google.
3) Vũ Quần Phương, Đợi



#323520 Về một bài toán bất đẳng thức

Gửi bởi Ban Biên Tập trong 08-06-2012 - 22:01

Bài toán 1.21 (Nguyễn Anh Khoa). Cho $a, b, c$ là ba số thực dương. Chứng minh bất đẳng thức
$$max \begin{Bmatrix} (a - 1)^2, (b - 1)^2, (c - 1)^2 \end{Bmatrix} \geq \frac{2}{3}(1 - a)(1 - b)(1 - c).$$
Lời Giải. Sử dụng bất đẳng thức
$$max \begin{Bmatrix} (a - 1)^2, (b - 1)^2, (c - 1)^2 \end{Bmatrix} \geq \frac{(a-1)^2+(b-1)^2+(c-1)^2}{3},$$
ta đưa bài toán về chứng minh bất đẳng thức mạnh hơn là
$$(a - 1)^2 + (b - 1)^2 + (c - 1)^2 \geq 2(1 - a)(1 - b)(1 - c),$$
tương đương với
$$a^2 + b^2 + c^2 + 2abc + 1 \geq 2(ab + bc + ca).$$
Bất đẳng thức cuối cùng hiển nhiên đúng. Bài toán được chứng minh xong. Đẳng thức xảy ra khi và chỉ khi a = b = c = 1 □


Bài toán 1.22 (Vasile Cirtoaje). Cho $a, b, c$ là ba số thực dương. Chứng minh rằng
$$4 (a+\frac{1}{a})(b+\frac{1}{b}) (c+\frac{1}{c}) \geq 9(a+b+c).$$
Lời Giải. Ta đặt $a=\frac{\sqrt{2}}{x}, b=\frac{\sqrt{2}}{y}, c=\frac{\sqrt{2}}{z}$ Bất đẳng thức cần chứng minh được viết lại như sau
$$(x^2 + 2)(y^2 + 2)(z^2 + 2) \geq 9(xy + yz + zx).$$
Đây chính là bất đẳngthức (1.5.1), nên ta có điều phải chứng minh.
Đẳng thức xảy ra khi và chỉ khi $a = b = c = \sqrt{2}$. □


Bài toán 1.23 (Phạm Kim Hùng). Cho $a, b, c, d$ là bốn số thực không âm thỏa mãn điều kiện $a + b + c + d = 4$. Chứng minh bất đẳng thức
$$(a^2 + 2)(b^2 + 2)(c^2 + 2)(d^2 + 2) \geq 81$$
Lời Giải. Không mất tính tổng quát của bài toán, ta có thể giả sử d = min{a, b, c, d}, khi đó ta có 0 - d - 1. Sử dụng bất đẳng thức (1.6.1), ta được
$$(a^2 + 2)(b^2 + 2)(c^2 + 2)(d^2 + 2) \geq 3(a + b + c)^2(d^2 + 2) = 3(4 - d)^2(d^2 + 2).$$
Mặt khác, ta lại có
$$3(4 - d)^2(d^2 + 2) - 81 = 3(1 - d)^3(5 - d) \geq 0, $$
vì vậy mà ta được
$$(a^2 + 2)(b^2 + 2)(c^2 + 2)(d^2 + 2) \geq 81.$$
Bài toán được chứng minh xong. Đẳng thức xảy ra khi và chỉ khi $a = b = c = d = 1.$


Bài toán 1.24 (Nguyễn Văn Huyện). Với mọi số thực $a, b, c$ thay đổi bất kỳ thỏa mãn điều kiện
$$\frac{1}{a^2+b^2+4}+\frac{1}{b^2+c^2+4}+\frac{1}{c^2+a^2+4} \geq \frac{2}{3}$$
hãy chứng minh bất đẳng thức
$$ab + bc + ca \leq \frac{3}{4}$$
Lời Giải. Do $a^2 + b^2 \geq \frac{(a+b)^2}{2}$ , nên từ giả thiết, ta có
$$\frac{1}{(a+b)^2+8}+\frac{1}{(b+c)^2+8}+\frac{1}{(c+a)^2+8} \geq \frac{1}{3}$$
Đặt $x = a + b, y = b + c, z = c + a$, ta có thể viết bất đẳng thức (1.24.1) lại thành
$$\frac{1}{x^2+8}+\frac{1}{y^2+8}+\frac{1}{z^2+8} \geq \frac{1}{3}$$
Bằng cách sử dụng kết quả "Bài toán 1.16", ta có ngay $x + y + z \leq 3$, hay là
$$a + b + c \leq \frac{3}{2}.$$
Mặt khác, theo bất đẳng thức quen thuộc $(a + b + c)^2 \geq 3(ab + bc + ca)$, ta lại có
$$\frac{9}{4} \geq (a + b + c)^2 \geq 3(ab + bc + ca),$$
từ đó suy ra được
$$ab + bc + ca \leq \frac{3}{2},$$
Bài toán được chứng minh xong. Đẳng thức xảy ra khi và chỉ khi $a = b = c = \pm 1$.


Bài toán 1.25. Cho $x,y,z$ là ba số thực không âm. Chứng minh bất đẳng thức
$$x^2 + y^2 + z^2 + \sqrt{3xyz(x + y + z)} \geq 2(xy + yz + zx).$$

Lời Giải. Bài toán có được bằng cách cộng ngược chiều hai bất đẳng thức sau đây
$$x^2 + y^2 + z^2 \geq (xy + yz + zx),$$
$$xy + yz + zx \geq \sqrt{3xyz(x + y + z)}.$$
Nếu $xyz = 0$ thì bài toán trở nên tầm thường nên ta chỉ cần xét $xyz \geq 0$. Để ý rằng bất đẳng thức cần chứng minh ở dạng thuần nhất nên ta có thể chuẩn hóa cho $xyz = 1$, và viết nó lại như sau
$$x^2 + y^2 + z^2 + \sqrt{3(x + y + z)} \geq 2(xy + yz + zx).$$
Mặt khác, theo bất đẳng thức AM-GM, thì
$$x + y + z \geq 3.$$
Vì thế để chứng minh bài toán thì ta cần chứng minh được
$$x^2 + y^2 + z^2 + 3 \geq 2(xy + yz + zx),$$
hay là
$$x^2 + y^2 + z^2 + 2xyz + 1 \geq 2(xy + yz + zx).$$
Đây chính là bất đẳng thức (1) nên ta có điều phải chứng minh. Trong trường hợp tổng quát, ta có đẳng thức xảy ra khi và chỉ khi $x = y = z$ hoặc $x = y, z = 0$ cùng các hoán vị. □


Bài toán 1.26. Cho $a, b, c$ là ba số thực dương. Chứng minh rằng
$$1+\frac{a}{b}+\frac{b}{c}+\frac{c}{a} \geq 2\sqrt{1+\frac{a}{c}+\frac{b}{a}+\frac{c}{b}}.$$
Lời Giải. Đặt
$$x=\frac{a}{b}, y=\frac{b}{c}, z=\frac{c}{a}$$
thì $x, y, z$ là các số dương và $xyz = 1$, khi đó bất đẳng thức trở thành
$$1+ x + y + z \geq 2\sqrt{1+\frac{1}{x}+\frac{1}{y}+\frac{1}{z}}.$$
hay là
$$1 + x + y + z \geq 2\sqrt{1 + yz + zx + xy.}$$
Bình phương hai vế và thu gọn lại, ta được
$$x^2 + y^2 + z^2 + 2(x + y + z) \geq 3 + 2(xy + yz + zx).$$
Vì $xyz = 1$ nên theo bất đẳng thức AM-GM, ta có
$$x + y + z \geq 3,$$
Vì thế để chứng minh bất đẳng thức trên thì ta cần chứng minh được
$$x^2 + y^2 + z^2 + x + y + z \geq 2(xy + yz + zx).$$
Đây chính là bất đẳng thức $(1.1.1)$, nên ta có điều phải chứng minh. □


Bài toán 1.27 (Trần Nam Dũng). Tìm số thực $k$ lớn nhất sao cho bất đẳng thức sau đúng với mọi $a, b, c$ dương
$$(a^2 + 2)(b^2 + 2)(c^2 + 2) \geq k(a^2 + b^2 + c^2) + (9 - k)(ab + bc + ca).$$
Lời Giải. Cho $a = b = 0,$ khi đó ta sẽ được
$$4(c^2 + 2) \geq kc^2.$$
Từ đó dễ dàng suy ra $k \leq 4$. Với $k = 4$, ta được bất đẳng thức
$$(a^2 + 2)(b^2 + 2)(c^2 + 2) \geq 4(a^2 + b^2 + c^2) + 5(ab + bc + ca),$$
tương đương với
$$a^2b^2c^2 + 2(a^2b^2 + b^2c^2 + c^2a^2) +8 \geq 5(ab + bc + ca), \ \ (1.27.1)$$
Đặt $x = ab, y = bc, z = ca$ thì bất đẳng thức $(1.27.1)$ trở thành
$$xyz + 2(x^2 + y^2 + z^2) + 8 \geq 5(x + y + z).$$
Đây chính là bất đẳng thức $(1.12.1)$ nên nó hiển nhiên đúng. Vậy $k = 4$, là giá trị lớn nhất cần tìm. □


Qua những ví dụ trên chắc hẳn các bạn cũng đã phần nào thấy được ứng dụng rộng rãi của bất đẳng thức (1), một bài toán tuy đơn giản nhưng những ứng dụng của nó thì thì quả thật không đơn giản tí nào. Chúng ta hãy cùng xem lại bài toán này một lần nữa Cho a, b, c là các số thực dương thay đổi bất kỳ. Chứng minh rằng
$$a^2 + b^2 + c^2 + 2abc + 1 \geq 2(ab + bc + ca).$$


Từ bài toán trên ta có thể sáng tạo ra hàng loạt bài toán liên quan khác, khó hơn như các bạn đã thấy. Đó chính là những bí ẩn sau vẻ đẹp của mỗi bài toán.

Trong bất đẳng thức điều kiện để một bất đẳng thức trở thành một bài toán hay thì điều đầu tiên là nó phải đẹp không có điều kiện rắc rối phức tạp và ở dạng chuẩn và ta phải Giải quyết được bài toán tổng quát của nó. Bất đẳng thức (1) cũng giống như thế, ta cũng có kết quả tổng quát của nó như sau

Tổng Quát. Với $x_i,x_2, ...,x_n (n \geq 2)$ là các số thực không âm, khi đó ta luôn có
$$(n - 1)(x_1 + x_2 + ... + x_n) + 2x_1x_2...x_n + n - 2 \geq (x_1 + x_2 + ... + x_n)^2$$



#322066 Về một bài toán bất đẳng thức

Gửi bởi Ban Biên Tập trong 03-06-2012 - 16:45

Bài toán 1.13. (Algebraic Inequalities Old and New Methods)
Cho x, y, z là ba số thực không âm thỏa mãn điều kiện $xy + yz + zx = 3$. Chứng minh bất đẳng thức
$$\frac{1}{x^2+2}+\frac{1}{y^2+2}+\frac{1}{z^2+2} \leq 1.$$
Lời Giải. Ta có bất đẳng thức cần chứng minh tương đương với
$$(x^2 + 2)(y^2 + 2)(z^2 + 2) \geq (x^2 + 2)(y^2 + 2) + (y^2 + 2)(z^2 + 2) + (z^2 + 2)(x^2 + 2).$$
Khai triển trực tiếp ra, ta được $$x^2y^2 + y^2z^2 + z^2x^2 + x^2y^2z^2 \geq 4.$$
Đặt $a = xy, b = yz$ và $c = zx$ thì $a + b + c = 3.$ Bất đẳng thức cần chứng minh trở thành $$a^2 + b^2 + c^2 + abc \geq 4.$$
Đây chính là bất đẳng thức $(1.10.1)$, nên nó hiển nhiên đúng. Bài toán được chứng minh xong. Đẳng thức xảy ra khi và chỉ khi $a = b = c = 1$, tức là $x = y = z = 1.$

Bài toán 1.14. Cho $a, b, c$ là ba số thực dương. Chứng minh bất đẳng thức:
\[\left( {{x^2} + 3} \right)\left( {{y^2} + 3} \right)\left( {{z^2} + 3} \right) \ge \frac{3}{4}{\left( {xy + yz + zx + \frac{{xyz}}{3}} \right)^2}\]
Lời Giải. Chia hai vế của bất đẳng thức cho $x^2y^2z^2$, ta có thể viết nó lại như sau
\[\left( {\frac{9}{{{x^2}}} + 3} \right)\left( {\frac{9}{{{y^2}}} + 3} \right)\left( {\frac{9}{{{z^2}}} + 3} \right) \ge 4{\left( {\frac{3}{x} + \frac{3}{y} + \frac{3}{z} + 1} \right)^2}\]
Đến đây bằng cách đặt $a = \frac{3}{x}, b = \frac{3}{y}$ và $c = \frac{3}{z}$, ta đưa bài toán về chứng minh
$$(a^2 + 3)(b^2 + 3)(c^2 + 3) \geq 4(a + b + c + 1)^2\,\,\,\,\,\, (1.14.1)$$
Khai triển trực tiếp bất đẳng thức này, ta được
$$5(a^2 + b^2 + c^2) + 3(a^2b^2 + b^2c^2 + c^2a^2) + a^2b^2c^2 + 23 \geq 8(a + b + c + ab + bc + ca)$$
Theo bất đẳng thức $(1.5.1)$ thì: $$a^2 + b^2 + c^2 + a^2b^2c^2 + 2 \geq 2(ab + bc + ca),$$
vì thế để chứng minh được bài toán ta cần chứng minh được
$$4(a^2 + b^2 + c^2) + 3(a^2b^2 + b^2c^2 + c^2a^2) +21 \geq 8(a + b + c) + 6(ab + bc + ca)$$
Bằng một số biến đổi đơn giản ta có bất đẳng thức này tương đương với
$$3 [(ab - 1)^2 + (bc - 1)^2 + (ca - 1)^2] +4 [(a - 1)^2 + (b - 1)^2 + (c - 1)^2] \geq 0,$$
là một bất đẳng thức hiển nhiên đúng. Bài toán được chứng minh xong. Đẳng thức xảy ra khi và chỉ khi $a = b = c = 1$ hay $x = y = z = 3$.

Bài toán 1.15. (Nguyễn Văn Huyện)
Với $a, b, c$ là ba số thực dương. Chứng minh rằng khi đó với mọi số dương $k \geq 2$, ta luôn có
$$(a^2 + k)(b^2 + k)(c^2 + k) \geq (k + 1)(a + b + c + k - 2)^2\,\,\,\,\,\,\,\, (1.15.1)$$
Lời Giải. Sử dụng bất đẳng thức $(1.8.1)$, ta cần chứng minh bất đẳng thức mạnh hơn là
$$\frac{(k+1)^2}{3}(a + b + c)^2 + k^3 - 3k - 2 \geq (k + 1)(a + b + c + k - 2)^2.$$
Bất đẳng thức này tương đương với
$$\frac{(k+1)^2}{3}(a + b + c)^2 + (k + 1)^3 - 3(k + 1)^2 \geq (k + 1)(a + b + c + k - 2)^2$$
$$\frac{k+1}{3}(a + b + c)^2 + (k + 1)^2 - 3(k + 1) \geq (a + b + c + k - 2)^2$$
$$\frac{k+1}{3}(a + b + c)^2 + (k + 1)^2 - 3(k + 1) \geq (a + b + c + k - 2)^2$$
$$\frac{k + 1}{3}(a + b + c)^2 + k^2 - k - 2 \geq (a + b + c)^2 + 2(k - 2)(a + b + c) + (k - 2)^2$$
$$\frac{k-2}{3}(a + b + c)^2 + 3(k - 2) \geq 2(k - 2)(a + b + c).$$
Bất đẳng thức cuối cùng đúng theo bất đẳng thức AM-GM nên ta có điều phải chứng minh.

NHẬN XÉT. Trong $(1.15.1)$ nếu ta cho $k = 2$ thì ta được $(1.6.1)$ còn nếu ta cho $k = 3$ thì ta được $(1.14.1).$

Bài toán 1.16. Với $a, b, c$ là ba số thực bất kỳ sao cho $$\frac{1}{a^2+8}+\frac{1}{b^2+8}+\frac{1}{c^2+8}=\frac{1}{3}$$
Hãy tìm giá trị lớn nhất và giá trị nhỏ nhất của biểu thức: $P = a + b + c. $

Lời Giải. Ta viết biểu thức điều kiện của bài toán lại như sau
\[\frac{1}{{{a^2} + 8}} = \left( {\frac{1}{6} - \frac{1}{{{b^2} + 8}}} \right) + \left( {\frac{1}{6} - \frac{1}{{{c^2} + 8}}} \right)\]
hay là \[\frac{1}{{{a^2} + 8}} = \frac{1}{6}\left( {\frac{{{b^2} + 2}}{{{b^2} + 8}} + \frac{{{c^2} + 2}}{{{c^2} + 8}}} \right)\]
Từ đó theo bất đẳng thức AM-GM, ta được
\[\frac{1}{{{a^2} + 8}} = \frac{1}{6}\left( {\frac{{{b^2} + 2}}{{{b^2} + 8}} + \frac{{{c^2} + 2}}{{{c^2} + 8}}} \right) \ge \frac{1}{3}\sqrt {\frac{{({b^2} + 2)({c^2} + 2)}}{{{b^2} + 8)({c^2} + 8)}}} \,\,\,\,\,\,\,{\mkern 1mu} {\mkern 1mu} {\mkern 1mu} {\mkern 1mu} {\mkern 1mu} {\mkern 1mu} {\mkern 1mu} (1.16.1)\]
Hoàn toàn tương tự, ta cũng có
$$\frac{1}{b^2+8} \geq \frac{1}{3} \sqrt{\frac{(c^2+2)(a^2+2)}{(c^2+8)(a^2+8)}}\, \, \, \,\,\,\,\,\,\,\,\,\,(1.16.2)$$
$$\frac{1}{c^2+8} \geq \frac{1}{3} \sqrt{\frac{(a^2+2)(b^2+2)}{(a^2+8)(b^2+8)}}\, \, \, \,\,\,\,\,\,\,\,\,\,(1.16.3)$$
Nhân tương ứng ba bất đẳng thức $(1.16.1), (1.16.2)$ và $(1.16.3)$ lại với nhau ta được
$$27 \geq (a^2 + 2)(b^2 + 2)(c^2 + 2).$$
Mặt khác, theo bất đẳng thức $(1.6.1)$ thì $(a^2 + 2)(b^2 + 2)(c^2 + 2) \geq 3(a + b + c)^2.$

Nên từ đó suy ra $(a + b + c)^2 \leq 9$

hay là $$-3 \leq a + b + c \leq 3\,\,\,\,\,\,\,\,\, (1.16.4)$$
Bằng tính toán trực tiếp ta thấy $P = -3$ khi và chỉ khi $(a, b, c)=(-1, -1, -1)$ và $P = 3$ khi và chỉ khi $(a, b, c)=(1,1,1)$.

Việc tìm được các giá trị cụ thể của $a, b, c$ thỏa mãn giả thiết của bài toán đồng thời bất đẳng thức $(1.16.4)$ trở thành đẳng thức cho phép ta kết luận $P_{min} = -3$ và $P_{max} = 3$.

NHẬN XÉT. Ta có bài toán tổng quát của bất đẳng thức trên như sau:

Cho $a, b, c$ là ba số thực bất kỳ và $k \geq 2$ là một số dương cho trước thỏa mãn điều kiện
$$\frac{1}{a^2+3k+2} \frac{1}{b^2+3k+2} \frac{1}{c^2+3k+2}=\frac{1}{k+1}$$
khi đó hãy tìm giá trị lớn nhất và giá trị nhỏ nhất của biểu thức $P = a + b + c.$

Lời Giải. Tương tự như trên ta viết biểu thức điều kiện của bài toán lại như sau
\[\frac{1}{{{a^2} + 3k + 2}} = \frac{1}{{2(k + 1)}}\left( {\frac{{{b^2} + k}}{{{b^2} + 3k + 2}} + \frac{{{c^2} + k}}{{{c^2} + 3k + 2}}} \right)\]
Từ đó sử dụng bất đẳng thức AM-GM, ta được
$$\frac{1}{a^2+3k+2} \geq \frac{1}{k+1} \sqrt{\frac{(b^2+k)(c^2+k)}{(b^2+3k+2)(c^2+3k+2)}}.$$
Đánh giá tương tự cho hai bất đẳng thức còn lại sau đó nhân tương ứng theo vế lại với nhau, ta thu được
$$(k + 1)^3 \geq (a^2 + k)(b^2 + k)(c^2 + k).$$
Kết hợp với bất đẳng thức $(1.8.1)$, ta có
$$(k + 1)^3 \geq \frac{(k + 1)^2}{2} (a + b + c)^2 + k^3 - k - 2$$
Biến đổi đơn giản hai vế, ta được
$$(a + b + c)^2 - 9.$$
Từ đó suy ra $P_{min} = -3$ và $P_{max} = 3$.

Bài toán 1.17. (Nguyễn Anh Khoa)
Cho $a, b, c$ là ba số thực dương thỏa mãn điều kiện $2(a^2 + b^2 + c^2) + abc = 7$. Chứng minh bất đẳng thức
$$a + b + c \leq 3.$$
Lời Giải. Biểu thức điều kiện của bài toán có thể viết lại như sau
$$15 = 2(a^2 + b^2 + c^2) + [a^2 + b^2 + c^2 + (a^2 + b^2 + c^2 + 2abc + 1)].$$
Bằng cách sử dụng bất đẳng thức $(1)$ và bất đẳng thức quen thuộc $(a^2 + b^2 + c^2) \geq \frac{1}{3}(a + b + c)^2$, ta có
$$15 \geq \frac{2}{3}(a + b + c)^2 + [a^2 + b^2 + c^2 + 2(ab + bc + ca)] = \frac{5}{3}(a + b + c)^2$$
Từ đó bằng cách lấy căn bậc hai hai vế, ta được $a + b + c \leq 3.$

Bài toán được chứng minh xong. Đẳng thức xảy ra khi và chỉ khi $a = b = c = 1$.

NHẬN XÉT. Bằng cách làm tương tự ta chứng minh được bài toán tổng quát sau đây
Nếu $a, b, c$ và $k \geq 1$ là các số thực dương thỏa mãn điều kiện $k(a^2 + b^2 + c^2) + abc = 3k + 1$, thì
$a + b + c \leq 3.$

(Nguyễn Văn Huyện)

Lời Giải. Thật vậy, biểu thức điều kiện có thể được viết lại như sau
$$3(2k + 1) = (2k - 2)(a^2 + b^2 + c^2) + [a^2 + b^2 + c^2 + (a^2 + b^2 + c^2 + 2abc + 1)],$$
Từ đó sử dụng bất đẳng thức $(1)$ và bất đẳng thức cơ bản $a^2 + b^2 + c^2 \geq \frac{1}{3}(a + b + c)^2$, ta có
$$3(2k + 1) \geq \frac{2k-2}{3} (a + b + c)^2 + (a + b + c)^2 = \frac{2k+1}{3}(a + b + c)^2\,\,\,\,\,\,\,\,\, (1.17.1)$$
Bằng cách lấy căn hai vế, ta có ngay điều phải chứng minh.

Ngoài ra ta cũng có bài toán đảo của bất đẳng thức trên như sau.

Nếu $a, b, c$ là ba số thực dương thỏa mãn điều kiện $a + b + c = 3$, khi đó với mọi số thực $ k \geq 1$ ta luôn có bất đẳng thức
$$k(a^2 + b^2 + c^2) + abc \geq 3k + 1. $$
Lời Giải. Bất đẳng thức cần chứng minh tương đương với
$$(k - 1)(a^2 + b^2 + c^2) + (a^2 + b^2 + c^2 + abc) \geq 3k + 1$$
Dễ thấy, bài toán có được bằng cách cộng hai bất đẳng thức sau lại với nhau
$$a^2 + b^2 + c^2 \geq 3$$
$$a^2 + b^2 + c^2 + abc \geq 4.$$
Những đây là những kết quả mà ta đã biết.

Bài toán 1.18. Cho $a, b, $ là ba số thực dương. Chứng minh bất đẳng thức
$$3(a^2 + b^2 + c^2) + abc + 11 \geq 7(a + b + c).$$
Lời Giải. Bất đẳng thức cần chứng minh tương đương với
$$2 [(a - 1)^2 + (b - 1)^2 + (c - 1)^2] + [a^2 + b^2 + c^2 + abc + 5 - 3(a + b + c)] \geq 0$$
là một kết quả hiển nhiên đúng theo bất đẳng thức $(1.11.1)$, nên ta có điều phải chứng minh. Đẳng thức xảy ra khi và chỉ khi $a = b = c = 1$.

Bài toán 1.19. (Nguyễn Văn Huyện)
Với $a, b, c$ là ba số thực dương. Chứng minh rằng với mọi số thực dương $ k \geq 1$, ta luôn có bất đẳng thức
$$k(a^2 + b^2 + c^2) + abc + 3k + 2 \geq (2k + 1)(a + b + c)\,\,\,\,\,\,\,\,\,\,\, (1.19.1)$$
Lời Giải. Nhân $2$ vào hai vế của bất đẳng thức, ta có thể viết nó lại như sau
$$(2k - 2)(a^2 + b^2 + c^2) + [a^2 + b^2 + c^2 + (a^2 + b^2 + c^2 + 2abc + 1)] + 3(2k + 1) \geq 2(2k + 1)(a + b + c)\,\,\,\,\,\,\,\,\,\,(1.15.1)$$
Ở $(1.17.1)$, ta đã chứng minh được
$$(2k - 2)(a^2 + b^2 + c^2) + [a^2 + b^2 + c^2 + (a^2 + b^2 + c^2 + 2abc + 1)] \geq \frac{2k+1}{3} (a + b + c)^2.$$
nên để chứng minh bất đẳng thức trên thì ta chỉ cần chỉ ra được
$$\frac{2k+1}{3}(a + b + c)^2 + 3(2k + 1) \geq 2(2k + 1)(a + b + c)$$
Là bất đẳng thức hiển nhiên đúng theo bất đẳng thức AM-GM. Chứng minh hoàn tất.

NHẬN XÉT. Tập hợp tất cả các giá trị của $k$ để $(1.19.1)$ luôn đúng là $k \geq \frac{1}{\sqrt{2}}$, tuy nhiên với $k \geq 1$ thì ta nhận được lời giải bằng cách sử dụng bất đẳng thức $(1)$. Đặc biệt với $k = \frac{1}{\sqrt{2}}$ thì ngoài trường tầm thường $a = b = c =1$ để bất đẳng thức xảy ra thì ta còn có thêm trường hợp khác nữa là $a = b = 1 + \frac{1}{\sqrt{2}}, c = 0$ cùng các hoán vi.

Bài toán 1.20. Cho $a, b, c$ là ba số thực dương. Chứng minh bất đẳng thức
$$(a^2 + 1)(b^2 + 1)(c^2 + 1) + 4 \geq 4(ab + bc + ca) + (abc - 1)^2. $$
Lời Giải. Sử dụng bất đẳng thức AM-GM, ta có:
$$(ab + 1)^2 + (bc + 1)^2 + (ca + 1)^2 \geq 4(ab + bc + ca)$$
Từ đó đưa bài toán về chứng minh bất đẳng thức mạnh hơn là
$$(a^2 + 1)(b^2 + 1)(c^2 + 1) + 4 \geq (ab + 1)^2 + (bc + 1)^2 + (ca + 1)^2 + (abc - 1)^2,$$
Tương đương với
$$a^2 + b^2 + c^2 + 2abc + 1 \geq 2(ab + bc + ca).$$
Nhưng đây chính là bất đẳng thức $(1)$ nên nó hiển nhiên đúng, tức ta có điều phải chứng minh. Đẳng thức xảy ra khi và chỉ khi $a = b = c = 1$


Còn tiếp...


#321642 Tìm số hạng tổng quát của dãy số bằng hàm Hypebolic

Gửi bởi Ban Biên Tập trong 02-06-2012 - 07:58

TÌM CÔNG THỨC TÍNH SHTQ CỦA DÃY SỐ BẰNG HÀM HYPEBOLIC

DƯ QUỐC ĐẠT

Giáo viên trường THPT Nguyễn Hữu Cầu, huyện Hốc môn, TP Hồ Chí Minh


I. Lời nói đầu:

Bài toán tìm công thức tính số hạng tổng quát của một dãy số cho dưới dạng truy hồi là một bài toán rất đa dạng và thường gặp trong những đề thi Olympic, thi học sinh giỏi quốc gia, quốc tế. Có nhiều hướng tiếp cận bài toán này và có nhiều cách giải khác nhau.


Trong bài viết này tôi giải quyết bái toán bằng cách sử dụng các hàm hypebolic .

Tác giả rất mong nhận được sự góp ý của các đồng nghiệp qua địa chỉ email: [email protected].

II. Nội dung:


II.1. Giới thiệu các hàm Hypebolic.

$\bullet $ Hàm sin hypebolic định bởi: ${\mathop{\rm sh}\nolimits} x = \frac{{{e^x} - {e^{ - \,x}}}}{2}$ có tập xác định $\mathbb{R}$ và tập giá trị $\mathbb{R}$

$\bullet $ Hàm cos hyperbolic định bởi: ${\mathop{\rm ch}\nolimits} x = \frac{{{e^x} + {e^{ - \,x}}}}{2}$ có tập xác định $\mathbb{R}$ và tập giá trị $\left[ {1, + \infty } \right)$

$\bullet $ Hàm tan hyperbolic định bởi: ${{\mathop{\rm th}\nolimits} x} = \frac{{{\mathop{\rm sh}\nolimits} x}}{{{\mathop{\rm ch}\nolimits} x}} = \frac{{{e^x} - {e^{ - \,x}}}}{{{e^x} + {e^{ - \,x}}}}$ có tập xác định $\mathbb{R}$ và tập giá trị $\left( { - 1,1} \right)$

$\bullet $ Hàm cot hyperbolic định bởi: $\coth x = \frac{{{\mathop{\rm ch}\nolimits} x}}{{{\mathop{\rm sh}\nolimits} x}} = \frac{{{e^x} + {e^{ - \,x}}}}{{{e^x} - {e^{ - \,x}}}}$ có tập xác định $\mathbb{R}\backslash \left\{ 0 \right\}$ và tập giá trị $\left( { - \,\infty \,,\, - 1} \right) \cup \left( {1\,,\, + \,\infty } \right)$

Các tính chất sau bạn đọc có thể tự chứng minh:

1. $c{h^2}x - s{h^2}x = 1$

2. $ch\left( {x \pm y} \right) = chx.chy \pm shx.shy$

3. $sh\left( {x \pm y} \right) = chx.shy \pm chy.shx$

4. $sh\left( {2x} \right) = 2shx.chx$

5. $sh\left( {3x} \right) = 4s{h^3}x + 3shx$

6. $ch\left( {2x} \right) = c{h^2}x + s{h^2}x = 2c{h^2}x - 1 = 2s{h^2}x + 1$

7. $ch\left( {3x} \right) = 4c{h^3}x - 3chx$

8. $ch\left( {4x} \right) = 8c{h^4}x - 8c{h^2}x + 1 = 8s{h^4}x + 8s{h^2}x + 1$

9. $sh\left( {4x} \right) = 8s{h^3}x.chx + 4shx.chx = 8s{h^3}x\sqrt {1 + s{h^2}x} + 4shx\sqrt {1 + s{h^2}x} $

10. $th\left( {2x} \right) = \frac{{2thx}}{{1 + t{h^2}x}}$

11. $th\left( {3x} \right) = \frac{{t{h^3}x + 3thx}}{{1 + 3t{h^2}x}}$

II.2 Các dạng thường gặp.

1. Dạng 1: $\left\{ \begin{array}{l} \mathbf{\textbf{Cho}\,\,\,{u_1}}\\ \mathbf{{u_{n + 1}} = 2u_n^2 - 1;\,\,\forall n \in \mathbb{N^*}} \end{array} \right.$

Ta xét các trường hợp sau:

* Trường hợp 1:

Nếu ${u_1} = 1$ thì ta được ${u_n}=1,\,\,\,\forall n \in \mathbb{{N}^*}$

Nếu ${u_1} = -1$ thì ta được ${u_n}=1,\,\,\,\forall n \ge 2$

* Trường hợp 2:

Nếu $-1<{u_1}<1$ thì ta đặt ${u_1}= \cos\alpha;\,\,\,\,\alpha \in \left( {0,\pi } \right)$

Chứng minh quy nạp ta được: ${u_n}=\cos\left({{2^{n-1}}\alpha} \right).$

* Trường hợp 3: Nếu ${u_1}>1$ thì ta đặt ${u_1}=ch \alpha$. Chứng minh quy nạp ta được ${u_n} = ch\left( {{2^{n - 1}}\alpha } \right)$

Lưu ý: ${u_1} = ch\alpha \Leftrightarrow {u_1} = \frac{{{e^\alpha } + {e^{ - \,\alpha }}}}{2}$. Giải phương trình ta chỉ cần chọn một nghiệm $e^{\alpha}$.

Khi đó: ${u_n} = \frac{{{{\left( {{e^\alpha }} \right)}^{{2^{n - 1}}}} + {{\left( {{e^{ - \,\alpha }}} \right)}^{{2^{n - 1}}}}}}{2}$

* Trường hợp 4: Nếu ${u_1}<-1$ thì ta đặt ${u_1}=-ch \alpha$. Chứng minh quy nạp ta được ${u_n} = ch\left( {{2^{n - 1}}\alpha } \right),\,\,\,\,\forall n \ge 2$

Các dạng dãy số quy về dạng 1

1.1. Dạng: $\left\{ \begin{array}{l} \mathbf{\textbf{Cho}\,\,{u_1}}\\ \mathbf{{u_{n + 1}} = u_n^2 - 2\,,\,\,\forall n \in \mathbb{N^*}} \end{array} \right.$

Đặt ${u_n}=2 {x_n}$ ta được: $\left\{ \begin{array}{l}
{x_1} = \frac{1}{2}{u_1}\\ {x_{n + 1}} = 2x_n^2 - 1,\,\forall n \in \mathbb{N^*} \end{array} \right.$

1.2. Dạng: $\left\{ \begin{array}{l} \mathbf{\textbf{Cho}\,\,{u_1}}\\ \mathbf{{u_{n + 1}} = au_n^2 - \frac{2}{a}\,\,,\,\,\forall n \in \mathbb{N^*}} \end{array} \right.\,\,\,\,\,\,\,\mathbf{\left( {a \ne 0} \right)}$

Đặt ${u_n}= \dfrac{2}{a} {x_n}$ ta được: $\left\{ \begin{array}{l} {x_1} = \frac{a}{2}{u_1}\\ {x_{n + 1}} = 2x_n^2 - 1\,,\,\,\forall n \in \mathbb{N^*} \end{array} \right.$

1.3. Dạng: $\left\{ \begin{array}{l} \mathbf{ \text{Cho}\,\,{u_1}}\\ \mathbf{{u_{n + 1}} = au_n^2 + b{u_n} + c\,,\,\,\forall n \in \mathbb{N^*}} \end{array} \right.\,\,\,\,\,\,\,\,\,\,\,\,\,\mathbf{\left( {a \ne 0,\,c = \frac{{{b^2} - 2b - 8}}{{4a}}} \right)}$

Ta đặt: ${u_n}=p {x_n} +q$, thế vào giả thiết và tìm $p,q$ để ta được: ${x_{n + 1}} = 2x_n^2 - 1$

1.4. Dạng: $\left\{ \begin{array}{l} \mathbf{ \text{Cho}\,\,{u_1}}\\ \mathbf{{u_{n + 1}} = u_n^2 - 2{a^{{2^n}}}\,\,,\,\,\forall n \in \mathbb{N^*}} \end{array} \right. \,\,\,\,\,\, \mathbf{(a \ne 0)}$

Đặt ${u_n} = 2{a^{{2^{n - 1}}}}{x_n}$ ta được: $\left\{ \begin{array}{l} {x_1} = \frac{1}{{2a}}{u_1}\\
{x_{n + 1}} = 2x_n^2 - 1\,,\,\forall n \in \mathbb{N^*} \end{array} \right.$

1.5. Dạng: $\left\{ \begin{array}{l} \mathbf{ \text{Cho}\,\,{u_1}}\\ \mathbf{{u_{n + 1}} = 2{a^{{2^n}}}u_n^2 - {a^{\left( {n + 1} \right){2^n}}}\,\,,\,\,\forall n \in \mathbb{N^*}} \end{array} \right.\,\,\,\,\,\,\, \mathbf{\left( {a \ne 0} \right)}$

Đặt ${u_n} = {a^{n{2^{n - 1}}}}{x_n}$ ta được: $\left\{ \begin{array}{l} {x_1} = \frac{1}{a}{u_1}\\ {x_{n + 1}} = 2x_n^2 - 1 \end{array} \right.$

1.6. Dạng: $\left\{ \begin{array}{l} \mathbf{ \text{Cho}\,\,\,{u_1} \ge - 2}\\ \mathbf{{u_{n + 1}} = \sqrt {2 + {u_n}} \,;\,\,\forall n \in \mathbb{N^*}} \end{array} \right.$

Ta xét các trường hợp sau:

* Trường hợp 1: Nếu $ - 2 \le {u_1} \le 2$ thì ta đặt ${u_1}=2\cos \alpha;\,\,\, \alpha \in \left({0, \pi} \right)$ rồi áp dụng công thức $1 + \cos \alpha = 2{\cos ^2}\frac{\alpha }{2}$, chứng minh quy nạp ta được: ${u_n} = 2\cos \frac{\alpha }{{{2^{n - 1}}}}\,,\,\,\forall n \in \mathbb{N^*}$

* Trường hợp 2: Nếu ${u_1}>2$ thì ta đặt ${u_1} = 2ch\alpha $ rồi áp dụng công thức $1 + ch\left( {2\alpha } \right) = 2c{h^2}\alpha $, chứng minh quy nạp ta được: ${u_n} = 2ch\frac{\alpha }{{{2^{n - 1}}}}\,,\,\,\forall n \in \mathbb{N^*}$

1.7. Dạng: $\left\{ \begin{array}{l} \mathbf{ \text{Cho}\,\,{u_1}}\\ \mathbf{{u_{n + 1}} = 2{u_n}\sqrt {1 + u_n^2} \,,\,\,\forall n \in \mathbb{N^*}} \end{array} \right.$

Đặt $sh \alpha = {u_1}$, giải phương trình để tìm $e^\alpha$

Chứng minh quy nạp ta được: $${u_n} = sh\left( {{2^{n - 1}}\alpha } \right) = \dfrac{{{{\left( {{e^\alpha }} \right)}^{{2^{n - 1}}}} - {{\left( {{e^{ - \,\alpha }}} \right)}^{{2^{n - 1}}}}}}{2}$$
Dạng mở rộng: $\left\{ \begin{array}{l} \text{Cho}\,\,{u_1}\\ {u_{n + 1}} = b\left( {{u_n} + c} \right)\sqrt {u_n^2 + 2c{u_n} + {c^2} + {a^2}} - c\,,\,\,\forall n \in \mathbb{N^*} \end{array} \right.$ Với $a,\,b >0 $ và $ab=2$

1.8. Dạng: $\left\{ \begin{array}{l} \mathbf{ \text{Cho}\,\,{u_1} > 1}\\ \mathbf{{u_{n + 1}} = a.{u_n} + b\sqrt {u_n^2 - 1} \,,\,\,\forall n \in \mathbb{N^*}} \end{array} \right.\,\,\,\,\textbf{với}\,\,\, \mathbf{a>0}\,\,\,\textbf{ và}\,\,\,\,\mathbf{ a^2 - b^2 =1}$

Đặt $\left\{ \begin{array}{l} ch\beta = a\\ sh\beta = b \end{array} \right.$, giải hệ để tìm ${e^\beta }$ và đặt $\left\{ \begin{array}{l} ch\alpha = {u_1}\\ sh\alpha = \sqrt {u_1^2 - 1} \end{array} \right.$, giải hệ để tìm ${e^\alpha }$

Ta được: ${u_2} = ch\alpha .ch\beta + sh\alpha .sh\beta = ch\left( {\alpha + \beta } \right)$

Chứng minh quy nạp ta được: $${u_n} = ch\left[ {\alpha + \left( {n - 1} \right)\beta } \right]\, = \frac{{\left( {{e^\alpha }} \right){{\left( {{e^\beta }} \right)}^{n - 1}} + \left( {\frac{1}{{{e^\alpha }}}} \right){{\left( {\frac{1}{{{e^\beta }}}} \right)}^{n - 1}}}}{2},\,\,\forall n \in \mathbb{N^*}$$
Dạng mở rộng: $\left\{ \begin{array}{l}\text{Cho}\,\,{u_1}\\ {u_{n + 1}} = \frac{{{u_n}}}{{a + \sqrt {cu_n^2 + b} }}\,\,\,,\,\,\,\forall n \in \mathbb{N^*} \end{array} \right.$ với $\alpha > 0,\,\,a > 0$ và $a^2 - b =1$

Biến đổi: $\frac{1}{{{u_{n + 1}}}} = \frac{{a + \sqrt {cu_n^2 + b} }}{{{u_n}}} = a\left( {\frac{1}{{{u_n}}}} \right) + \sqrt {c + b\left( {\frac{1}{{u_n^2}}} \right)} $. Đặt ${x_n} = \frac{1}{{{u_n}}}$ ta đưa về dạng trên

1.9. Dạng: $\left\{ \begin{array}{l} \mathbf{ \text{Cho}\,\,{u_1}}\\ \mathbf{{u_{n + 1}} = \frac{{2{u_n}}}{{1 + u_n^2}}\,,\,\,\forall n \in \mathbb{N^*}} \end{array} \right.$

Dạng này ta chia trường hợp như sau:

* Nếu ${u_1}=1$ hay ${u_1}=-1$ thì ${u_n}=1$ hay ${u_n}=-1$

* Nếu $-1 < {u_1} < 1$ thì ta đặt ${u_1}=th \alpha$

Chứng minh quy nạp ta được: $${u_n} = th\left( {{2^{n - 1}}\alpha } \right) = \frac{{{{\left( {{e^\alpha }} \right)}^{{2^{n - 1}}}} - {{\left( {{e^{ - \,\alpha }}} \right)}^{{2^{n - 1}}}}}}{{{{\left( {{e^\alpha }} \right)}^{{2^{n - 1}}}} + {{\left( {{e^{ - \,\alpha }}} \right)}^{{2^{n - 1}}}}}} = \frac{{{{\left( {{e^{2\alpha }}} \right)}^{{2^{n - 1}}}} - 1}}{{\left( {{e^{2\alpha }}} \right) + 1}}\,,\,\,\forall n \in \mathbb{N^*}$$
* Nếu ${u_1} < -1$ hay ${u_1} > 1$ thì ta đặt ${u_1} = \coth \alpha = \frac{1}{{th\alpha }}$

Chứng minh quy nạp ta được: $${u_n} = th\left( {{2^{n - 1}}\alpha } \right) = \frac{{{{\left( {{e^{2\alpha }}} \right)}^{{2^{n - 1}}}} - 1}}{{\left( {{e^{2\alpha }}} \right) + 1}}\,,\,\,\forall n \ge 2$$
1.10. Dạng: $\left\{ \begin{array}{l} \mathbf{ \text{Cho}\,\,{u_1}}\\ \mathbf{{u_{n + 1}} = 4u_n^3 - 3{u_n}\,,\,\,\forall n \in \mathbb{N^*}} \end{array} \right.\,\,\,\,\,\,\,\,\mathbf{\left( a \right)}$ và $\left\{ \begin{array}{l}
\mathbf{\text{Cho}\,\,{u_1}}\\ \mathbf{{u_{n + 1}} = 4u_n^3 + 3{u_n}\,,\,\,\forall n \in \mathbb{N^*}} \end{array} \right.\,\,\,\,\,\,\,\mathbf{\left( b \right)}$

Với dạng 1.10a) Ta xét các trường hợp sau:

* Trường hợp 1: Nếu ${u_1}=1$ hay ${u_1}=-1$ thì ta được ${u_n}=1$ hay ${u_n}=-1,\,\,\,\,\,\,\forall \in \mathbb{N^*}$

* Trường hợp 2: Nếu $-1 < {u_1} <1$ thi ta đặt ${u_1}=\cos \alpha;\,\,\, \alpha \in \left({0; \pi} \right)$

Chứng minh quy nạp ta được: ${u_n} = \cos \left( {{3^{n - 1}}\alpha } \right)$

* Trường hợp 3: Nếu ${u_1}>1$ thì ta đặt ${u_1}=ch \alpha$.

Chứng minh quy nạp ta được: $${u_n} = ch\left( {{3^{n - 1}}\alpha } \right) = \frac{{{{\left( {{e^\alpha }} \right)}^{{3^{n - 1}}}} + {{\left( {{e^\alpha }} \right)}^{{3^{n - 1}}}}}}{2}$$
* Trường hợp 4: Nếu ${u_1}<-1$ thì ta đặt $-{u_1}=ch \alpha$

Chứng minh quy nạp ta được: $${u_n} = - \,ch\left( {{3^{n - 1}}\alpha } \right) = - \,\frac{{{{\left( {{e^\alpha }} \right)}^{{3^{n - 1}}}} + {{\left( {{e^\alpha }} \right)}^{{3^{n - 1}}}}}}{2}$$
Với dạng 1.10b) ta đặt ${u_1}=sh \alpha$. Chứng minh quy nạp ta được: $${u_n} = sh\left( {{3^{n - 1}}\alpha } \right) = \,\frac{{{{\left( {{e^\alpha }} \right)}^{{3^{n - 1}}}} - {{\left( {{e^\alpha }} \right)}^{{3^{n - 1}}}}}}{2}$$

Các dạng dãy số quy về dạng 1.10.

1.10.1. Dạng: $\left\{ \begin{array}{l} \text{Cho}\,\,{u_1}\\ {u_{n + 1}} = au_n^3\, \pm 3{u_n}\,\,;\,\,\forall n \in \mathbb {N^*} \end{array} \right.\,\,\,\,\,\,\left( {a > 0} \right)$

Nhận xét: Ta tìm cách quy về dạng trên như sau: Đặt ${u_n}=b{v_n}$.

Thế vào giả thiết ta được: $b{v_{n + 1}} = a{b^3}v_n^3 \pm 3b{v_n} \Rightarrow {v_{n + 1}} = a{b^2}v_n^3 \pm 3{v_n}$

Ta cần tìm $b$ sao cho $a{b^2} = 4 \Leftrightarrow b = \frac{2}{{\sqrt a }}$

1.10.2. Dạng: $\left\{ \begin{array}{l} {u_1} = \alpha \\ {u_{n + 1}} = au_n^3 + bu_n^2 + c{u_n} + d\,,\,\,\forall n \in \mathbb{N^*} \end{array} \right.$

với $a>0,\,c = \frac{{{b^2}}}{{3a}}\,,\,\,d = \frac{{b\left( {c - 3} \right)}}{{9a}}\,,\,\,\alpha > - \,\frac{b}{{3a}}$ hay $a>0,\,c = \frac{{{b^2} + 9a}}{{3a}}\,,\,\,d = \frac{{{b^3} + 18ab}}{{27{a^2}}}\,,\,\,\alpha > \frac{2}{{\sqrt a }} - \frac{b}{{3a}}$

Đặt ${u_n}=a{v_n}+b$, thế vào giả thiết rồi tìm hai số $a, b$ sao cho ${v_{n + 1}} = 4v_n^3 \pm 3{v_n}$

1.11. Dạng: $\left\{ \begin{array}{l}
\mathbf{ \text{Cho}\,\,{u_1}}\\
\mathbf{{u_{n + 1}} = 8u_n^4 - 8u_n^2 + 1\,,\,\,\forall n \in \mathbb{N^*}}
\end{array} \right.\,\,\,\,\,\,\mathbf{\left( a \right)}$ và $\left\{ \begin{array}{l}
\mathbf{ \text{Cho}\,\,{u_1}}\\
\mathbf{{u_{n + 1}} = \left( {8u_n^3 + 4{u_n}} \right)\sqrt {1 + u_n^2} \,,\,\,\forall n \in \mathbf{N^*}}
\end{array} \right.\,\,\,\,\,\,\mathbf{\left( b \right)}$

Với dạng 1.11a) Ta xét các trường hợp sau:

* Trường hợp 1:

Nếu ${u_1} = 1$ thì ta được ${u_n} = 1,\,\,\forall \in \mathbb{N^*}$

Nếu ${u_1} = – 1$ thì ta được ${u_n} = 1,\,\,\forall n \le 2$

* Trường hợp 2: Nếu $– 1 < {u_1} < 1$ thì ta đặt ${u_1} = \cos \alpha;\,\,\,\alpha \in \left({0; \pi} \right)$.

Chứng minh quy nạp ta được: ${u_n} = \cos \left( {{4^{n - 1}}\alpha } \right)$

* Trường hợp 3 : Nếu ${u_1} > 1$ thì ta đặt ${u_1} = ch \alpha$.

Chứng minh quy nạp ta được: $${u_n} = ch\left( {{4^{n - 1}}\alpha } \right) = \frac{{{{\left( {{e^\alpha }} \right)}^{{4^{n - 1}}}} + {{\left( {{e^\alpha }} \right)}^{{4^{n - 1}}}}}}{2}$$
* Trường hợp 4: Nếu ${u_1}< – 1$ thì ta đặt $– {u_1} = ch \alpha$.

Chứng minh quy nạp ta được: $${u_n} = \,ch\left( {{4^{n - 1}}\alpha } \right) = \frac{{{{\left( {{e^\alpha }} \right)}^{{4^{n - 1}}}} + {{\left( {{e^\alpha }} \right)}^{{4^{n - 1}}}}}}{2}\,,\,\,\forall n \ge 2$$
Với dạng 1.11b) ta đặt ${u_1} = sh \alpha$. Chứng minh quy nạp ta được: $${u_n} = sh\left( {{4^{n - 1}}\alpha } \right) = \,\frac{{{{\left( {{e^\alpha }} \right)}^{{4^{n - 1}}}} - {{\left( {{e^\alpha }} \right)}^{{4^{n - 1}}}}}}{2}$$

1.12. Dạng: $\left\{ \begin{array}{l}
\mathbf{ \text{Cho}\,\,{u_1}}\\
\mathbf{{u_{n + 1}} = \frac{{u_n^3 + 3{u_n}}}{{1 + 3u_n^2}}\,,\,\,\forall n \in \mathbb{N^*}}
\end{array} \right.$

Ta xét các trường hợp sau:

* Nếu ${u_1} = 1$ hay ${u_1} = – 1$ thì ${u_n} = 1$ hay ${u_n} = – 1$

* Nếu $– 1 < {u_1} < 1$ thì ta đặt $th \alpha ={u_1}$.

Chứng minh quy nạp ta được: $${u_n} = th\left( {{3^{n - 1}}\alpha } \right) = \frac{{{{\left( {{e^{2\alpha }}} \right)}^{{3^{n - 1}}}} - 1}}{{{{\left( {{e^{2\alpha }}} \right)}^{{3^{n - 1}}}} + 1}}$$
* Nếu ${u_1} < – 1$ hay ${u_1} > 1$ thì ta đặt $coth \alpha ={u_1}$ .

Chứng minh quy nạp ta được: $${u_n} = \coth \left( {{3^{n - 1}}\alpha } \right) = \frac{{{{\left( {{e^{2\alpha }}} \right)}^{{3^{n - 1}}}} + 1}}{{{{\left( {{e^{2\alpha }}} \right)}^{{3^{n - 1}}}} - 1}}$$

Bây giờ, chúng ta hãy xét một số ví dụ sau.

Ví dụ 1: Tìm số hạng tổng quát của dãy số $({u_n})$ định bởi: $\left\{ \begin{array}{l}
{u_1} = \frac{1}{3}\\
{u_{n + 1}} = 3u_n^2 - \frac{2}{3}\,,\,\,\forall n \in \mathbb{N^*}
\end{array} \right.$

Giải: Đặt ${u_n} = \frac{2}{3}{x_n}$ ta được dãy số $({x_n})$ định bởi: $\left\{ \begin{array}{l}
{x_1} = \frac{1}{2}\\
{x_{n + 1}} = 2x_n^2 - 1,\,\,\forall n \in \mathbb{N^*}
\end{array} \right.$

Ta có: ${x_1} = \cos \frac{\pi }{3}$. Giả sử ${x_n} = \cos \left( {{2^{n - 1}}\frac{\pi }{3}} \right)$. Khi đó ${x_{n + 1}} = 2{\cos ^2}\left( {{2^{n - 1}}\frac{\pi }{3}} \right) - 1 = \cos \left( {{2^n}\frac{\pi }{3}} \right)$

Vậy: ${x_n} = \cos \left( {{2^{n - 1}}\frac{\pi }{3}} \right)$ suy ra ${u_n} = \frac{2}{3}\cos \left( {{2^{n - 1}}\frac{\pi }{3}} \right)$

Ví dụ 2: Tìm số hạng tổng quát của dãy số $({u_n})$ định bởi: $\left\{ \begin{array}{l}
{u_1} = \frac{{15}}{2}\\
{u_{n + 1}} = u_n^2 - {2.3^{{2^n}}},\,\,\forall n \in \mathbb{N^*}
\end{array} \right.$

Giải: Đặt ${u_n} = {2.3^{{2^{n - 1}}}}{x_n}$ ta được dãy số $({x_n})$ định bởi: $\left\{ \begin{array}{l}
{x_1} = \frac{5}{4}\\
{x_{n + 1}} = 2x_n^2 - 1,\,\,\forall n \in \mathbb{N^*}
\end{array} \right.$

Đặt $ch\alpha = \frac{5}{2} \Leftrightarrow \frac{{{e^\alpha } + {e^{ - \,\alpha }}}}{2} = \frac{5}{4}$, giải phương trình ta chọn nghiệm ${e^\alpha } = 2$

Ta có: ${x_1} = ch\alpha $. Giả sử ${x_n} = ch\left( {{2^{n - 1}}\alpha } \right)$. Khi đó: ${x_{n + 1}} = 2c{h^2}\left( {{2^{n - 1}}\alpha } \right) - 1 = ch\left( {{2^n}\alpha } \right)$

Vậy: ${x_n} = ch\left( {{2^{n - 1}}\alpha } \right) = \frac{{{{\left( {{e^\alpha }} \right)}^{{2^{n - 1}}}} + {{\left( {{e^{ - \,\alpha }}} \right)}^{{2^{n - 1}}}}}}{2} = \frac{{{2^{{2^{n - 1}}}} + {{\left( {\frac{1}{2}} \right)}^{{2^{n - 1}}}}}}{2}$ suy ra ${u_n} = {6^{{2^{n - 1}}}} + {\left( {\frac{3}{2}} \right)^{{2^{n - 1}}}}$

Ví dụ 3: Tìm số hạng tổng quát của dãy số $({u_n})$ định bởi: $\left\{ \begin{array}{l}
{u_1} = 3\\
{u_{n + 1}} = \sqrt {2 + {u_n}} \,,\,\,\forall n \in \mathbb{N^*}
\end{array} \right.$

Giải: Đặt ${u_1} = 2ch\alpha \Leftrightarrow 3 = {e^\alpha } + {e^{ - \,\alpha }}$. Giải phương trình ta chọn nghiệm ${e^\alpha } = \frac{{3 + \sqrt 5 }}{2}$

Giả sử ${u_n} = 2ch\frac{\alpha }{{{2^{n - 1}}}}$. Khi đó: $${u_{n + 1}} = \sqrt {2 + 2ch\frac{\alpha }{{{2^{n - 1}}}}} = \sqrt {2\left( {1 + ch\frac{\alpha }{{{2^{n - 1}}}}} \right)} = \sqrt {4c{h^2}\frac{\alpha }{{{2^n}}}} = 2c{h^2}\frac{\alpha }{{{2^n}}}$$
Vậy: $${u_n} = 2ch\frac{\alpha }{{{2^{n - 1}}}} = {\left( {{e^\alpha }} \right)^{\frac{1}{{{2^{n - 1}}}}}} + {\left( {\frac{1}{{{e^\alpha }}}} \right)^{\frac{1}{{{2^{n - 1}}}}}} = {\left( {\frac{{3 + \sqrt 5 }}{2}} \right)^{\frac{1}{{{2^{n - 1}}}}}} + {\left( {\frac{{3 - \sqrt 5 }}{2}} \right)^{\frac{1}{{{2^{n - 1}}}}}}\,,\,\,\forall n \in \mathbb{N^*}$$

Còn tiếp...




#319239 Về một bài toán bất đẳng thức

Gửi bởi Ban Biên Tập trong 25-05-2012 - 00:02

VỀ MỘT BÀI TOÁN BẤT ĐẲNG THỨC

NGUYỄN VĂN HUYỆN

Sinh viên CN10B trường Đại học Giao thông vận tải tp.Hồ Chí Minh

[email protected]


1. Lời nói đầu:
Bất đẳng thức không thuần nhất là một phần quan trọng, hay và tương đối khó trong bất đẳng thức vì chúng ta không có một phương pháp thực sự “tốt” nào để giải quyết loạt các bài toán này. Những lời giải cho những bất đẳng thức dạng này thường mang những ý tưởng khá hay, độc đáo và thường là những phương pháp không mẫu mực. Ở bài viết này tác giả xin được giới thiệu đến bạn đọc một bài toán bất đẳng thức không thuần nhất tương đối đơn giản nhưng lại có nhiều ứng dụng trong việc giải quyết các bất đẳng thức thuần nhất và không thuần nhất khác, thậm chí là những bất đẳng thức trong đề thi học sinh giỏi quốc gia và quốc tế.

(Bài viết được trích ra từ bài viết cùng tên của tác giả đăng trong chuyên đề Toán học số 9 của trường Phổ Thông Năng Khiếu Đại học Quốc gia Tp.HCM và được tác giả bổ sung, điều chỉnh.)

2. Nội dung:
Bài toán gốc: Cho $a, b, c$ là các số thực dương thay đổi bất kì. Chứng minh rằng:
$$a^2+b^2+c^2+2abc+1\geq 2(ab+bc+ca) \ \ \ (1)$$
Lời giải 1:
Ta sẽ sử dụng phương pháp tam thức bậc hai để chứng minh bài toán. Bất đẳng thức được chuyển về dạng tam thức bậc hai như sau:
$$f(a)=a^2+2a(bc-b-c)+(b-c)^2+1\geq 0$$
* Nếu $bc\geq b+c$ thì ta có ngay điều phải chứng minh.
* Xét trường hợp ngược lại $bc\leq b+c$, và điều này tương đương với $(b-1)(c-1)\leq 1$. Khi đó ta tính được biệt thức $\Delta '$ của $f(a)$ là:
$$\Delta '=(bc-b-c)^2-(b-c)^2-1=bc(b-2)(c-2)-1$$
Ta xét hai trường hợp:
Trường hợp 1: Có đúng một trong hai số $b, c$ lớn hơn $2$, số còn lại không lớn hơn $2$. Trong trường hợp này ta có $(b-2)(c-2)\leq 0$ từ đó suy ra $\Delta '\leq 0$.
Trường hợp 2: Cả hai số $b, c$ đều không lớn hơn 2. Khi đó theo bất đẳng thức AM-GM, ta có :
$$\Delta '=bc(2-b)(2-c)-1\leq \left[\frac{b+c+(2-b)+(2-c)}{4}\right]^4-1= 0.$$

Tóm lại trong mọi trường hợp ta đều có $\Delta '\leq 0$. Tức $f(a)\geq 0$ và đây là điều phải chứng minh. Đẳng thức xảy ra khi và chỉ khi $a=b=c=1.$

Lời giải 2: Theo nguyên lý Dirichlet, ta thấy rằng trong ba số $a, b, c$ sẽ có hai số hoặc cùng $\geq 1$ hoặc cùng $\leq 1$. Giả sử hai số đó là $a, b$ khi đó:
$$(a-1)(b-1)\geq 0.$$
Từ đây, bằng cách sử dụng hằng đẳng thức:
$$a^2+b^2+c^2+2abc+1-2(ab+bc+ca)=(a-b)^2+(c-1)^2+2c(a-1)(b-1)\geq 0$$
Ta thu được ngay bất đẳng thức (1), phép chứng minh hoàn tất.

Lời giải 3: Ta sẽ sử dụng phương pháp dồn biến để chứng minh bài toán. Giả sử $c$ là số bé nhất và đặt:
$$f(a,b,c)=a^2+b^2+c^2+2abc+1-2(ab+bc+ca)$$
Ta có:
$$f(a,b,c)-f(\sqrt{ab},\sqrt{ab},c)=(\sqrt{a}-\sqrt{b})^2(a+b+2\sqrt{ab}-2c)\geq 0$$
Do đó $f(a,b,c)\geq f(\sqrt{ab},\sqrt{ab},c)$, vậy ta chỉ cần chứng minh $f(\sqrt{ab},\sqrt{ab},c)\geq 0$.
Thật vậy, nếu đặt $t=\sqrt{ab}$ thì ta có:
$$f(t,t,c)=2t^2+c^2+2t^2c-2(t^2+2tc)+1=(c-1)^2+2c(t-1)^2\geq 0$$
Bài toán được chứng minh xong.

Lời giải 4: Sử dụng lần lượt bất đẳng thức AM-GM, ta có:
$$2abc+1=abc+abc+1\geq 3\sqrt[3]{a^2b^2c^2}\geq \frac{9abc}{a+b+c}$$
Do đó, ta chỉ cần chứng minh:
$$a^2+b^2+c^2+\frac{9abc}{a+b+c}\geq 2(ab+bc+ca)$$
Thực hiện phép khi triển trực tiếp, ta có bất đẳng thức tương đương với:
$$a^3+b^3+c^3+3abc\geq ab(a+b)+bc(b+c)+ca(c+a)$$
Đúng vì đây chính là bất đẳng thức Schur dạng bậc ba nên ta có điều phải chứng minh.

Bất đẳng thức $(1)$ được Darij Grinberg đề xuất trên diễn đàn toán học Mathlinks.ro vào năm 2004. Mặc dù chỉ là một kết quả đơn giản nhưng bất đẳng thức này lại có nhiều ứng dụng vào việc chứng minh các bất đẳng thức ba biến. Sau đây, chúng ta sẽ đi vào xét các bài toán cụ thể để hiểu rõ vì sao chúng tôi lại nói như vậy.

Bài toán 1.1. (Moscow Mathematical Olympiad 2000). Cho $a, b, c$ là các số dương thỏa mãn $abc = 1$. Chứng minh bất đẳng thức sau
$$a^2 + b^2+c^2+a+b+c \geq 2(ab + bc + ca) \ \ \ (1.1.1)$$

Lời Giải.
Sử dụng bất đẳng thức AM-GM, ta có
$$a + b + c \geq 3\sqrt[3]{abc} = 3 = 2abc + 1$$.
Vì thế để chứng minh bài toán, ta chỉ cần chứng minh
$$a^2+b^2+c^2+2abc+1 \geq 2(ab+bc+ca)$$
Đây chính là bất đẳng thức (1) nên ta có ngay điều phải chứng minh.
Đẳng thức xảy ra khi và chỉ khi $a = b = c = 1.$

Bài toán 1.2. (Viet Nam Mathematical Olympiad 2006). Tìm hằng số $k$ lớn nhất để bất đẳng thức
$$\frac{1}{a^2}+\frac{1}{b^2}+\frac{1}{c^2}+3k \geq (k+1)(a+b+c)$$
luôn đúng với mọi $a, b, c$ dương thỏa mãn $abc = 1.$

Lời Giải.
Cho $a = b = t (t > 0, t = 1)$ và $c =\frac{1}{t^2}$, khi đó $a, b, c$ là các số dương và $abc = 1$. Do đó, theo yêu cầu của bài toán ta phải có
$$\frac{2}{t^2}+t^4+3k \geq (k+1)(2t+\frac{1}{t^2})$$
Bất đẳng thức này tương đương với mỗi bất đẳng thức trong dãy các bất đẳng thức sau đây
$$\frac{2}{t^2}+t^4-3 \geq (k+1)(2t+\frac{1}{t^2}-3),$$
$$\frac{t^6-3t^2+2}{t^2} \geq \frac{(k+1)(2t^3-3t^2+1)}{t^2}$$
$$\frac{t^2-1)^2(t^2+2)}{t^2} \geq \frac{(k+1)(t-1)^2(2t+1)}{t^2}$$
$$\frac{(t+1)^2(t^2+2)}{2t+1} \geq k+1, \forall t>0$$
Cho $t \to 0^+$, ta được $2 \geq k+1$, suy ra $k \leq 1$. Ta sẽ chứng minh rằng 1 chính là hằng số cần tìm, tức là
$$\frac{1}{a^2}+\frac{1}{b^2}+\frac{1}{c^2}+3 \geq 2(a+b+c).$$
Đặt $x = \frac{1}{a}, y= \frac{1}{b}, z=\frac{1}{c}$ thì ta có $xyz=1, 3=2xyz+1$ và
$$x^2+y^2+z^2+2xyz+1 \geq 2(xy+yz+zx)$$
hiển nhiên đúng theo (1). Vậy ta có kết luận $k_{max}=1.$

Bài toán 1.3. (Mircea Lascu, Romania Junior Team Selecsion Test 2005).
Cho các số dương $ a, b, c$ thỏa mãn điều kiện $(a + b)(b + c)(c + a) = 1.$ Chứng minh rằng
$$ab+bc+ca \leq \frac{3}{4}.$$
Lời Giải.
Đặt $x = a + b, y = b + c, z = c + a$ thì ta có $xyz = 1$ và
$$a=\frac{z + x -y }{2}, b=\frac{x+y-z}{2}, c=\frac{y+z-x}{2}.$$
Bất đẳng thức cần chứng minh được viết lại như sau
$$\frac{z + x -y }{2}.\frac{x+y-z }{2}+\frac{x+y-z }{2}.\frac{y+z-x }{2}+\frac{y+z-x }{2}.\frac{z + x -y }{2} \leq \frac{3}{4}$$
Sau khi thu gọn, ta được
$$x^2+y^2+z^2+3 \geq 2(xy+yz+zx),$$
hay là
$$x^2+y^2+z^2+2xyz+1 \geq 2(xy+yz+zx).$$
Đây chính là bất đẳng thức (1) nên ta có điều phải chứng minh.
Đẳng thức xảy ra khi và chỉ khi $a = b = c = \frac{1}{2}$

Bài toán 1.4. (Gabriel Dospinescu, Marian Tetiva, Mircea Lascu).
Chứng minh rằng với mọi số thực dương a, b, c, ta đều có
$$a^2+b^2+c^2 + 2abc + 3 \geq (a + 1)(b + 1)(c + 1).$$

Lời Giải.
Sau khi khai triển và rút gọn, ta có bất đẳng thức tương đương
$$a^2+b^2+c^2+ abc + 2 \geq ab + bc + ca + a + b + c,$$
hay là
$$2(a^2+b^2+c^2) + 2abc + 4 \geq 2(ab + bc + ca + a + b + c).$$
Theo bất đẳng thức (1), ta có
$$a^2+b^2+c^2 + 2abc + 1 \geq 2(ab + bc + ca).$$
Sử dụng đánh giá này, ta đưa được bài toán về chứng minh
$$a^2+b^2+c^2+2abc+1 \geq 2(ab+bc+ca).$$
Bất đẳng thức này tương đương với bất đẳng thức hiển nhiên đúng là
$$(a-1)^2+(b-1)^2+(c-1)^2 \geq 0$$
Bài toán được chứng minh xong. Đẳng thức xảy ra khi và chỉ khi $a = b = c = 1.$

Bài toán 1.5. (Asian Pacific Mathematical Olympiad 2004).
Chứng minh bất đẳng thức sau luôn đúng với mọi số thực dương a, b, c bất kỳ
$$(a^2 + 2)(b^2+ 2)(c^2+ 2) \geq 9(ab + bc + ca), \ \ (1.5.1)$$

Lời Giải. Bất đẳng thức cần chứng minh tương đương với
$$a^2b^2c^2+3(a^2+b^2+c^2)+2(a^2b^2+b^2c^2+c^2a^2+3)+2 \geq 9(ab+bc+ca).$$
Theo bất đẳng thức AM-GM, ta có
$$3(a^2+b^2+c^2)=3(\frac{a^2+b^2}{2}+\frac{b^2+c^2}{2}+\frac{c^2+a^2}{2}) \geq 3(ab+bc+ca)$$
Và $$2(a^2b^2+b^2c^2+c^2a^2+3)=2 \begin{bmatrix} (a^2b^2+1)+(b^2c^2+1)+(c^2a^2+1) \end{bmatrix} \geq 4(ab+bc+ca).$$
Từ đó, bài toán được đưa về chứng minh
$$a^2+b^2+c^2+a^2b^2c^2+2 \geq 2(ab+bc+ca)$$
Bất đẳng thức này được viết lại thành
$$ \begin{bmatrix} a^2+b^2+c^2+2abc+1-2(ab+bc+c+a) \end{bmatrix}+(abc-1)^2 \geq 0$$
hiển nhiên đugns theo (1). Đẳng thức xảy ra khi và chỉ khi $a=b=c=1.$

NHẬN XÉT: Bài toán còn đúng trong trường hợp $a,b,c$ là các số thực bát kì. Thật vậy, từ chứng mnih trên ta thấy rằng
$$(a^2+2)(b^2+2)(c^2+2)=( \begin{vmatrix}a \end{vmatrix}^2+2)( \begin{vmatrix}b \end{vmatrix}^2+2)( \begin{vmatrix}c \end{vmatrix}^2+2) \geq 9(\begin{vmatrix}a \end{vmatrix} \begin{vmatrix}b \end{vmatrix}+ \begin{vmatrix}b \end{vmatrix} \begin{vmatrix}c \end{vmatrix}+ \begin{vmatrix}c \end{vmatrix} \begin{vmatrix}a \end{vmatrix}) \geq 9(ab+bc+ca).$$

Bài toán 1.6. (Crux Mathematicorum).
Chứng minh bất đẳng thức sau luôn đúng với mọi số thực dương $a, b, c$ bất kỳ
$$(a^2+2)(b^2+2)(c^2+2) \geq 3(a+b+c)^2$$

Lời Giải. Tương tự như trên, ta cũng sử dụng phép khai triển trực tiếp và viết lại bất đẳng thức dưới dạng
$$a^2b^2c^2+a^2+b^2+c^2+2(a^2b^2+b^2c^2+c^2a^2+3)+2 \geq 6(ab+bc+ca).$$
Đến đây, ta cũng sử dụng bất đẳng thức AM-GM để thu được
$$2(a^2b^2+b^2c^2+c^2a^2+3) \geq 4(ab+bc+ca),$$
và từ đó đưa được bất đẳng thức về chứng minh
$$a^2+b^2+c^2+a^2b^2c^2+2 \geq 2(ab+bc+ca).$$
Đây chính là bất đẳng thức (1.5.1) đã được chứng minh ở phần trên. Bài toán được chứng minh xong. Đẳng thức xảy ra khi $a = b = c = 1.$

Nhận Xét. Vì $(a+b+c)^2 \geq 3(ab+bc+ca),$ nên từ bài toán này ta có thể dễ dàng suy ra
$$(a^2+2)(b^2+2)(c^2+2) \geq 9(ab+bc+ca)$$
Vậy bài toán này chính là một kết quả mạnh hơn của bất đẳng thức Asian Pacific Mathematical Olypiad 2004. Ngoài ra ta còn có thể làm chặt bất đẳng thức này hơn nữa, ta cùng xét bài toán sau đây.

Bài toán 1.7. (Nguyễn Đình Thi) Cho $a, b, c$ là các số thực dương. Chứng minh rằng
$$(a^2+2)(b^2+2)(c^2+2) \geq 3(a+b+c)^2+(abc-1)^2.$$
.
Lời Giải. Sau khi khai triển và rút gọn, ta được bất đẳng thức tương đương
$$2(a^2b^2+b^2c^2+c^2a^2+3)+a^2+b^2+c^2+2abc+1 \geq 6(ab+bc+ca).$$
Theo bất đẳng thức AM-GM thì
$$2(a^2b^2+b^2c^2+c^2a^2+3) \geq 4(ab+bc+ca).$$
Do đó ta chỉ cần chứng minh
$$a^2+b^2+c^2+2abc+1 \geq 2(ab+bc+ca).$$
Đây chính là bất đẳng thức (1).
Bài toán được chứng minh xong. Đẳng thức xảy ra khi và chỉ khi $a = b = c = 1.$

Bài toán 1.8. (Nguyễn Văn Huyện).
Với mọi số thực dương $a, b, c$ và $k \geq 2$ là một số thực bất kỳ, khi đó ta luôn có bất đẳng thức
$$(a^2+k)(b^2+k)(c^2+k) \geq \frac{(k+1)^2}{3}(a+b+c)^2+k^3-3k-2, \ \ (1.8.1)$$

Lời Giải. Ta cũng thực hiện phép khai triển và viết bất đẳng thức trên thành
$$a^2b^2c^2+k(a^2b^2+b^2c^2+c^2a^2)+k^2(a^2+b^2+c^2)+3k+2 \geq \frac{(k+1)^2}{3}(a+b+c)^2.$$
Sử dụng bất đẳng thức (1.5.1), ta được
$$a^2+b^2+c^2+(a^2+b^2+c^2+a^2b^2c^+2) \geq a^2+b^2+c^2+2(ab+bc+ca)=(a+b+c)^2,$$
từ đó chỉ cần chứng minh được
$$k(a^2b^2+b^2c^2+c^2a^2)+(k^2-2)(a^2+b^2+c^2)+3k \geq \frac{k^2+2k+2}{3}(a+b+c)^2.$$
Theo bất đẳng thức AM-GM, thì
$$(a^2b^2+1)+(b^2c^2+1)+(c^2a^2+1) \geq 2(ab+bc+ca)$$
Suy ra
$$k(a^2b^2+b^2c^2+c^2a^2)+(k^2-2)(a^2+b^2+c^2)+3k \geq 2k(ab+bc+ca)+(k^2-2)(a^2+b^2+c^2)$$
$$=(k^2-k-2)(a^2+b^2+c^2)+k(a+b+c)^2$$
$$ \geq \frac{k^2-k-2}{3}(a+b+c)^2+k(a+b+c)^2$$
$$=\frac{k^2+2k-2}{3}(a+b+c)^2.$$
Bài toán được chứng minh xong. Đẳng thức xảy ra khi và chỉ khi $a = b = c = 1.$

Nhận Xét. Trong (1.8.1) nếu cho $k = 2$, thì ta thu được (1.6.1).

Bài toán 1.9. (Iran Mathematical Olympiad 2002).
Cho các số dương $a, b, c$ thỏa mãn $ a^2+b^2+c^2 + abc = 4.$ Chứng minh rằng
$$a+b+c \leq 3.$$
Lời Giải. Từ giả thiết sử dụng bất đẳng thức $(1)$, ta có
$$9 = 2(a^2+b^2+c^2 + abc) + 1 = a^2+b^2+c^2 + (a^2+b^2+c^2 + 2abc + 1)$$
$$\geq a^2+b^2+c^2+ 2(ab + bc + ca)$$
$$= (a + b + c)2,$$
Vì $a, b, c$ là các số dương nên sau khi lấy căn hai vế, ta được
$$a + b + c \leq 3.$$
Chứng minh hoàn tất. Đẳng thức xảy ra khi và chỉ khi $a = b = c = 1$.

Bài toán 1.10. (Mathematical Reflections 4/2006).
Cho $a, b, c$ là ba số thực dương thỏa mãn điều kiện $a + b + c = 3$. Chứng minh bất đẳng thức
$$a^2+b^2+c^2 + abc \geq 4 \ \ \ (1.10.1)$$

Lời Giải. Nhân 2 vào hai vế của bất đẳng thức, ta được
$$2(a^2+b^2+c^2) + 2abc \geq 8.$$
Bây giờ bằng cách sử dụng bất đẳng thức (1), ta có
$2(a^2+b^2+c^2) + 2abc = $[$a^2+b^2+c^2+ (a^2+b^2+c^2 + 2abc + 1)$]$-1$
$\geq $[$a^2+b^2+c^2+2(ab+bc+ca)$]$-1$
$$=(a+b+c)^2-1$$
$$=8.$$
Bài toán được chứng minh xong. Đẳng thức xảy ra khi và chỉ khi $a = b = c = 1$.

Bài toán 1.11. (Nguyễn Văn Huyện).
Cho $a, b, c$ là ba số thực dương. Chứng minh bất đẳng thức
$$2(a^2+b^2+c^2)+2abc+10 \geq 6(a+b+c), \ \ \ (1.11.1)$$

Lời Giải. Nhân 2 và hai vế của bất đẳng thức, ta được
$$2(a^2+b^2+c^2) + 2abc +10 \geq 6(a + b + c).$$
Bất đẳng thức cần chứng minh tương đương với mỗi bất đẳng thức sau đây
$$[a^2 + b^2 + c^2 + 2(ab + bc + ca)] + a^2 + b^2 + c^2 + 2abc + 10 \geq 6(a + b + c) + 2(ab + bc + ca),$$
$$(a + b + c)2 + a^2 + b^2 + c^2 + 2abc +10 \geq 6(a + b + c) + 2(ab + bc + ca),$$
$$[(a + b + c)2 — 6(a + b + c) + 9] + a^2 + b^2 + c^2 + 2abc +1 \geq 2(ab + bc + ca),$$
$$(a + b + c - 3)2 + [a^2 + b^2 + c^2 + 2abc + 1 - 2(ab + bc + ca)] \geq 0.$$
Là kết quả hiển nhiên đúng theo bất đẳng thức (1) nên ta có điều phải chứng minh.
Bài toán được chứng minh xong. Đẳng thức xảy ra khi $a = b = c = 1.$

Nhận Xét. Từ bài này nếu cho $a + b + c = 3$, ta sẽ nhận được "Bài toán 1.10", còn nếu ta cho $a^2 + b^2 + c^2 + abc = 4$ thì ta sẽ nhận được "Bài toán 1.9".

Bài toán 1.12. (Trần Nam Dũng, Hello IMO 2007, Toán Học và Tuổi Trẻ).
Chứng minh rằng với mọi $a, b, c$ dương, ta luôn có
$$abc + 2(a^2 + b^2 + c^2) + 8 \geq 5(a + b + c), \ \ (1.12.1)$$

Lời Giải 1. Sử dụng bất đẳng thức AM-GM, ta có
$$a+b+c=\frac{1}{3}.3.(a+b+c) \leq \frac{1}{6}[9+(a+b+c)^2].$$
Do đó ta chỉ cần chứng minh
$$12(a^2 + b^2 + c^2) + 6abc + 48 \geq 5 [(a + b + c)2 + 9] .$$
Bất đẳng thức này tương đương với
$$7(a^2 + b^2 + c^2) + 6abc + 3 \geq 10(ab + bc + ca),$$
$$4(a^2 + b^2 + c^2 - ab - bc - ca) + 3 [a^2 + b^2 + c^2 + 2abc +1 - 2(ab + bc + ca)] \geq 0,$$
đúng vì ta có $a^2 + b^2 + c^2 \geq ab + bc + ca$ (theo AM-GM) và
$$a^2 + b^2 + c^2 + 2abc +1 \geq 2(ab + bc + ca)$$ (theo (1)).
Bài toán được chứng minh xong. Đẳng thức xảy ra khi và chỉ khi $a = b = c = 1$

Lời Giải 2. Ta có bất đẳng thức cần chứng minh tương đương với
$$(a - 1)^2+ (b -1)^2 + (c -1)^2 + [a^2 + b^2 + c^2 + abc + 5 - 3(a + b + c)] \geq 0,$$
là một kết quả hiển nhiên đúng theo bất đẳng thức (1.11.1), nên ta có điều phải chứng minh.

Còn nữa ....


#318610 Phương pháp đại số (phương pháp gien)

Gửi bởi Ban Biên Tập trong 22-05-2012 - 22:50

8. Phương pháp đại số (Phương pháp gien)
Nếu từ một nghiệm của phương trình đã cho ta có quy tắc để xây dựng ra một nghiệm mới thì quy tắc đó chính là gien. Phương pháp gien là phương pháp dựa vào gien để tìm tất cả các nghiệm của phương trình đã cho từ các nghiệm cơ sở. Để tìm các nghiệm cơ sở, ta áp dụng bước lùi, tức là quy tắc ngược của quy tắc tiến nói trên. Minh họa tốt nhất cho ý tưởng này là phương trình Pellphương trình Markov. Ta bắt đầu bằng phương trình Pell.

Phương trình Pell cổ điển là phương trình dạng, $x^2 – Dy^2 = 1$ trong đó $D$ là số nguyên dương không chính phương. Nếu $D = k^2$ thì từ phân tích $(x – ky)(x + ky) = 1$ ta suy ra phương trình đã cho có các nghiệm nguyên duy nhất là $(±1, 0)$. Trong trường hợp $D$ bất kỳ thì $(±1, 0)$ cũng là nghiệm của phương trình Pell. Ta gọi nghiệm này là nghiệm tầm thường.


Với số nguyên dương $D$ không chính phương cho trước, đặt
$$S = \{ (x, y) \in \mathbb{N}^* \times \mathbb{N}^* | x^2 – Dy^2 = 1 \}$$
là tập hợp tất cả các nghiệm nguyên dương của phương trình Pell
$$x^2 – Dy^2 = 1, \ \ (1)$$.


Ta có định lý quan trọng sau
Định lý 8.1. Nếu $D$ là số nguyên dương không chính phương thì $S \ne \varnothing$, tức là phương trình $(1)$ có nghiệm không tầm thường.

Chứng minh định lý này khá phức tạp, dựa vào lý thuyết liên phân số hoặc phương pháp hình học. Tuy nhiên, về mặt ứng dụng (trong các bài toán phổ thông), định lý này là không thực sự cần thiết vì với $D$ cho trước, ta có thể tìm ra một nghiệm nguyên dương của $(1)$ bằng phương pháp thử và sai. Ta bỏ qua định lý này và chuyển sang định lý mô tả tất cả các nghiệm của $(1)$ khi biết nghiệm cơ sở.


Với $(x, y), (x’, y’) \in S$ ta có nếu $x > x’$ thì $y > y’$. Do đó có thể định nghĩa $(x, y) > (x’ y’) \Leftrightarrow x > x’$. Với thứ tự này, $S$ là một tập sắp thứ tự tốt. Gọi $(a, b)$ là phần tử nhỏ nhất của $S$ theo thứ tự trên. Ta gọi $(a, b)$ là nghiệm cơ sở của $(1)$.

Định lý 8.2. Nếu $(a, b)$ là nghiệm cơ sở của $(1)$ và $(x, y)$ là một nghiệm nguyên dương tùy ý của $(1)$ thì tồn tại số nguyên dương $n$ sao cho $x + y \sqrt{D} = (a + b \sqrt{D})n$ và từ đó mọi nghiệm của $(1)$ đều có thể tìm được bởi công thức
\[x = \frac{{{{(a + b\sqrt D )}^n} + {{(a - b\sqrt D )}^n}}}{2},\;y = \frac{{{{(a + b\sqrt D )}^n} - {{(a - b\sqrt D )}^n}}}{{2\sqrt D }}\ \ \ (2)\].
Chứng minh: Nhận xét rằng nếu $(x, y)$ là nghiệm của $(1)$ thì
$$x’ = ax – Dby ; y’ = ay – bx $$
cũng là nghiệm của $(1)$ (có thể không nguyên dương)

Trước hết, do $\sqrt{D}$ vô tỷ nên nếu $x + y\sqrt{D} = (a + b\sqrt{D})n$ thì $x - y\sqrt{D} = (a - b\sqrt{D})n$ và từ đó $x^2 – Dy^2 = (a + b\sqrt{D})n(a - b\sqrt{D})n = (a^2 – Db^2)n = 1$, suy ra $(x, y)$ là nghiệm của $(1)$ và ta có công thức như trên.

Tiếp theo, giả sử không phải nghiệm nào của $(1)$ cũng có dạng $(2)$. Gọi $(x^*, y^*)$ là nghiệm nhỏ nhất không có dạng $(2)$ thì rõ ràng $x^* > a, y^* > b$.
Theo nhận xét:
$$x’ = ax^* - Dby^* ; y’ = ay^* - bx^* \ \ (3)$$
là nghiệm của $(1)$

Dễ dàng kiểm tra được rằng
$1) x^* > x’ > 0$
$2) y^* > y’ > 0$.

Từ đó, do tính nhỏ nhất của $(x^*, y^*)$, tồn tại $n$ nguyên dương sao cho $x’+ y’\sqrt{D} = (a+b\sqrt{D})n$. Giải hệ $(3)$ với ẩn là $(x^*, y^*)$, ta được (chú ý $a^2 – Db^2 = 1$)
$$x^* = ax’ + Dby’, y^* = ay’ + bx’$$
Từ đó
$$ x^*+ y^*\sqrt{D} = ax’ + Dby’ + (ay’+bx’)\sqrt{D} = (a+b\sqrt{D})(x’+y’\sqrt{D}) = (a+b\sqrt{D})^{n+1}$$
mâu thuẫn!.
Vậy điều giả sử là sai và $(2)$ là tất cả các nghiệm của $(1)$.

Tiếp theo, ta xét phương trình dạng Pell, tức là phương trình dạng$$x^2 – Dy^2 = k \ \ (4)$$
trong đó $D$ không chính phương và $k \in \{ 0, 1 \}$.
Ta có một số nhận xét sau
+ Không phải với cặp $D, k$ nào phương trình $(4)$ cũng có nghiệm. Ví dụ: phương trình $x^2 – 3y^2 = -1$.
+ Nếu phương trình $(4)$ có nghiệm nguyên dương thì nó có vô số nghiệm nguyên dương. Lý do: nếu $(x, y)$ là nghiệm của $(4)$ thì
$$x’ = ax + Dby ; y’ = ay + bx$$
cũng là nghiệm của $(4)$, trong đó $(a, b)$ là nghiệm cơ sở của phương trình.

Như thường lệ, ta đặt
$$S = \{ (x, y) \in \mathbb{N}^* \times \mathbb{N}^* | x^2 – Dy^2 = k \}$$
và gọi $(a, b)$ là nghiệm cơ sở của phương trình Pell tương ứng $x^2 – Dy^2 = 1$. Nghiệm $(x_0, y_0)$ thuộc S được gọi là nghiệm cơ sở của $(4)$ nếu không tồn tại $(x’, y’) \in S$ sao cho
$$x = ax’ + Dby’ ;y = ay’ + bx’$$
Gọi $S_0$ là tập hợp tất cả các nghiệm cơ sở. Ta có định lý quan trọng sau:


Định lý 8.3. Với mọi $D, k$ ta có $|S_0| < \infty$.
Chứng minh: Nếu $S_0 = \varnothing$ thì $|S_0| = 0 < \infty$. Tiếp theo giả sử $S_0 \neq \varnothing $. Gọi $(x, y)$ là một nghiệm cơ sở nào đó của $(4)$. Xét hệ
$$\left\{\begin{matrix}ax’ + Dby’ = x\\ ay’ + bx’ = y\end{matrix}\right.$$
có nghiệm $x’ = ax – Dby, y’ = ay – bx$. Dễ dàng chứng minh được $(x’)^2 – D(y’)^2 = 1$. Vì $(x, y) \in S_0$ nên theo định nghĩa $(x’, y’) \notin S$. Điều này xảy ra khi và chỉ khi
\[\left[ \begin{array}{l} x' \le 0\\ y' \le 0\end{array} \right.\; \Leftrightarrow \left[ \begin{array}{l} ax \le Dby\\ ay \le bx \end{array} \right. \Leftrightarrow \left[ \begin{array}{l} {x^2} \le - kD{b^2}\ \ \ (5)\\ {y^2} \le k{b^2}\ \ \ \ \ \ (6) \end{array} \right.\]

Nếu $(5)$ xảy ra thì ta có đánh giá $y^2 = \frac{x^2-k}{D} \leq - \frac{k(Db^2+1)}{D}$
Nếu $(6)$ xảy ra thì ta có $x^2 = Dy^2 + k \leq Dkb^2 + D$.
Trong cả hai trường hợp, ta có $|S_0| < \infty$.

Cuối cùng, chú ý rằng từ một nghiệm $(x, y)$ bất kỳ của $(4)$ không thuộc $S_0$, bằng cách đi ngược xuống bằng công thức $x’ = ax – Dby, y’ = ay – bx$ ta luôn có thể đi đến một nghiệm cơ sở của $(4)$. Như vậy, với định lý trên, phương trình dạng Pell đã được giải quyết hoàn toàn. Dưới đây chúng ta xem xét một ví dụ:

Ví dụ 8.1. Tìm tất cả các nghiệm nguyên dương của phương trình $x^2 – 5y^2 = –4, \ \ (1)$.
Lời giải: Bằng phép thử tuần tự, ta tìm được nghiệm cơ sở của phương trình $x^2 – 5y^2 = 1$ là $(9, 4)$. Theo phép chứng minh định lý 8.3, nghiệm cơ sở của $(1)$ thỏa mãn
$$x^2 \leq 4.5.42 ,y^2 \leq \frac{4.5.42 + 4}{5} \Rightarrow x < 17, y < 9$$
Dùng phép thử tuần tự, ta tìm được hai nghiệm cơ sở là $(1, 1)$ và $(11, 5)$. Từ hai nghiệm này, bằng công thức
$$x’ = 9x + 20y, y’ = 4x +9y$$
ta tìm được tất cả các nghiệm của $(1)$.

Với phép giải phương trình dạng Pell, trên thực tế ta đã có thể giải tất cả các phương trình Diophant bậc 2, tức là phương trình dạng
$$ax^2 + bxy + cy^2 + dx + ey + f = 0$$
Dựa vào lý thuyết đường cong bậc 2, ta có thể đưa phương trình trên về một trong các dạng chính sau
+ Dạng ellip: $ax^2 + by^2 = c (a, b, c > 0)$ – có hữu hạn nghiệm, giải bằng phương pháp thử và sai
+ Dạng parabol: $ax^2 + by + c$ - giải bằng đồng dư bậc 2
+ Dạng hypebol: $ax^2 – by^2 = c$ - phương trình dạng Pell
Ngoài ra còn có các dạng suy biết như hai đường thẳng cắt nhau, hai đường thẳng song song, ellip ảo … Dưới đây, ta xét một ví dụ áp dụng:

Ví dụ 8.2: Tìm tất cả các cặp số nguyên dương $(m, n)$ thỏa mãn phương trình
$$m(m+1) + n(n+1) = 3mn$$
Lời giải: Xét phương trình đã cho như phương trình bậc 2 theo $m$
$$m^2 – (3n-1)m + n(n+1) = 0$$
Phương trình này có nghiệm nguyên dương khi và chỉ khi $\Delta$ là số chính phương, tức là
$$(3n-1)2 – 4n(n+1) = y^2 \Leftrightarrow y^2 – 5(n-1)^2 = - 4 $$
ta thu được phương trình dạng Pell mà ta đã biết cách giải.

Đặc biệt với $k = -1$, phương trình $x^2 – Dy^2 = -1$ được gọi là phương trình Pell loại 2. Định lý dưới đây cho chúng ta điều kiện có nghiệm của phương trình này.

Định lý 8.4. Cho d là một số nguyên dương không chính phương. Xét phương trình
$$x^2 – dy^2 = -1 \; \; \ (1)$$
Gọi $(a, b)$ là nghiệm nhỏ nhất của phương trình Pell liên kết với phương trình $(1$), tức là phương trình
$$x^2 – dy^2 = 1 \; \; (1’)$$
Khi đó phương trình $(1)$ có nghiệm khi và chỉ khi hệ sau có nghiệm nguyên dương
$$\left\{ \begin{array}{l} a = x^2 + dy^2 \; \; \; (2) \\ b = 2xy \end{array} \right.$$.
Chứng minh. Giả sử $(x_0; y_0)$ là nghiệm nguyên dương của hệ phương trình $(2)$, vì $(a ; b)$ là nghiệm nguyên dương của phương trình Pell liên kết nên
$$a^2 – db^2 = 1 $$
Do vậy, từ $(2)$ ta có
$$(x_0 + dy_0)^2 – d(2x_0y_0)^2 = 1 \Leftrightarrow (x_0^2 – dy_0^2)^2 = 1 \Leftrightarrow x_0^2 – dy_0^2 = 1 \vee x_0^2 – dy_0^2 = -1$$.
Nếu $x_0^2 – dy_0^2= 1$ thì $(x_0, y_0)$ là nghiệm nguyên dương của phương trình Pell liên kết. Do $(a, b)$ là nghiệm nhỏ nhất của $(1’)$ nên $x_0 \ge a, y_0 \ge b$, mà $a = x_0^2 + dy_0^2 \Rightarrow x_0 \ge x_0^2 + dy_0^2$, vô lý!
Nếu $x_0^2 – dy_0^2 = - 1$ thì $(x_0; y_0)$ là nghiệm nguyên dương của phương trình $(1)$.

Đảo lại: Giả sử phương trình $(1)$ có nghiệm nguyên dương. Khi đó gọi $(x_0; y_0)$ là nghiệm nguyên dương nhỏ nhất của $(1)$. Ta sẽ chứng minh $(x_0; y_0)$ chính là nghiệm nguyên dương của hệ $(2)$.

Thật vậy, đặt $u = x_0^2 + dy_0^2$, $v = 2u_0v_0$ thì $u^2 – dv^2 = (x_0^2 + dy_0^2)^2 – d(2x_0y_0)^2 = (x_0^2-dy_0^2)^2 = 1$ (vì $x_0^2 – dy_0^2 = -1$). Suy ra $u, v$ là nghiệm của phương trình $(1’)$. Do $(a; b)$ là nghiệm nguyên dương nhỏ nhất của $(1’)$ nên $u \ge a, v \ge b$.

Ta chứng minh $u = a, v = b$.
Giả sử trái lại $u > a, v > b$ (vì $u > a \Leftrightarrow v > b$). Ta có $0 < a - b\sqrt d < (a - b\sqrt d )(a + b\sqrt d ) = {a^2} - d{b^2} = 1$. Do đó
\[(a - b\sqrt d )({x_0} + {y_0}\sqrt d ) < {x_0} + {y_0}\sqrt d \Leftrightarrow a{x_0} - bd{y_0} + (a{y_0} - b{x_0})\sqrt d < {x_0} + {y_0}\sqrt d \]
Từ $u > a, v > b$ suy ra $a + b\sqrt d < u + v\sqrt d = {({x_0} + {y_0}\sqrt d )^2}$. Do vậy
$$- \left( ax_0-bdy_0 \right) + \left( ay_0-bx_0 \right) \sqrt{d} = \left( a+b \sqrt{d} \right) \left( -x_0+y_0 \sqrt{d} \right) < \left( x_0+y_0 \sqrt{d} \right)^2 \left( -x_0+y_0 \sqrt{d} \right)$$
Hay $$- \left( ax_0-bdy_0 \right)+ \left(ay_0 -bx_0 \right) \sqrt{d} < \left( dy_0^2-x_0^2 \right) \left( x_0+y_0 \sqrt{d} \right) = x_0+y_0 \sqrt{d}.$$
Đặt $s = ax_0 – bdy_0$; $t = ay_0 – bx_0$, ta có
$$\begin{array}{l} s+t \sqrt{d} < x_0+y_0 \sqrt{d} \; \; \; \; \; (3) \\ -s+t \sqrt{d} < x_0+y_) \sqrt{d} \; \; \; (4) \end{array}$$
Ta thấy
$$\begin{aligned} s^2 – dt^2 & = (ax_0^2 – bdy_0)^2 – d(ay_0-bx_0)^2 \\ & = a^2(x_0^2-dy_0^2) + b^2d(dy_0^2-x_0^2) \\ & = -a^2 + db^2 \\ & = -1. \end{aligned}$$
Vậy $s^2 – dt^2 = -1$, suy ra $s \ne 0$ (vì nếu $s = 0$ thì $dt^2 = 1$ suy ra $d = 1$, vô lý).

Ta chứng minh $t > 0$.
Ta có $$t > 0 \Leftrightarrow ay_0 > bx_0 \Leftrightarrow a^2y_0^2 > b^2x_0^2 \Leftrightarrow (1+db^2)y_0^2 > b^2(dy_0^2-1) \Leftrightarrow y_0^2 > -b^2.$$ Bất đẳng thức cuối cùng đúng do đó $t > 0$.

Do $s \ne 0$ nên $s > 0$ hoặc $–s > 0$.
Nếu $s > 0$ thì $(s; t)$ là nghiệm nguyên dương của phương trình $(1)$. Mà $(x_0; y_0)$ là nghiệm nguyên dương bé nhất của phương trình này nên $s \ge x_0, t \ge y_0$ suy ra $$s+ t \sqrt{d} \ge x_0+y_0 \sqrt{d}$$ trái với $(3)$.

Trường hợp $–s > 0$ thì $(-s; t)$ là nghiệm nguyên dương của phương trình $(1)$ và lý luận tương tự, ta đi đến một bất đẳng thức mâu thuẫn với $(4)$.
Vậu $(u; v) = (a; b)$.


Nhận xét. Thực tế là ta đã chứng minh kết quả sau : Nếu $(a ; b)$ là nghiệm nhỏ nhất của $(1’)$ và $(m, n)$ là nghiệm nhỏ nhất của $(1)$ thì $a = m^2 + dn^2, b = 2mn.$

Một điều thú vị khác nữa là phương trình dạng $Ax^2 – By^2 = 1$ cũng có mối liên quan chặt chẽ đến phương trình $x^2 – Aby^2 = 1$. Ta có kết quả sau:

Định lý 8.5. Cho phương trình $$Ax^2 – By^2 = 1 \; \; \; \; (1)$$
với $A$ và $AB$ không chính phương. Gọi $(a, b$) là nghiệm nhỏ nhất của phương trình Pell kết hợp
$$x^2 – ABy^2 = 1 \; \; \; \; (2)$$
Giả sử phương trình $(1)$ có nghiệm và $(x_0; y_0)$ là nghiệm nhỏ nhất của nó thì $(x_0; y_0)$ là nghiệm duy nhất của hệ phương trình
$$\left\{ \begin{array}{l} a = Ax^2 + By^2 \\ b = 2xy \end{array} \right.$$


Chứng minh. Giả sử $(x_0; y_0)$ là nghiệm nhỏ nhất của $(1)$. Đặt $u = Ax_0^2 + By_0^2, v = 2x_0y_0$ thì ta có
$$u^2 – ABv^2 = (Ax_0^2 + By_0^2)^2 – AB(2x_0y_0)^2 = (Ax_0^2 – By_0^2)^2 = 1.$$
Chứng tỏ $(u; v)$ là nghiệm của phương trình $(2)$. Mà $(a; b)$ là nghiệm nhỏ nhất của phương trình này nên $u \ge a, v \ge b.$
Ta chứng minh $u = a, v = b.$
Thật vậy, giả sử trái lại $u > a ; v > b.$

\[a - b\sqrt {AB} < (a - b\sqrt {AB} )(a + b\sqrt {AB} ) = {a^2} - AB{b^2} = 1\]
\[ \Rightarrow (a - b\sqrt {AB} )(\sqrt A {x_0} + \sqrt B {y_0}) < (\sqrt A {x_0} + \sqrt B {y_0})\]
\[ \Rightarrow (a{x_0} - Bb{y_0})\sqrt A + (a{y_0} - Ab{x_0})\sqrt B < (\sqrt A {x_0} + \sqrt B {y_0})\]
Lại có
\[\begin{array}{l}(a + b\sqrt {AB} ) < (u + v\sqrt {AB} ) = {\left( {\sqrt A {x_0} + \sqrt B {y_0}} \right)^2}\\ \Rightarrow (a{x_0} - Bb{y_0})\sqrt A - (a{y_0} - Ab{x_0})\sqrt B = (a + b\sqrt {AB} )(\sqrt A {x_0} - \sqrt B {y_0}) \\ < {(\sqrt A {x_0} + \sqrt B {y_0})^2}(\sqrt A {x_0} - \sqrt B {y_0}) = (\sqrt A {x_0} + \sqrt B {y_0}) \end{array}\]
Đặt $s = ax_0 – Bby_0, t = ay_0 – Abx_0$ thì các bất đẳng thức trên có thể viết lại thành
\[\begin{array}{l}s\sqrt A + t\sqrt B < {x_0}\sqrt A + {y_0}\sqrt B \ \ \ (3)\\ s\sqrt A - t\sqrt B < {x_0}\sqrt A + {y_0}\sqrt B \ \ \ (4)\end{array}\]
Tiếp theo, ta có $$(As^2 – Bt^2) = A(ax_0 – Bby_0)^2 – B(ay_0 – Abx_0)^2= (a^2 – ABb^2)(Ax_0^2 – By_0^2) = 1.1 = 1.$$
Ta thấy $s > 0$ vì
$$s > 0 \Leftrightarrow ax_0 > Bby_0 \Leftrightarrow a^2x_0^2 > B^2b^2y_0^2 \Leftrightarrow a^2x_0^2 > Bb^2(Ax_0^2-1) \Leftrightarrow (a^2-ABb^2)x_0^2 > - Bb^2 \Leftrightarrow x_0^2 > - Bb^2.$$
Bất đẳng thức cuối cùng đúng, do đó $s > 0$.
Ta thấy $t \ne 0$ vì
$$t = 0 \Leftrightarrow ay_0 = Abx_0 \Leftrightarrow a^2y_0^2 = A^2b^2x_0^2 \Leftrightarrow (ABb^2+1)y_0^2 = Ab^2(By_0^2+1) \Leftrightarrow y_0^2 = Ab^2$$
Điều này không thể xảy ra do $A$ không chính phương.

Nếu $t > 0$ thì $(s; t)$ là nghiệm nguyên dương của (1), mà (x0 ; y0) là nghiệm nhỏ nhất của $(1)$ $s \ge x_0 ; t \ge y_0$, do vậy $$s \sqrt{A}+ t \sqrt{B} \ge \sqrt{A}x_0+ \sqrt{B}y_0$$
Điều này mâu thuẫn với $(3)$.

Tương tự, với $t < 0$ thì $(s ; -t)$ là nghiệm nguyên dương của $(1)$ và ta cũng dẫn đến một bất đẳng thức mâu thuẫn với $(4)$.
Vậy $u = a, v = b$ hay $(x_0 ;y_0)$ là nghiệm của hệ trên.

Ví dụ 8.3. (Vietnam TST 2009) Cho $a, b$ là các số nguyên dương không chính phương sao cho $a.b$ cũng không chính phương. Chứng minh rằng ít nhất một trong hai phương trình
$$ax^2 – by^2 = 1$$ và $$ax^2 – by^2 = –1. $$
không có nghiệm nguyên dương.


Lời giải. Giả sử cả hai phương trình
$$ax^2 – by^2 = 1 \; \; \; \; \; (1)$$

$$bx^2 – ay^2 = 1 \; \; \; \; \; (2)$$
có nghiệm .
Gọi $(m ; n)$ là nghiệm nhỏ nhất của phương trình $x^2 – aby^2 = 1$, $(x_1;y_1)$ là nghiệm nhỏ nhất của phương trình $(1)$ và $(x_2,y_2)$ là nghiệm nhỏ nhất của phương trình $(2)$. Áp dụng định lý 8.5, ta có
$$\left\{ \begin{array}{l} m = ax_1^2 + by_1^2 \\ n = 2x_1y_1 \end{array} \right.$$
và $$\left \{ \begin{array}{l} m = bx_2^2+ ay_2^2 \\ n = 2x_2y_2 \end{array} \right.$$
Do $ax_1^2 = by_1^2 + 1$ và $ay_2^2 = bx_2^2 – 1$ nên từ đây ta suy ra
$$ax_1^2 + by_1^2= bx_2^2 + ay_2^2 \Leftrightarrow 2by_1^2 + 1 = 2bx_2^2 – 1\Leftrightarrow b(x_2^2-y_1^2) = 1$$
Điều này không thể xảy ra do $b > 1$


Phương trình Markov cổ điển là phương trình dạng
$$x_1^2+x_2^2+ \cdots + x_n^2 = kx_1 \cdots x_n \; \; \; \; (1)$$
Ở đây $n$ và $k$ là các tham số nguyên dương. Trường hợp riêng khi $n = k = 3$ – phương trình
$$x^2+y^2+z^2 = 3xyz \; \; \; \; (2)$$
được nghiên cứu chi tiết trong bài báo của A.A Markov về dạng toàn phương dương đăng ở Báo cáo VHL KH Liên Xô năm 1951; “dạng Markov”, liên quan chặt chẽ đến phương trình dạng $(2)$ được sử dụng trong lý thuyết xấp xỉ các số vô tỷ bằng các số hữu tỷ.

Đầu tiên, ta chú ý đến một tính chất thú vị của phương trình Markov. Nếu phương trình $(1)$ có một nghiệm thì nó sẽ có rất nhiều nghiệm và có thể tạo ra các nghiệm đó bằng cách sau đây. Ta sẽ coi một biến, chẳng hạn $x_n$, là “ẩn số”, còn tất cả các biến khác như các tham số. Khi đó, vì phương trình
$$x^2 – kx_1 \cdots x_{n-1}x + x_1^2+…+ x_{n-1}^2 = 0$$
là phương trình bậc 2 theo x và có nghiệm $x = x_n$, nên nó có nghiệm nguyên thứ hai $x’_n = u$; theo định lý Viet ta có
$$u = kx_1 \cdots x_{n-1} – x_n = (x_1^2+ \cdots + x_{n-1}^2)/x_n \; \; \; \; (3)$$
Chú ý rằng $u < x_n$ khi và chỉ khi
$$x_1^2+ \cdots + x_{n-1}^2 kx_1 \cdots x_{n-1} \; \; \; \; (4)$$
Quá trình này có thể thực hiện với mọi biến số $x_j$ trong vai trò của $x_n$. Nhưng chỉ đối với một biến - biến lớn nhất là có thể xảy ra $(4)$ và ta thu được nghiệm mới $(x_1,x_2, …, x’_n)$ “nhỏ hơn” nghiệm cũ (thứ tự theo tổng các biến); như vậy, theo đa số là các nghiệm tăng lên và ta có cây nghiệm.

Tiếp theo, trừ những trường hợp đặc biệt, ta sẽ giả sử rằng $x_1 \le x_2 \le \cdots \le x_n$. Ta sẽ nói nghiệm $(x_1,x_1, \cdots , x_n)$ là nghiệm gốc (nghiệm cơ sở), nếu
$$x_1^2+ \cdots + x_{n-1}^2\ge x_n^2 \Leftrightarrow 2x_n \le kx_1 \cdots x_{n-1} \; \; \; (5)$$
(từ nghiệm này, tất cả các nhánh cây đi đến các nghiệm bên cạnh, đều tăng)


Bổ đề 8.4. Nếu phương trình $(1)$ có nghiệm nguyên dương thì nó có nghiệm gốc.

Bổ đề 8.5. Nếu $n > 2$, $(x_1,x_2, \cdots, x_n)$) là nghiệm gốc, ngoài ra, $x_1 \le x_2 \le \cdots \le x_n$. Khi đó
$$x_1 \cdots x_{n-2} \le \dfrac{2(n-1)}{k}.$$


Chứng minh:
$$\begin{aligned} kx_1 \cdots x_{n-2}x_{n-1}^2 & \le kx_1 \cdots x_{n-2}x_{n-1}x_n \\ & = x_1^2+ \cdots + x_{n-1}^2+ x_n^2 \\ & \le 2(x_1^2+ \cdots + x_{n-1}^2) \\ & \le 2(n-1)x_{n-1}^2 \end{aligned}$$

Bổ đề 8.6. Nếu $x_1 \le x_2 \le \cdots \le x_n$ là các số nguyên dương bất kỳ thoả mãn điều kiện $1 < x_n^2 \le x_1^2+ \cdots + x_{n-1}^2$, thì tỷ số $R = \dfrac{x_1^2+ \cdots + x_n^2}{x_1x_2 \cdots x_n}$ không vượt quá $\dfrac{n+3}{2}.$.

Định lý 8.7. Nếu phương trình $(1)$ có nghiệm và $n \ne k$, thì $n \ge 2k – 3$ khi $n \ge 5$ và $n > 4k – 6$ khi $n = 3, n = 4.$

Từ các định lý và bổ đề trên, với $n$ cho trước, việc tìm tất cả các giá trị $k$ sao cho $(1)$ có nghiệm thực hiện được dễ dàng. Hơn nữa, phương pháp gien được sử dụng ở trên có thể áp dụng cho các phương trình dạng tương tự, ví dụ phương trình $(x+y+z)^2 = kxyz$.

Cuối cùng là một ví dụ khác về ứng dụng của gien

Ví dụ 8.3: (Iran 2001) Giả sử $x, y, z$ là các số nguyên dương thỏa mãn điều kiện xy = z2 + 1. Chứng minh rằng tồn tại các số $a, b, c$ và d sao cho $x = a^2+b^2, y = c^2+d^2$ và $z = ac+ bd.$

Bài tập
8.1 (Ailen 1995) Tìm tất cả các số nguyên a sao cho phương trình $$x^2 + axy + y^2 = 1$$ có vô số nghiệm nguyên phân biệt $x, y.$
8.2. (Đài Loan 1998) Tồn tại hay không nghiệm của phương trình
$$x^2+y^2+z^2+u^2+v^2 = xyzuv – 65$$
trong tập hợp các số nguyên lớn hơn $1998$?
8.3. (Việt Nam 1999) Cho hai dãy số $(x_n)$, $(n=0, 1, 2, …)$ và $(y_n)$, $(n=0, 1, 2, …)$ được xác định
8.4. (Việt Nam 2002) Tìm tất cả các số nguyên dương n sao cho phương trình
$$x+y+z+t=n \sqrt{xyzt}$$có nghiệm nguyên dương.
8.5. (Ba Lan 2002) Tìm tất cả các cặp số nguyên dương $x, y$ thỏa mãn phương trình $$(x+y)^2 – 2(xy)^2 = 1.$$
8.6. (Mỹ 2002) Tìm tất cả các cặp sắp thứ tự các số nguyên dương $(m, n)$ sao cho $mn – 1$ chia hết $m^2+n^2.$
8.7. (Việt Nam 2002, vòng 2) Chứng minh rằng tồn tại số nguyên $m \ge 2002$ và m số nguyên dương phân biệt $a_1,a_2, \cdots a_m$ sao cho
$$\prod_{i=1}^{m}a_i^2-4\sum_{i=1}^{m}a_i^2$$ là số chính phương.
8.8. (Việt Nam 2002, vòng 2) Tìm tất cả các đa thức $p(x)$ với hệ số nguyên sao cho đa thức
$$q(x) = (x^2+6x+10)(p(x))^2 – 1 $$
là bình phương của một đa thức với hệ số nguyên.
8.9. (THTT 6/181) Với giá trị nguyên dương nào của p thì phương trình $x^2+y^2 + 1 = pxy$ có nghiệm nguyên dương?
8.10. (THTT 4/202) Cho ba số nguyên $a, b, c; \ a > 0$, $ac – b^2 = p_1p_2 \cdots p_m$ trong đó $p_1p_2 \cdots p_m$ là các số nguyên tố khác nhau. Gọi $M(n)$ là số các cặp số nguyên $(x, y)$ thỏa mãn
$$ax^2 + 2bxy + cy^2 = n.$$
8.11. (Đề đề nghị IMO 95) Tìm số nguyên dương $n$ nhỏ nhất sao cho $19n+1$ và $95n+1$ đều là các số chính phương.
8.12. Tam giác với cạnh $3, 4, 5$ và tam giác với cạnh $13, 14, 15$ có các cạnh là các số nguyên liên tiếp và có diện tích nguyên. Hãy tìm tất cả các tam giác có tính chất như vậy.
8.13. Chứng minh rằng nếu cả $3n+1$ và $4n+1$ đều là các số chính phương thì $n$ chia hết cho $56$.
8.14*. Trong các hàng của tam giác Pascal, hãy tìm hàng có chứa ba số hạng liên tiếp lập thành một cấp số cộng.
8.15. (Mỹ 1986) Tìm số nguyên dương $n > 1$ nhỏ nhất sao cho trung bình bình phương của $n$ số nguyên dương đầu tiên là một số nguyên.
8.16. (IMO 88) Nếu $a, b, q = \dfrac{a^2+b^2}{ab+1}$ là các số nguyên dương thì $q$ là số chính phương.
8.17. (MOCP 03) Tìm tất cả giá trị $n$ sao cho phương trình $(x+y+z)^2 = nxyz$ có nghiệm nguyên dương.
8.18. (PTNK 03). Tìm tất cả các số nguyên dương $k$ sao cho phương trình $$x^2 - (k^2-4)y^2 = - 24$$ có nghiệm nguyên dương.
8.19. Chứng minh rằng phương trình $(k^2-4)x^2 – y^2 = 1$ không có nghiệm nguyên với mọi $k>3$.
8.20. (Mathlinks) Cho A là tập hợp hữu hạn các số nguyên dương. Chứng minh rằng tồn tại tập hợp hữu hạn các số nguyên dương B sao cho $A \in B$ và $\prod _{x \in \boldsymbol{B}}x=\sum_{x \in \boldsymbol{B}}x^2.$
8.21. (AMM 1995) Cho x, y là các số nguyên dương sao cho $xy + x$ và $xy + y$ là các số chính phương. Chứng minh rằng có đúng một trong hai số $x, y$ là số chính phương.


Tài liệu tham khảo
1. Jean-Marie Monier. Đại số 1 – giáo trình toán tập 5, NXBGD-Dunod 1999.
2. Hà Huy Khóai - Phạm Huy Điển. Số học thuật toán, NXB ĐHQG HN 2003.
3. Lê Hải Châu. Các bài thi học sinh giỏi Toán PTTH toàn quốc, NXBGD 1994.
4. Nguyễn Sinh Nguyên, Nguyễn Văn Nho, Lê Hoành Phò. Tuyển tập các bài dự tuyển Olympic Toán học Quốc tế 1991-2001, NXBGD 2003.
5. Nguyễn Văn Nho. Olympic Toán học châu Á – Thái Bình Dương 1989-2002, NXBGD 2003.
6. Tập thể tácgiả. Tuyển tập 5 năm Tạp chí Toán học và Tuổi trẻ, NXBGD 2003.
7. Arthur Engel. Problem Solving Strategies, Springer 1998
8. Goeoge Polya. Gabor Szego, Problems and Theorems in Analysis II, Springer 1976
9. Harvey Cohn. Advanced Number Theory, Dover Publications 1980
10. Titu Andreescu, Juming Feng. Mathematical Olympiads 1999-2000:Olympiads Problems from Around the World, MMA 2000.
11. Titu Andreescu, Juming Feng, Hojoo Lee. Mathematical Olympiads 2001-2002: Olympiads Problems from Around the World, MMA 2002.
12. Titu Andreescu & Razvan Gelca. Mathematical Olympiads Challenge. Bikhauser 2000.
13. Walter Mientka & others. Mathematical Olympiads 1996-1997: Olympiads Problems from Around the World, MMA 1997.
14. Walter Mientka & others. Mathematical Olympiads 1997-1998: Olympiads Problems from Around the World, MMA 1998.
15. Б.О. Бугаенко. Уравнения Пелля, Москва 2001.
16. Д.Ф. Базылев. Диофантовы уравнения, Минск 1999.
17. А.О. Гельфонд. Решение уравнений в целых числах, Наука 1978.
18. В. Серпинский. О решений уравнений в целых числах, ФМЛ 1961.
19. В. Серпинский. 250 задач по элементарной теории чисел, ФМЛ 1968. Có bản dịch đề bài tiếng Việt do Sở GD Tp HCM in năm 1982.
20. Дэвенпорт. Высшая арифметика, Наука 1956.
21. Các tạp chí Kvant, AMM, Toán học & Tuổi trẻ, Toán học trong nhà trường.
22. Tư liệu Internet, đặc biệt là các website: www.mccme.ruwww.mathlinks.ro.


:)


#318210 Một số điều nên và không nên trong giảng dạy toán

Gửi bởi Ban Biên Tập trong 21-05-2012 - 08:43

Trong loạt bài này, tôi sẽ viết dần một số quan điểm của tôi về những điều nên và không nên trong giảng dạy. Những quan điểm này được rút ra từ kinh nghiệm bản thân, việc nghiên cứu các liệu về giáo dục, sự trao đổi với đồng nghiệp và sinh viên, và những suy nghĩ để làm sao dạy học tốt hơn. Tất nhiên có những quan điểm của tôi có thể còn phiến diện. Xin mời mọi người trao đổi, viết lên những quan điểm và kinh nghiệm của mình.
Hình đã gửi
Tôi sẽ chủ yếu nói về việc dạy toán, tuy rằng nhiều điểm áp dụng được cho hầu hết các môn học khác. Tôi sẽ dùng từ “giảng viên” để chỉ cả giảng viên đại học lẫn giáo viên phổ thông, từ “học sinh” (student) để chỉ học sinh sinh viên hay học viên ở mọi cấp học, từ phổ thông cho đến sau đại học. Tôi viết không theo thứ tự đặc biệt nào.
Hình đã gửi
1) Nên: Thỉnh thoảng thay đổi môn dạy nếu có thể. Nếu dạy một môn nhiều lần, thì cải tiến thường xuyên phương pháp và nội dung dạy môn đó.
Không nên: Dạy mãi năm này qua năm khác một môn, với giáo trình nhiều năm không thay đổi.
Các chức vụ quản lý lãnh đạo thường có nhiệm kỳ, và thường có nguyên tắc là không ai làm quá 2 nhiệm kỳ ở cùng 1 vị trí. Lý do là để tạo sự thay đổi cải tiến thường xuyên, tránh sự trì trệ. Ngay trong việc dạy học cũng vậy: một người mà dạy quá nhiều năm cùng một thứ, thì dễ dẫn đến nhàm chán trì trệ. Để tránh chuyện đó, có những cơ sở đại học có qui định là các môn học cũng có nhiệm kỳ: ai mà dạy môn nào đó được 4-5 năm rồi thì phải giao cho người khác đảm nhiệm, trừ trường hợp không tìm được người thay thNhiều khoa toán có phân chia việc dạy các môn cho các tổ bộ môn, ví dụ môn “phương trình vi phân” thì chỉ dành cho người của tổ bộ môn phương trình vi phân dạy. Việc phân chia như vậy có cái lợi là đảm bảo chất lượng dạy,đặc biệt là trong điều kiện trình độ giảng viên nói chung còn thấp, phải “chuyên môn hóa” trong việc dạy để đảm bản chất lượng tối thiểu. Tuy nhiên nó có điểm hạn chế, là nó tạo ra xu hướng người của tổ bộ môn nào sẽ chỉ biết chuyên ngành hẹp đấy, tầm nhìn không mở rộng ra. Ỏ một số trường đại học tiên tiến, nơi có nhiều giảng viên trình độ cao (và với nguyên tắc là đã là giáo sư hay giảng viên cao cấp thì đủ trình độ để dạy bất cứ môn nào trong các môn toán bắt buộc ở bậc cử nhân), công việc giảng dạy không phân chia theo tổ bộ môn hẹp như vậy, mà giảng viên (cao cấp) nào cũng có thể đăng ký dạy bất cứ môn nào ở bậc cử nhân.
Tất nhiên, việc thay đổi môn dạy đòi hỏi các giảng viên phải cố gắng hơn trong việc chuẩn bị bài giảng (mỗi lần đổi môn dạy, là một lần phải chuẩn bị bài giảng gần như từ đầu), nhưng đổi lại nó làm tăng trình độ của bản thân giảng viên, giúp cho giảng viên tìm hiểu những cái mới (mà nếu không đổi môn dạy thì sẽ không tìm hiểu, do sức ỳ). Đặc biệt là các môn ở bậc cao học: việc chuẩn bị bài giảng cho một môn cao học mới có thể giúp ích trực tiếp cho việc nghiên cứu khoa học của giảng viên.
Tôi có một số kinh nghiệm cá nhân về việc này. Ví dụ như một lần năm 1999 tôi nhận dạy 1 học kỳ cao học về hệ động lực Hamilton, và trong quá trình đọc tài liệu để chuẩn bị bài giảng cho môn đó, tôi phát hiện ra một số vấn đề cơ bản liên quan đến dạng chuẩn địa phương của hệ động lực chưa được nghiên cứu, và điều đó thúc đẩy tôi nghiên cứu được một số kết quả khá tốt. Năm 2008 tôi nhận dạy môn đại số (mở rộng trường và một ít đại số giao hoán) cho sinh viên toán năm thứ 4, tuy rằng trước đó tôi hầu như không đụng chạm đến những thứ đó. Việc dạy môn đại số đã giúp tôi nắm chắc thêm được một số kiến thức về đại số, ví dụ như hiểu thêm ý nghĩa của tính chất Noether (đây là tính chất đặc trưng của “đại số”, đối ngược với “giải tích”).
Tất nhiên có nhiều người, do điều kiện công việc, phải dạy cùng một môn (ví dụ như môn Toán lớp 12) trong nhiều năm. Để tránh trì trệ trong trường hợp đó, cần thường xuyên cải tiến phương pháp và nội dung giảng dạy (đưa vào những ví dụ minh họa mới và bài tập mới từ thực tế hiện tại, sử dụng những công nghệ mới và công cụ học tập mới, tìm các cách giải thích mới dễ hiểu hơn, v.v.)
2) Nên: Dạy và kiểm tra kiến thức học sinh theo lối “học để hiểu”
Không nên: Tạo cho học sinh thói quen học vẹt, chỉ nhớ mà không hiểu
Các nhà giáo dục học và thần kinh học trên thế giới đã làm nhiều phân tích và thí nghiệm cho thấy, khi bộ óc con người “hiểu” một cái gì đó (tức là có thể “make sense” cái đó, liên tưởng được với những kiến thức và thông tin khác đã có sẵn trong não) thì dễ nhớ nó (do thiết lập được nhiều “dây nối” liên quan đến kiến thức đó trong mạng thần kinh của não — một neuron thần kinh có thể có hàng chục nghìn dây nối đến các neuron khác), còn khi chỉ cố nhồi nhét các thông tin riêng lẻ vào não (kiểu học vẹt) mà không liên hệ được với các kiến thức khác đã có trong não, thì thông tin đó rất khó nhớ, dễ bị não đào thải.
Thực ra thì môn học nào cũng cần “hiểu” và “nhớ”, tuy rằng tỷ lệ giữa “hiểu” và “nhớ” giữa các môn khác nhau có khác nhau: ví dụ như ngoại ngữ thì không có gì phức tạp khó hiểu lắm nhưng cần nhớ nhiều (tất nhiên để nhớ được các câu chữ ngoại ngữ thì cũng phải liên tưởng được các câu chữ đó với hình ảnh hay ỹ nghĩa của chúng và với những thứ khác có trong não), nhưng toán học thì ngược lại: không cần nhớ nhiều lắm, nhưng phải hiểu được các kiến thức, và quá trình hiểu đó đòi hỏi nhiều công sức thời gian. Có những công thức và định nghĩa toán mà nếu chúng ta quên đi chúng ta vẫn có thể tự tìm lại được và dùng được nếu đã hiểu bản chất của công thức và định nghĩa đó, còn nếu chúng ta chỉ nhớ công thức và định nghĩa đó như con vẹt mà không hiểu nó, thì cũng không dùng được nó, và như vậy thì cũng không hơn gì người chưa từng biết nó. Ví dụ như công thức tính Christoffel symbol cho liên thông Riemann của một Riemannian metric là một công thức hơi dài, và tôi chẳng bao giờ nhớ được chính xác nó lâu tuy “mang tiếng” là người làm hình học vi phân: cứ mỗi lần đụng đến thì xem lại, nhớ được một lúc, rồi lại quên. Nhưng điều đó không làm tôi băn khoăn, vì tôi hiểu bản chất của Christoffel symbol và các tính chất cơ bản của liên thông Riemann, từ đó có thể tự nghĩ ra lại được công thức nếu cần thiết (tốn một vài phút) hoặc tra trên internet ra ngay.
Sinh viên ngày nay (là những chuyên gia của ngày mai) có thể tra cứu rất nhanh mọi định nghĩa, công thức, v.v., nhưng để hiểu chúng thì vẫn phải tự hiểu, không có máy móc nào hiểu hộ được. Cách đây 5-10 năm, theo thông lệ của những người dạy trước tôi, tôi thường không cho phép sinh viên mang tài liệu vào phòng thi trong các kỳ thi cuối học kỳ, và đề bài thi hay có 1 câu hỏi lý thuyết (tức là phát biểu đúng 1 định nghĩa hay định lý gì đó thì được điểm). Nhưng trong thời đại mới, việc nhớ y nguyên các định nghĩa và định lý có ít giá trị, mà cái chính là phải hiểu để mà sử dụng được chúng. Bởi vậy những năm gần đây, trong các kỳ thi tôi dần dần cho phép học sinh mang bất cứ tài liệu nào vào phòng thi, và đề thi không còn các câu hỏi “phát biểu định lý” nữa. Thay vào đó là những bài tập (tương đối đơn giản, và thường gần giống các bài có trong các tài liệu nhưng đã thay tham số) để kiểm tra xem học sinh có hiểu và sử dụng được các kiến thức cơ bản không.
Về mặt hình thức, chương trình học ở Việt Nam (kể cả bậc phổ thông lẫn bậc đại học) khá nặng, nhưng là nặng về “nhớ” mà nhẹ về “hiểu”, và trình độ trung bình của học sinh Việt Nam thì yếu so với thế giới (tất nhiên vẫn có học sinh rất giỏi, nhưng tỷ lệ học sinh giỏi thực sự rất ít, và cũng khó so được với giỏi của phương Tây). Vấn đề không phải là do người Việt Nam sinh ra kém thông minh, mà là do điều kiện và phương pháp giáo dục, chứ trẻ em gốc Việt Nam lớn lên ở nước ngoài thường là thành công trong đường học hành. Hiện tượng rất phổ biến ở Việt Nam là học sinh học thuộc lòng các “kiến thức” trước mỗi kỳ kiểm tra, rồi sau khi kiểm tra xong thì “chữ thầy trả thầy”. Việt Nam rất cần cải cách chương trình giáo dục theo hướng tăng sự “hiểu” lên, và giảm sự “học gạo”, “nhớ như con vẹt”. Tôi có phỏng vấn nhiều sinh viên tốt nghiệp loại giỏi ngành toán ở Việt Nam, nhưng khi hỏi một số kiến thức khá cơ bản thì nhiều em lại không biết. Lỗi không phải tại các em mà có lẽ tại hệ thống giáo dục. Nhiều thầy cô giáo chỉ khuyến khích học sinh làm bài kiểm tra giống hệt lời giải mẫu của mình, chứ làm kiểu khác đi, tuy có thể thú vị hơn cách của thầy thì có khi lại bị trừ điểm. Tôi đã chứng kiến trường hợp sinh viên chỉ đạt điểm thi 7-8 lại giỏi hơn sinh viên đạt điểm thi 9-10 vì kiểu chấm thi như vậy. Kiểu chấm điểm như thế chỉ khuyến khích học vẹt chứ không khuyến khích sự sáng tạo hiểu biết.
3) Nên: Dạy những cái cơ bản nhất, nhiều công dụng nhất
Không nên: Mất nhiều thời giờ vào những thứ ít hoặc không dùng đến
Trên đời có rất nhiều cái để học, trong khi thời gian và sức lực của chúng ta có hạn, và bởi vậy chúng ta luôn phải lựa chọn xem nên học (hay dạy học) cái gì. Nếu chúng ta phung phí quá nhiều thời gian vào những cái ít công dụng (hoặc thậm chí phản tác dụng, ví dụ như những lý thuyết về chính trị hay kinh tế trái ngược với thực tế), thì sẽ không còn đủ thời gian để học (hay dạy học) những cái quan trọng hơn, hữu ích hơn.
Tất nhiên, mức độ “quan trọng, hữu ích” của từng kiến thức đối với mỗi người khác nhau thì khác nhau, và phụ thuộc vào nhiều yếu tố như thời gian, hoàn cảnh, sở trường, v.v. Ví dụ như học nói và viết tiếng Việt cho đàng hoàng là không thể thiếu với người Việt, nhưng lại không cần thiết với người Nga. Những người muốn làm nghề toán thì phải học nhiều về toán, còn sinh viên đại học các ngành khác nói chung chỉ cần học một số kiến thức toán cao cấp cơ bản nhất mà sẽ cần trong công việc của họ. Những người muốn làm toán ứng dụng, thì ngoài các môn toán, cần phải học các môn mà họ định mang toán ứng dụng vào đó.
Ngay trong các môn toán, không phải các kiến thức nào cũng quan trọng như nhau. Và “độ quan trọng” và “độ phức tạp” là hai khái niệm khác nhau: không phải cái gì quan trọng cũng phức tạp khó hiểu, và không phải cái gì rắm rối khó hiểu cũng quan trọng. Giảng viên cần tránh dẫn dắt học sinh lao đầu vào những cái rắm rối phức tạp nhưng ít công dụng. Thay vào đó, cần dành nhiều thời gian cho những cái cơ bản, nhiều công dụng nhất. Nếu là cái vừa cơ bản và vừa khó, thì lại càng cần dành đủ thời gian cho nó, vì khí nắm bắt được nó tức là nắm bắt được một công cụ mạnh.
Một ví dụ là đạo hàm và tích phân. Đây là những khái niệm cơ bản vô cùng quan trọng trong toán học. Học sinh cần hiểu định nghĩa, bản chất và công dụng của chúng, và nắm được một số nguyên tắc cơ bản và công thức đơn giản, ví dụ như nguyên tắc Leibniz cho đạo hàm của một tích, hay công thức “đạo hàm của sin x bằng cos x”. Tuy nhiên nếu bắt học sinh học thuộc hàng trăm công thức tính đạo hàm và tích phân khách nhau, thì sẽ tốn thời gian vô ích vì phần lớn các công thức thức đó sẽ không dùng đến sau này, hoặc nếu dùng đến thì có thể tra cứu được dễ dàng. Một lần tôi thấy có một sách tiếng Việt về tính tích phân cho học sinh, dày hơn 150 trang, với rất nhiều công thức phức tạp dài dòng (ví dụ như công thức tính tính phân của một hàm số có dạng thương của hai biểu thức lượng giác), mà ngay những người làm toán chuyên nghiệp cũng rất hiếm khi cần đến. Thay vì tốn nhiều thời gian vào những công thức phức tạp mà không cần dùng đó, học những thứ cơ bản khác sẽ có ích hơn.
Một lần nhà xuất bản Springer có lần nhờ tôi làm phản biện cho 1 quyển sách về hình học vi phân và ứng dụng. Tôi đã khuyên Springer không in sách đó, và một trong các lý do là quyển sách chứa quá nhiều khái niệm mà ngay trong sách đó cũng không dùng đến. Ví dụ như khái niệm “không gian Lindeloff” được đưa vào ngay ở đầu sách, phát biểu thành 1 định nghĩa có đánh số hẳn hoi (chứ không phải là chỉ nhắc qua nó trong một “remark”), nhưng không dùng đến nó lúc nào trong sách, tôi không hiểu người viết sách đưa định nghĩa đó vào trong sách để làm gì.
Một ví dụ khác: các bất đẳng thức. Có những bất đẳng thức “có tên tuổi”, không phải vì nó “khó”, mà là vì nó có ý nghĩa (nó xuất hiện trong các vấn đề hình học, số học, phương trình vi phân, v.v.). Chứ nếu học một đống hàng ngàn bất đẳng thức mà không biết chúng dùng để làm gì, thì khá là phí thời gian. Phần lớn các bất đẳng thức (không kể các bất đẳng thức có tính tổ hợp) có thể được chứng minh khá dễ dàng bằng một phương pháp cơ bản, là phương pháp dùng đạo hàm hoặc sai phân. Phương pháp này học sinh phổ thông có thể học được, nhưng thay vào đó học sinh lại được học các kiểu mẹo mực để chứng minh bất đẳng thức. Các mẹo mực có ít công dụng, chỉ dùng được cho bài toán này nhưng không dùng được cho bài toán khác (bởi vậy mới là “mẹo mực” chứ không phải “phương pháp”). “Mẹo mực” có thể làm cho cuộc sống thêm phong phú, nhưng nếu mất quá nhiều thời gian vào “mẹo mực” thì không còn thời gian cho những cái cơ bản hơn, giúp tiến xa hơn. Như là trong công nghệ, có cải tiến cái đèn dầu đến mấy thì nó cũng không thể trở thành đèn điện.
Hồi còn nhỏ, có lần tôi đi thi học sinh giỏi (lớp 6 ?), có bài toán tìm cực đại. Tôi dùng đạo hàm tính ngay ra điểm cực đại, và có bạn khác cùng lớp cũng biết làm như vậy. Cách làm đó là do chúng tôi tự đọc sách mà ra chứ không được dạy. Nhưng khi viết lời giải thì lại phải giả vờ “đoán mò” điểm cực đại, rồi viết hàm số dưới dạng một số (giá trị tại điểm đó) cộng với một biểu thức hiển nhiên là không âm (ví dụ như vì có dạng bình phương) thì mới được điểm, chứ nếu viết đạo hàm thì mất hết điểm. Nếu như thầy giáo trừ điểm học sinh, vì học sinh giải bài thi bằng một phương pháp “cơ bản” nhưng “không có trong sách thầy”, thì điều đó sẽ góp phần làm cho học sinh học mẹo mực, thiếu cơ bản.
Qua phỏng vấn một số sinh viên đại học và cao học ngành toán của Việt Nam, tôi thấy họ được học nhiều môn “cao cấp”, nhưng vẫn thiếu kiến thức cơ bản. Ví dụ như họ học giải tích hàm, với những định lý trừu tượng khá là khó. Nhưng họ lại không biết công thức Parceval cho chuỗi Fourier là gì, trong khi chuỗi Fourier là một trong những khái niệm giải tích cơ bản và nhiều ứng dụng nhất của toán. Tôi không có ý nói giải tích hàm là “không cơ bản”. Nó là thứ cần thiết. Nhưng nếu những khái niệm và định lý của giải tích hàm chỉ được học một cách hình thức, không có liên hệ với chuỗi Fourier hay với các ví dụ cụ thể khác, thì đó là học “trên mây trên gió”.

Còn tiếp ... Theo http://zung.zetamu.net




#315429 Toán học trong dự báo thời tiết

Gửi bởi Ban Biên Tập trong 09-05-2012 - 21:45

Khí quyển của chúng ta, một cách cơ bản nhất, chính là một hệ khí động lực mà sự tiến triển của nó được mô tả bởi hệ các phương trình chuyển động chất lỏng Navier-Stokes cùng với các phương trình trạng thái và bảo toàn, goi tắt là hệ PES (1). Với một điều kiện ban đầu và điều kiện biên cho trước, bài toán dự báo thời tiết có thể được coi một cách hình thức là bài toán tìm nghiệm phụ thuộc tường minh vào không thời gian của hệ PES nói trên.

Hình đã gửi




Hình đã gửi
Hệ PES này áp dụng cho tất cả các quá trình trong khí quyển từ những xoáy bụi nhỏ mà chúng ta đôi khi bắt gặp trên các con phố, các cụm mây có phân bố như những gợn sóng nhỏ trên bầu trời, cho đến các cơn bão nhiệt đới với quy mô không gian khoảng 1000 km hay các quá trình khí hậu với quy mô thời gian nhiều năm. Tuy nhiên, gần 100 năm kể từ khi bài toán dự báo khí tượng được đặt ra một cách tường minh như vậy bởi Richarson, tất cả nhưng gì chúng ta có thể làm được lại chỉ là một bài toán dự báo thời tiết chỉ với hạn dài nhất đến thời điểm này là 2 tuần với một độ chính xác rất khiêm tốn (2) (tất cả chúng ta hẳn đã có không ít lần bắt gặp các bản tin dự báo sai, ngay cả với các trung tâm dự báo hàng đầu thế giới của Mỹ hay Châu Âu). Mấu chốt của tính dự báo rất khiêm tốn này nằm ở hai khó khăn chính. Thứ nhất, hệ phương trình Navier-Stokes chưa có lời giải chính xác cho đến tại thời điểm hiện tại sau gần 200 năm đưa ra bởi Navier năm 1822. Đây chính là bài toán thiên niên kỷ số ba của viện toán học Clay mà tầm quan trọng của nó đã đi vào tất cả các lĩnh vực sâu sắc nhất của đời sống hàng ngày. Khó khăn thứ hai là do quy mô không thời gian của các quá trình thời tiết biến đổi trong khoảng quá rộng từ các xoáy rối cỡ vài mm cho đến các quá trình sóng hành tinh với quy mô hàng ngàn km mà không một hệ thống thám sát nào có thể ghi nhận đầy đủ để chúng ta cỏ thể đo đạc và tìm hiểu chi tiết.

Cách tiếp cận lý thuyết duy nhất của các nhà khí tượng học đến thời điểm này là sử dụng một tập các giả thiết xấp xỉ gần đúng cho một hiện tượng nào đó để đơn giản hóa hệ PES cho hiện tượng này. Một ví dụ điển hình là các nghiên cứu lý thuyết về bão nhiệt đới. Sư phức tạp của hệ PES cũng như các hiểu biết vật lý‎ còn hết sức khiêm tốn về các quá trình chi tiết trong bão khiến các nhà nghiên cứu khí tượng không thể mô tả một cách giải tích tất cả các quá trình trong bão một cách đầy đủ. Thay vào đó, bão sẽ được chia ra làm các pha phát triển khác nhau mà tương ứng với nó là các xấp xỉ cho pha đó. Ví dụ, trong giai đoạn chín muồi của bão mà cường độ của chúng gần như không thay đổi theo thời gian, nhà khí tượng học Emanuel (Viện công nghệ Massachusetts) năm 1986 đã đề xuất một mô hình trong đó bão được xem như là một chu trình Carnot nhiệt với một cấu trúc đối xứng trục. Cụ thể, bão lấy nhiệt từ bề mặt đại dương với nhiệt độ cao (khoảng 300 K) và sau đó trao đổi với tầng nhiệt độ lạnh hơn ở tầng độ cao khoảng 18 km (khoảng 220 K) trong quá trình chuyển hóa năng lượng ẩn nhiệt có được từ bốc hơi nước sang động năng của bão. Như thế, cường độ cực đại mà một cơn bão có được sẽ được quy đinh bởi nhiệt độ bề mặt biển tương ứng. Đây là một kết luận rất quan trọng cho phép khảo sát mối liên hệ giữa sự ấm nóng toàn cầu và cường độ bão trong tương lai. Trong pha phát triển của bão, chúng ta tuy nhiên lại không thể giả thiết các cơn bão là ở trạng thái dừng như ở trên. Thay vào đó, chúng ta phải xấp xỉ hệ PES theo một cách khác mà cho phép chúng ta tìm được nghiệm phụ thuộc tường minh vào thời gian. Các mô hình lý‎ thuyết khác nhau cho pha phát triển này của bão đều cho thấy bão có thể xem như một hệ động lực không cân bằng và sự phát triển của bão chính là một quá trình hồi tiếp dương của một hệ bất ổn định. Nhưng ngay cả trong những xấp xỉ này, phần lớn các hệ đơn giản hóa vẫn quá phức tạp đến nỗi chưa ai chứng minh được nghiệm giải tích liệu có tồn tại, và nếu có tồn tại liệu nghiệm đó có duy nhất. Chỉ trong một vài tình huống rất đặc biệt lý‎ tưởng hóa thì chúng ta mới thu được các kết quả giải tích. Trong các trường hợp còn lại hệ phương trình thu được vẫn quá phức tạp để chúng ta có thể hi vọng vào một nghiệm đầy đủ.

Các bài toán khí tượng lý‎ thuyết một mặt là rất cần thiết bởi chúng cho phép chúng ta hiểu sâu hơn về quá trình động lực học trong khí quyển. Nhưng mặt khác rõ ràng là các khó khăn cả về mặt lý thuyết lẫn thực tế nêu ở trên lại không cho phép chúng ta tìm hiểu chi tiết bản chất của khí quyển như chúng ta trông đợi. Vậy dự báo thời tiết mà chúng ta theo dõi hàng ngày đang diễn ra bằng cách nào? Quy trình dự báo thời tiết nghiệp vụ hiện đại là tổ hợp của rất nhiều các cấu thành bao gồm các bộ phận xử lý ảnh vệ tinh, thu phát dữ liệu thám sát, kiểm tra đánh giá, v.v., nhưng cốt lõi nhất của tất cả các trung tâm dự báo hiện nay là một mô hình máy tính giải hệ PES nêu ở trên bằng phương pháp số, gọi tắt là dự báo thời tiết số (NWP). Nói một cách đơn giản, chúng ta chia khí quyển liên tục của chúng ta thành một khí quyển rời rạc được mô tả bởi một mạng các điểm nút theo các phương ngang và thẳng đứng với một độ phân giải cho trước. Hệ PES khi đó được chuyển thành một hệ các phương trình cho các điểm nút này, và chúng ta khi đó sẽ mô phỏng khí quyển rời rạc này trên các siêu máy tính. Các kết quả thu được từ các mô phỏng sẽ được xem như là trạng thái khí quyển trong tương lai và từ đó đưa ra các bản tin dự báo. Theo nghĩa này, NWP chính là bài toán tích phân số hệ phương trình đạo hàm riêng từ một điều kiện ban đầu và điều kiện biên cho trước. Do đặc thù của hệ PES này phụ thuộc mạnh vào điều kiện ban đầu, một cách rõ ràng là các mô hình dự báo thời tiết được xây dựng dựa trên hệ PES phải có được điều kiện ban đầu (biên) tốt nhất có thể. Các nghiên cứu của Lorenz từ đầu nhưng năm 1960 đã chỉ ra rằng một sự thay đổi nhỏ của điều kiện ban đầu có thể dẫn đến một sự thay đổi rất lớn của trạng thái tích phân theo thời gian. Sự phụ thuộc mạnh của dự báo thời tiết vào điều kiện ban đầu này đôi khi dẫn đến các kết quả hoàn toàn sai lệch so với trạng thái thực của khí quyển chỉ do một vài sai số quan trắc nào đó (3). Đây chính là giới hạn lớn nhất của bài toán dự báo thời tiết số vì dường như chúng ta sẽ không bao giờ có thể có được một hệ thống thám sát toàn cầu đủ chính xác với một độ phân giải tuỳ ý. Các nghiên cứu lý thuyết của Lorenz từ năm 1960 đã đặt ra một giới hạn trên cho hạn dự báo của các mô hình số là vào khoảng 2 tuần, và đây được xem như là điểm tới hạn của bài toán dự báo thời tiết NWP.

Song song với bài toán tạo trường ban đầu tốt nhất, lớp bài toán có tầm quan trọng không kém trong các mô hình dự báo số chính là các thuật toán sai phân hóa hệ PES. Đây là nguồn gốc của các sai số nội tại của mô hình mà trong suốt các thập kỷ vừa qua hàng loạt các thuật toán sai phân hữu hạn hay tích phân phổ đã được liên tục phát triển và thử nghiệm để giảm tối đa các nghiệm phi vật lý của thuật toán. Cùng với đòi hỏi độ phân giải ngày càng cao của các mô hình để có thể chi tiết hóa các quá trình vật lý thì các đòi hỏi về khả năng tính toán cũng tăng lên rất nhanh đến mức độ phân giải khả thi nhất của chúng ta hiện nay cũng chỉ vào khoảng 10 km đối với các bài toán nghiệp vụ, một độ phân giải rất thô cho các quá trình mưa hay lốc xoáy. Thêm vào đó, sự hiểu biết không đầy đủ về các quá trình vật lý khí quyển cũng ngăn cản chúng ta có được nhưng biểu diễn chính xác của các quá trình vật lý trong mô hình.

Tất cả nhưng khó khăn về mặt lý thuyết cũng như khả năng quan trắc ở trên đã và đang hạn chế nghiêm trọng đến năng lực dự báo thời tiết của con người mà hiện tại chúng ta vẫn chưa vượt qua được. Bài toán dự báo thời tiết do đó vẫn đang là một bài toán chưa có lời giải.

----------
1) PES trong bài này là viết tắt của Primitive Equation Systems. Một cách tổng quát, hệ PES trong khí tượng bao gồm phương trình chuyển động Navier-Stokes, phương trình trạng thái, phương trình bảo toàn khối lượng, và phương trình bảo toàn năng lượng. Trong một số tài liệu, hệ PES có thể bao gồm các phương trình Navier-Stokes và phương trình bảo toàn khối lượng.
2) Dự báo thời tiết trong khuôn khổ đề cập ở đây là các dự báo dựa trên hệ động lực PES. Các dự báo bằng phương pháp thống kê nói chung có thể cho các bản tin dự báo xa hơn (với độ chính xác tuy nhiên kém hơn). Kiểu dự báo thống kê này tuy nhiên lại không mang thuộc tính động lực học của khí quyển và sẽ không được đề cập ở đây.
3) Một ví dụ minh hoạ của Lorenz cho sự phụ thuộc vào điều kiện ban đầu này chính là hiệu ứng “con bướm” mà một sự vỗ cánh nhỏ của nó tại Brazil được mường tượng là có thể tạo ra các cơn lốc xoáy ở Texas, Bắc Mỹ.



Theo Tia Sáng




#315427 Ngôn ngữ phổ quát của các con số

Gửi bởi Ban Biên Tập trong 09-05-2012 - 21:43

Vâng, thành thật xin lỗi bạn, chúng tôi đã sửa lại. Cảm ơn bạn


#314786 Phương pháp đại số (phương pháp gien)

Gửi bởi Ban Biên Tập trong 06-05-2012 - 23:01

5. Xây dựng nghiệm
Có nhiều bài toán không yêu cầu tìm tất cả các nghiệm của phương trình, mà chỉ yêu cầu chứng minh phương trình có vô số nghiệm. Trong trường hợp như thế, ta chỉ cần xây dựng một họ nghiệm chứa tham số là đủ. Việc xây dựng như thế có thể được thực hiện bằng các giả định, giới hạn miền nghiệm. Các siêu phẳng là những miền giới hạn thông dụng.

Ví dụ 5.1. (Italy 1996) Chứng minh rằng phương trình $a^2 + b^2 = c^2 + 3$ có vô số nghiệm nguyên $(a, b, c)$.
Lời giải: Chọn $c = b+1$ thì ta được phương trình $a^2 = 2b + 4, \ \ (*)$. Bây giờ chỉ cần chọn $a = 2k, b = 2(k^2-1)$ là ta được nghiệm của $(*)$ và như vậy phương trình ban đầu có họ nghiệm $a = 2k, b = 2(k2-1), c = 2k^2 – 1$.

Ví dụ 5.2. (Canada 1991) Chứng minh rằng phương trình $x^2 + y^3 = z^5$ có vô số nghiệm nguyên dương.

Lời giải: Chú ý rằng $2^m + 2^m = 2^{m+1}$. Đặt $x = 2^{\frac{m}{2}}, y = 2^{\frac{m}{3}}, z = 2^{\frac{m+1}{5}}$, khi đó $x^2 + y^3 = z^5$. Ta chỉ cần tìm $m$ sao cho $\frac{m}{2}, \frac{m}{3}, \frac{m+1}{5}$ nguyên là xong. Đây là một bài toán bậc nhất đơn giản và ta có thể tìm được $ m = 6(5k+4)$.

Ví dụ 5.3. Chứng minh rằng với mọi $n \geq 2$, phương trình $x^2 + y^2 = z^n$ luôn luôn có vô số nghiệm nguyên dương.

Lời giải: Xét số phức $\alpha = a + bi$. Giả sử $\alpha ^n = x + yi$ thì ta có
\[\sqrt {{x^2} + {y^2}} = |{\alpha ^n}| = |\alpha {|^n} = {\left( {\sqrt {{a^2} + {b^2}} } \right)^n}\]
Từ đó $x^2 + y^2 = (a^2 + b^2)^n$.

Ví dụ 5.4. Chứng minh rằng $x^3 + y^3 + z^3 = 2$ có vô số nghiệm nguyên.
Lời giải. Đặt $x = 1 – a, y = 1 + a$ thay vào phương trình, ta được $6a^2 + z^3 = 0$
Từ đây, nếu chọn $a$ sao cho $-6a^2$ là một lập phương đúng thì ta sẽ được $(1-a; 1+a, (1 - a;1 + a;\sqrt[3]{{ - 6{a^2}}})$ là một nghiệm nguyên của phương trình. Rõ ràng có thể chọn $a = -6k^3$.
Vậy $(1-6k^3, 1+6k^3, -6k^2)$ với $k$ nguyên là một họ nghiệm của phương trình đã cho. Do đó, phương trình đã cho có vô số nghiệm nguyên.

Ví dụ 5.5. Chứng minh rằng phương trình $x^4 + y^4 + z^4 = 2$ có vô số nghiệm hữu tỷ.
Lời giải. Ta tìm các nghiệm $(x, y, z)$ trong mặt phẳng $z = x + y$. Khi đó
$$x^4 + y^4 + z^4 = x^4 + y^4 + (x+y)^4 = 2(x^2+y^2+xy)^2$$
Từ đó, nếu ta chứng minh được phương trình $x^2 + xy + y^2 = 1$ có vô số nghiệm hữu tỷ thì bài toán được giải quyết. Về cách giải bài toán này, xem phương pháp cát tuyến trong phần 7.

Bài tập

5.6. Dựa vào hằng đẳng thức
$$[2(3x+2y+1)+1]^2 – 2(4x+3y+2)^2 = (2x+1)^2 – 2y^2$$
chứng minh rằng phương trình $x^2 + (x+1)^2 = y^2$ có vô số nghiệm nguyên dương.

5.7. Dựa vào hẳng đẳng thức
$$[2(7y+12x+6)]^2 – 3[2(4y+7x+3)+1]^2 = (2y)^2 – 3(2x+1)^2$$
chứng minh rằng phương trình $(x+1)^3–x^3=y^2$ có vô số nghiệm nguyên dương.

5.8. Chứng minh rằng tồn tại vô số các cặp số hữu tỷ dương $(x, y)$ sao cho $x^3 + y^3 = 9$.

5.9. (Bulgaria 1999) Chứng minh rằng phương trình $x^3 + y^3 + z^3 + t^3 = 1999$ có vô số nghiệm nguyên.

5.10. Chứng minh rằng với mọi $n \geq 2$, luôn tồn tại $n$ số nguyên dương có tổng bằng tích.

5.11. (IMO 82) Chứng minh rằng nếu $n$ là số nguyên dương sao cho phương trình $x^3 – 3xy^2 + y^3 = n$ có nghiệm nguyên $(x, y)$, thì phương trình này có ít nhất 3 nghiệm như vậy. Chứng minh rằng phương trình đã cho không có nghiệm khi $n = 2891$.

5.12. Chứng minh rằng phương trình $(x^2+x+1)(y^2+y+1) = z^2 + z + 1$ có vô số nghiệm nguyên.

5.13. (THTT 4/187, dự tuyển IMO 92) Chứng minh rằng, với số nguyên dương m bất kỳ sẽ tồn tại vố số các cặp số nguyên $(x, y)$ sao cho:
1) $x$ và $y$ nguyên tố cùng nhau
2) $y$ chia hết $x^2 + m$;
3) $x$ chia hết $y^2 + m$.

5.14. (Saint Peterburg 2003) Chứng rằng tồn tại các số nguyên dương $a > 1, b > 1, c > 1$ sao cho $a^2 – 1$ chia hết cho $b$, $b^2 – 1$ chia hết cho $c$ và $c^2 – 1$ chia hết cho $a$ và $a + b + c > 2003$.

5.15. Chứng minh rằng phương trình $x^2 + y^2 + z^2 = xyz$ có vô số nghiệm nguyên dương.

6. Phương pháp số học
Các tính chất của số nguyên liên quan đến số nguyên tố, ước số chung, bội số chung như Định lý cơ bản của số học, các định lý Fermat, Euler, Wilson … đóng một vai trò quan trọng trong việc tìm kiếm lời giải của phương trình Diophant. Chúng tôi nhắc lại một số định lý (không chứng minh) và đưa ra một số ví dụ áp dụng.

Định lý 6.1. (Bổ đề)
1) Cho $n > 1$ là một số nguyên dương. Nếu $a, b, c$ là các số nguyên thỏa mãn điều kiện $a.b = c^n$ và $(a, b) = 1$ thì $a = (a’)^n, b = (b’)^n$ với các số nguyên $a’, b’$ nào đó.
2) Nếu $a$ hữu tỷ, $a^n$ nguyên với $n$ nguyên dương nào đó thì $a$ nguyên.
3) Nếu $a$ nguyên $\sqrt[n]{a}$ hữu tỷ thì $\sqrt[n]{a}$ nguyên.

Định lý 6.2. (Định lý nhỏ Fermat) Nếu $p$ là số nguyên tố và $a$ là một số nguyên tùy ý thì $a^p – a$ chia hết cho $p$. Nếu $(a, p) = 1$ thì $a^p-1 \equiv 1 (\mod p)$.

Định lý 6.3. (Định lý Euler). Nếu $m$ là số nguyên dương, $(a, m) = 1$ thì $a^{\varphi (m)} \equiv 1
(\mod m)$, trong đó $\varphi $ là Phi-hàm Euler – số các số nguyên dương nhỏ hơn $m$ nguyên tố cùng nhau với $m$.

Định lý 6.4. (Định lý Wilson). $p$ là số nguyên tố khi và chỉ khi $(p-1)! + 1$ chia hết cho $p$.

Định lý 6.5. (Định lý Fermat-Euler) Nếu $p = 4k+1$ thì tồn tại các số nguyên dương $a, b$ sao cho $p = a^2 + b^2$.

Định lý 6.6. (Một tính chất quan trọng) Cho p là số nguyên tố dạng $4k+3$ và $(a, b) = 1$. Khi đó $a^2+b^2$ không chia hết cho $p$.

Ví dụ 6.1: (Phương trình Pythagore) Tìm nghiệm tổng quát của phương trình $x^2 + y^2 = z^2$ trong tập hợp các số nguyên dương.
Giải bài toán này ta có kết quả sau mà ta phát biểu như một định lý:
Định lý 6.7. Mọi nghiệm nguyên dương của phương trình $x^2 + y^2 = z^2$ đều có thể viết dưới dạng $x = (m^2-n^2)k, y = 2mnk, z = (m^2+n^2)k$ hoặc $x = 2mnk, y = (m^2-n^2)k, z = (m^2+n^2)k$, trong đó các số nguyên $m, n, k$ thỏa mãn các điều kiện sau:
  • $(m, n) = 1, (x, y) = k$
  • các số $m, n$ khác tính chẵn lẻ
  • $m > n > 0, k > 0$.
Chứng minh.
Giả sử $(x, y) = k$, khi đó $x = ka, y = kb$ trong đó $(a, b) = 1$. Ta có
$$(ka)^2 + (kb)^2 = z^2 \Leftrightarrow a^2 + b^2 = {\left( {\frac{z}{k}} \right)^2}$$.
Đặt $z = kc, c \in \mathbb{Q}$, khi đó $a^2 + b^2 = c^2$. Do $c \in \mathbb{Q}, c^2 \in \mathbb{N}$, nên theo định lý 6.1, $c \in \mathbb{N}$ . Vì $(a, b) = 1$ nên ít nhất 1 trong hai số $a, b$ phải lẻ. Giả sử rằng $b$ lẻ, khi đó $a^2, b^2$ khi chia 4 dư 1, như thế $c^2$ chia 4 dư 2, điều này không thể vì $c^2$ chia hết cho 2 mà không chia hết cho 4. Vậy $b$ chẵn và như thế $c^2 = a^2 + b^2$ lẻ. Ta có
$${b^2} = (c - a)(c + a) \Leftrightarrow {\left( {\frac{b}{2}} \right)^2} = \frac{{c - a}}{2}.\frac{{c + 1}}{2}$$
Dễ dàng kiểm tra rằng $\frac{c-a}{2}, \frac{c+a}{2}$ là các số nguyên nguyên tố cùng nhau. Như thế, theo định lý 6.1, tồn tại các số nguyên dương $m, n$ sao cho $\frac{c-a}{2} = n^2, \frac{c+a}{2} = m^2$, từ đó $c = m^2 + n^2, a = m^2 – n^2;b = 2mn$, trong đó $(m, n) = 1$.

Ví dụ 6.2: (Lebesgue) Giải phương trình $x^2 – y^3 = 7$ trong tập hợp các số tự nhiên.
Lời giải: Nếu $y$ là số chẵn, tức là $y = 2k$ thì $x = 8k^3 + 7$ chia 8 dư 7 là điều không thể. Vậy $y$ lẻ. Ta có
$$x^2 + 1 = y^3 + 8 \Leftrightarrow x^2 + 1 = (y+2)(y^2 – 2y + 4)$$
Nếu $y$ chia 4 dư 1 thì $y+2$ có dạng $4k+3$. Nếu $y$ chia 4 dư 3 thì $y^2 – 2y + 4$ có dạng $4k+3$. Vì vậy, trong mọi trường hợp, vế trái đều có ước dạng $4k+3$ và do đó có ước nguyên tố dạng $4k+3$, điều này mâu thuẫn với định lý 6.6.

Ví dụ 6.3: (Euler) Chứng minh rằng phương trình $4xy – x – y = z^2$ không có nghiệm nguyên dương.

Hướng dẫn: Viết phương trình dưới dạng $(4x – 1)(4y – 1) = 4z^2 + 1$ và sử dụng định lý 6.6.

Ví dụ 6.4: a) Cho $x, y, z$ là các số nguyên thỏa mãn điều kiện $\frac{x}{y} + \frac{y}{z} + \frac{z}{x}$ nguyên. Chứng minh rằng $xyz$ là lập phương của một số nguyên.
b) Tìm nghiệm nguyên của phương trình $\frac{x}{y} + \frac{y}{z} + \frac{z}{x}=3$.
Hướng dẫn: Viết phương trình dưới dạng $x^2z + y^2x + z^2y = kxyz$. Gọi $p$ là ước nguyên tố của $(x, y)$. Hãy chứng minh rằng $xyz$ chia hết cho $p^3$.

Bài tập

6.5. Giải phương trình trong tập hợp các số nguyên dương
\[\frac{1}{x} + \frac{1}{y} = \frac{1}{{2003}}\]

6.6. Tìm nghiệm nguyên của phương trình $x^3 – 5y^2 = 13$.

6.7. Chứng minh rằng phương trình $x^3 + 3 = 4y(y+1)$ không có nghiệm nguyên.

6.8*. Chứng minh rằng phương trình $x^7 + y^7 = 1998^z$ không có nghiệm nguyên dương.

6.9*. Chứng minh rằng nếu $p$ là số nguyên tố, $n$ là số nguyên dương thì phương trình
$$x(x+1) = p^{2n}y(y+1)$$
không có nghiệm nguyên dương.

6.10*. Tìm tất cả nghiệm của phương trình
$$y^2 = x^3 +(x+4)^2$$
trong tập hợp các số nguyên.

6.11. Chứng minh rằng nếu $c$ là số nguyên dương lẻ thì phương trình
$$x^2 – y^3 = (2c)^3 – 1$$
không có nghiệm nguyên dương.

6.12. (Euler) Chứng minh rằng phương trình
$$4xyz – x – y – t^2 = 0$
không có nghiệm nguyên dương.

6.13. (Nga 1997) Tìm tất cả các cặp số nguyên tố sao cho $p^3 – q^5 = (p+q)^2$.

6.14*. (Legendre) Chứng minh rằng phương trình $ax^2 + by^2 = cz^2$ trong đó $a, b, c$ là các tham số không có ước chính phương, đôi một nguyên tố cùng nhau có nghiệm nguyên dương khi và chỉ khi hệ phương trình đồng dư sau có nghiệm $(\alpha, \beta, \gamma)$:
\[bc \equiv {\alpha ^2}(\mod a), ca \equiv {\beta ^2}(\mod b), ab \equiv - {\gamma ^2}(\mod c)\].

7. Phương pháp hình học
Hình học có những ứng dụng rất bất ngờ trong việc giải các bài toán số học. Chúng ta chắc chắn còn nhớ bài toán của IMO 42 “Cho các số nguyên dương $a, b, c, d$ với $a > b > c > d > 0$. Giả sử $ac + bd = (b+d+a-c)(b+d-a+c)$. Chứng minh rằng $ab + cd$ không phải là số nguyên tố” đã được giải hết sức ấn tượng bằng … định lý hàm số cosin và định lý Ptolémé. Dưới đây, ta sẽ xét hai ví dụ ứng dụng của hình học trong số học với hai phương pháp tiếp cận khác nhau.
Phương pháp thứ nhất được gọi là phương pháp cát tuyến, sử dụng ý tưởng của hình học giải tích vào việc nghiên cứu các điểm nguyên và điểm hữu tỷ trên đường cong. Chính hướng đi này đã dẫn đến khái niệm đường cong elliptic, một trong những viên gạch cơ bản đặt nền móng cho việc chứng minh định lý lớn Fermat. Ở đây, chúng ta chỉ giới hạn ở một ví dụ nhỏ.
Ví dụ 7.1: Tìm tất cả các nghiệm nguyên khác $(0, 0, 0)$ của phương trình
$$x^2 + 2y^2 = 3z^2, \ \ (1)$$
Lời giải: Chia hay vế của phương trình cho $z^2$, ta được phương trình
$${\left( {\frac{x}{z}} \right)^2} + 2{\left( {\frac{y}{z}} \right)^2} = 3$$
Đặt $u = \frac{x}{z}, v = \frac{y}{z}$, ta được phương trình
$$u^2 + 2v^2 = 3, \ \ (2)$$
trong đó $u, v$ hữu tỷ. Bài toán quy về việc tìm tất cả các nghiệm hữu tỷ của $(2)$
Ta cần tìm tất cả các điểm hữu tỷ nằm trên đường cong $(E): u^2 + 2v^2 = 3$. Chú ý rằng $(1, 1)$ là một điểm hữu tỷ của $(E)$. Nếu $(u_0, v_0)$ là một điểm hữu tỷ khác $(1, 1)$ thì đường thẳng qua $(1, 1)$ và $(u_0, v_0)$ sẽ có hệ số góc hữu tỷ. Mặt khác, nếu $y = k(x-1) + 1$ là đường thẳng qua $(1, 1)$ với hệ số góc $k$ hữu tỷ thì, áp dụng định lý Viet cho phương trình hoành độ giao điểm, giao điểm thứ hai của đường thẳng trên với $©$ cũng có tọa độ hữu tỷ. Tính toán trực tiếp ta có tọa độ của điểm này là
\[u = \frac{{2{k^2} - 4k - 1}}{{2{k^2} + 1}},\;v = \frac{{ - 2{k^2} - 2k + 1}}{{2{k^2} + 1}}\]
Từ đây ta cũng tìm được nghiệm tổng quát của $(1)$. Ví dụ với $k=-1$ ta được $u = \frac{5}{3}, v = \frac{1}{3}$ và ta có nghiệm $(5, 1, 3)$ của $(1)$.

Phương pháp thứ hai là phương pháp điểm nguyên, lưới nguyên. Rất nhiều định lý sâu sắc của số học được chứng minh bằng cách tính số điểm nguyên trong một miền. Chẳng hạn Luật thuận nghịch bình phương cho ký hiệu Legendre đã được chứng minh như vậy. Dưới đây, chúng ta xét một ứng dụng khác của phương pháp lưới nguyên.

Ví dụ 7.2: Bổ đề Minkowsky và định lý Minkowsky
Định lý Minkowsky là một ví dụ rất thú vị về ứng dụng của hình học trong lý thuyết số. Chúng ta bắt đầu từ một kết quả rất đơn giản nhưng hữu ích
Bổ đề 7.1. Trên mặt phẳng cho hình $F$ có diện tích lớn hơn 1. Khi đó tồn tại hai điểm $A, B \in F$, sao cho véctơ $\overrightarrow {AB}$ có tọa độ nguyên.
Chứng minh: Lưới nguyên cắt hình $F$ thành các mẩu nhỏ. Chồng các mẩu này lên nhau, do tổng diện tích của các mẩu lớn hơn 1, nên có ít nhất 2 mẩu có điểm chung. Gọi $A, B$ là hai điểm nguyên thuỷ ứng với điểm chung này thì $A, B$ là hai điểm cần tìm.

Bổ đề 7.2. (Bổ đề Minkowsky) Trên mặt phẳng cho hình lồi $F$ nhận gốc tọa độ làm tâm đối xứng và có diện tích lớn hơn 4. Khi đó nó chứa một điểm nguyên khác gốc tọa độ.
Chứng minh: Xét phép vị tự tâm $O$, tỷ số $\frac{1}{2}$ , biến $F$ thành $G$. Do $G$ có diện tích lớn hơn 1 nên theo bổ đề 1, tồn tại hai điểm $A, B$ thuộc $G$ sao cho véc-tơ $\overrightarrow {AB}$ có toạ độ nguyên. Gọi $A’$ là điểm đối xứng với $A$ qua $O$. Do hình $G$ đối xứng qua gốc toạ độ nên $A’$ thuộc $G$. Do $G$ lồi nên trung điểm $M$ của $A’B$ thuộc $G$. Gọi $N$ là điểm đối xứng của $O$ qua $M$ thì $N$ thuộc $F$ và $ON = AB$, suy ra $N$ là điểm nguyên khác $O$ (đpcm).

Định lý 7.3. (Định lý Minkowsky) Cho $a, b, c$ là các số nguyên, trong đó $a > 0; ac – b^2 = 1$. Khi đó phương trình
$$ax^2 + 2bxy + cy^2 = 1$$
có nghiệm nguyên.

Bài tập

7.3. Tìm tất cả các cặp $(x, y)$ các số hữu tỷ dương sao cho $x^2 + 3y^2 = 1$

7.4. Chứng minh rằng một đường cong bậc hai bất kỳ hoặc không chứa điểm hữu tỷ nào, hoặc chứa vô số điểm hữu tỷ.

7.5. Hãy tìm ví dụ một đường cong bậc hai không chứa điểm hữu tỷ nào.

7.6*. Chứng minh rằng nếu $D$ là số nguyên không chính phương thì phương trình $x^2 – Dy^2 = 1$ luôn có nghiệm nguyên dương.

7.7. Cho $p, q$ là các số nguyên dương nguyên tố cùng nhau. Bằng cách đếm các điểm nguyên, hãy chứng minh công thức
\[\left[ {\frac{q}{p}} \right] + \left[ {\frac{{2q}}{p}} \right] + ... + \left[ {\frac{{(p - 1)q}}{p}} \right] = \frac{{(p - 1)(q - 1)}}{2}\].


#312541 Phương pháp đại số (phương pháp gien)

Gửi bởi Ban Biên Tập trong 25-04-2012 - 00:08

BBT: Đây là bài viết của TS Trần Nam Dũng, ĐHKHTN Tp HCM từng được post trên Diễn đàn toán học, bài viết viết xong tháng 7/2004, bổ sung và chỉnh lý, tháng 9/2009.


1. Mở đầu
Phương trình nghiệm nguyên hay còn được gọi là phương trình Diophant là một trong những dạng toán lâu đời nhất của Toán học. Từ Euclid, Diophantus, qua Fibonacci, rồi đến Fermat, Euler, Lebesgue … và thời hiện đại là Gelfold, Matiasevic, Shenzel, Serpinsky … phương trình Diophant đã trải qua một lịch sử phát triển lâu dài.


Hình đã gửi


Thông qua việc giải các phương trình Diophant, các nhà toán học đã tìm ra được những tính chất sâu sắc của số nguyên, số hữu tỷ, số đại số. Giải phương trình Diophant đã đưa đến sự ra đời của Liên phân số, Lý thuyết đường cong elliptic, Lý thuyết xấp xỉ Diophant, Thặng dư bình phương, Số học modular …


Hình đã gửi
Trong các kỳ thi học sinh giỏi quốc gia và quốc tế, phương trình Diophant vẫn thường xuyên xuất hiện dưới các hình thức khác nhau và luôn được đánh giá là khó do tính không mẫu mực của nó.

Bài giảng này có mục đích đưa ra một số phương pháp cơ bản để tấn công các bài toán về phương trình Diophant. Tuy nhiên, với khả năng còn hạn hẹp của mình, chúng tôi hoàn toàn không có tham vọng bao quát hết các vấn đề về phương trình Diophant. Chúng tôi chủ yếu chỉ giới hạn trong các phương trình đa thức, bỏ qua các phương trình Diophant bậc nhất và không đề cập đến các phương trình có chứa hàm mũ.

Đây là một tập tài liệu mở, một số chứng minh được bỏ qua, một số lời giải chỉ trình bày sơ lược hoặc bỏ qua. Chúng tôi rất mong nhận được ý kiến đóng góp của quý thầy cô, quý anh chị và bạn bè đồng nghiệp, cũng như của các em sinh viên, học sinh để tập tài liệu được hoàn thiện hơn.

Chúng tôi xin chân thành cảm ơn GS Nguyễn Văn Mậu và Trường Đại học Khoa học Tự nhiên ĐHQG HN đã tổ chức khoá bồi dưỡng này và cho phép chúng tôi được báo cáo trước các thầy cô, anh chị và các bạn.

Phiên bản 2.0 này của bài viết đã được sửa chữa, chỉnh lý và bổ sung thêm một số vấn đề sau:
1) Có lời giải và phân tích chi tiết cho một số ví dụ và bài tập
2) Bổ sung một số kết quả quan trọng liên quan đến phương trình Pell
3) Bổ sung thêm một số bài tập mới
4) Bổ sung thêm phần đọc thêm về đường cong hữu tỷ và đường cong elliptic.

2. Phương pháp chọn mô-đun
Một số chính phương không thể tận cùng bằng 2, 3, 7, 8. Một số chính phương chia 3 dư 0 hoặc 1. Một số chính phương chia 8 dư 0, 1 hoặc 4. Những tính chất đơn giản đó nhiều khi lại là chìa khóa để giải nhiều phương trình Diophant. Và đó chính là ý tưởng chính của phương pháp chọn mô-đun.

Ví dụ 2.1: (Việt Nam 2003, bảng B) Hỏi có tồn tại hay không các số nguyên $x, y, u, v, t$ thỏa mãn điều kiện sau


$$x^2 + y^2 = (x+1)^2 + u^2 = (x+2)^2 + v^2 = (x+3)^2 + t^2$$

Lời giải: Không.
Cách 1: Thật vậy, giả sử hệ trên tồn tại nghiệm. Đặt $N$ là giá trị chung của cả 4 biểu thức.
Ta có $x^2 \mod 8 \in \{ 0, 1, 4 \}$, suy ra $x^2 + y^2 \in \{ 0, 1, 2, 4, 5 \}$. Ngoài ra, nếu $x$ chạy qua một hệ thặng dư đầy đủ $\mod 4$ thì $x^2 \mod 8$ sẽ là $(0, 1, 0, 4)$ hay một hóan vị vòng quanh của bộ trên.
Như thế $N \mod 8 \in \{0, 1, 4\} \cap \{1, 2, 5\} \cap \{0, 1, 4\} \cap \{0, 4, 5\} = \varnothing$. Mâu thuẫn.
Vậy không tồn tại các số nguyên $x, y, u, v, t$ thỏa mãn điều kiện đầu bài.

Cách 2: Từ hai phương trình đầu ta suy ra $y^2 + v^2 – 2u^2 = 2(x+1)^2 – x^2 – (x+2)^2 = - 2$. Như thế, $y^2 + v^2 = 2(u^2-1)$, suy ra $y, v$ cùng tính chẵn lẻ. Nếu $y, v$ cùng lẻ thì về trái chia $8$ dư $2$, suy ra $u$ chẵn, nhưng khi đó, vế trái chia $8$ dư $-2$, mâu thuẫn.

Ví dụ 2.2. Chứng minh rằng phương trình $x^2 – 3y^2 = –1$ không có nghiệm nguyên dương.
Lời giải: $x^2+1$ không thể chia hết cho $3$

Ví dụ 2.3. (Đề đề nghị IMO 1981) Cho phương trình nghiệm nguyên $z^2 = (x^2-1)(y^2-1) + n$. Hỏi phương trình có nghiệm không nếu
(a) $n = 1981$
(b) $n = 1985$
© $n = 1984$?
Lời giải
(a)Ta có $z^2 \equiv (x^2-1)(y^2-1) + 5 \mod 8$. Vì
$$z^2 \equiv 0, 1, 4 \mod 8, x2 – 1 \equiv 0, 3, 7 \mod 8, (x2-1)(y2-1) \equiv 0, 1, 5 \mod; (x2-1)(y2-1) + 5 \equiv 2, 5, 6 \mod 8$$
nên ta có $z^2 \neq (x^2-1)(y^2-1) + 5 \mod 8$. Mâu thuẫn
(b)Tương tự, xét $\mod 9$.
© $n = 1984$. Rút gọn phương trình, ta được $x^2 + y^2 + z^2 – x^2y^2 = 1985,\ \ (1)$. Ý tưởng của ta là đi tìm biểu diễn $x^2 + y^2 = 1985$. Khi đó $z = xy$ cho ta nghiệm. Bằng cách xét chữ số cuối cùng, ta nhanh chóng đi đến nghiệm $72 + 442$ và $312 + 322$ bằng phương pháp và sai. Từ đó được các nghiệm $(x, y, z) = (7, 44, 7.44)$ và $(31, 32, 31.32)$
Ghi chú: phương trình $(1)$ có vô số nghiệm nguyên dương, hãy thử chứng minh.

Bài tập
2.4. (Thụy Điển 2003) Tìm tất cả các cặp số nguyên dương $x, y$ sao cho
$$x^2 – 3xy = 2002$$.

2.5. Tìm nghiệm nguyên của các phương trình sau
1) $x^2 – 2y^2 = 3$
2) $2x^2 – 5y^2 = 7$
3) $5x^2 + 6x + 11 = y^2 + 4y$

2.6. Tìm nghiệm nguyên của phương trình $x^{10} + y^{10} – z^{10} = 1999$.

2.7. Chứng minh rằng phương trình $x^2 – 2y^2 + 8z = 3$ không có nghiệm nguyên $x, y, z$.

2.8. Tìm tất cả các giá trị nguyên dương $1 < k < 10$ sao cho hệ phương trình $x^2 + ky^2 = z^2, kx^2 + y^2 = t^2$ có nghiệm nguyên dương.

2.9. Chứng minh rằng phương trình $x^2 + y^2 + z^2 + x + y + z = 1$ không có nghiệm hữu tỷ.

3. Giới hạn miền nghiệm. Thử và sai
Cho $x$ là số nguyên. Nếu $|x| \leq N$ với số nguyên dương $N$ cho trước thì $x$ chỉ có thể nhận hữu hạn giá trị. Cụ thể $x \in \{ -N, -N+1, …, 0, 1, …, N \}$. Điều đơn giản là chìa khóa để giải nhiều phương trình Diophant bằng cách giới hạn miền nghiệm, sau đó dùng phép thử.

Ví dụ 3.1: (Bài toán Arnold) Tìm tất cả các bộ ba số nguyên dương sao cho tích của hai số bất kỳ cộng $1$ chia hết cho số còn lại.
Lời giải: Gọi $a, b, c$ là $3$ số nguyên dương cần tìm. Không mất tính tổng quát, ta có thể giả sử $a \leq b \leq c$. Theo đề bài, ta có
\[ab + 1 \vdots c,\;bc + 1 \vdots a,\;ca + 1 \vdots b\]
Suy ra $\frac{{(ab + 1)(bc + 1)(ca + 1)}}{{abc}}$ là số nguyên. Điều này tương đương với
$$\frac{1}{a} + \frac{1}{b} + \frac{1}{c} + \frac{1}{{abc}} \in \mathbb{Z} \ \ \ (1)$$
Như vậy $$\frac{1}{a} + \frac{1}{b} + \frac{1}{c} + \frac{1}{{abc}} \geqslant 1$$
Sử dụng điều kiện $a \leq b \leq c$ ta suy ra
\[\frac{3}{a} + \frac{1}{{{a^3}}} \geqslant 1\]
Từ đó dễ dàng suy ra $a \leq 3$.
Ta xét các trường hợp
*) $a = 1$. Khi đó $(1)$ tương đương với $\frac{1}{b} + \frac{1}{c} + \frac{1}{{bc}} \in \mathbb{Z}$. Lý luận tương tự ta suy ra $b \leq 2$.
+) Nếu $b = 1$, ta được ngay $\frac{2}{c} \in \mathbb{Z}$ suy ra $c = 1$, hoặc $c = 2$. Ta được bộ nghiệm $(1 ; 1 ; 1), (1 ; 1 ; 2)$. Các nghiệm này thoả mãn điều kiện (chú ý $(1)$ chỉ là điều kiện cần!).
+) Nếu $b = 2$, ta được $\frac{1}{2} + \frac{1}{c} + \frac{1}{{2c}} = k \in \mathbb{Z}$. Do $c \geq 2$ nên $\frac{1}{2} + \frac{1}{c} + \frac{1}{{2c}} \leqslant \frac{5}{4}$ suy ra $k = 1$. Từ đó tính được $c = 3$. Thử lại ta thấy nghiệm $(1; 2; 3)$ thoả mãn điều kiện.
*) $a = 2$. Khi đó $(1)$ tương đương với $\frac{1}{2} + \frac{1}{b} + \frac{1}{c} + \frac{1}{{2bc}} = k \in \mathbb{Z}$. Do $c \geq b \geq a = 2$ nên \[\frac{1}{2} + \frac{1}{b} + \frac{1}{c} + \frac{1}{{2bc}} \leqslant \frac{1}{2} + \frac{1}{2} + \frac{1}{2} + \frac{1}{8} = \frac{{13}}{8}\]
nên suy ra $k = 1$. Ta được phương trình
\[\frac{1}{b} + \frac{1}{c} + \frac{1}{{2bc}} = \frac{1}{2} \Leftrightarrow 2(b + c) + 1 = bc \Leftrightarrow (b - 2)(c - 2) = 5\]
Suy ra $b = 3, c = 7$. Thử lại bộ $(2; 3; 7)$ thoả mãn điều kiện.
*) $a = 3$. Khi đó $(1)$ tương đương với \[\frac{1}{3} + \frac{1}{b} + \frac{1}{c} + \frac{1}{{3bc}} = k \in \mathbb{Z}\]. Lý luận tương tự như ở trên, ta được $k = 1$. Tức là
\[\frac{1}{b} + \frac{1}{c} + \frac{1}{{3bc}} = \frac{2}{3}\]
+) Nếu $b > 3$ thì ta có \[\frac{1}{b} + \frac{1}{c} + \frac{1}{{3bc}} \leqslant \frac{1}{4} + \frac{1}{4} + \frac{1}{{48}} < \frac{2}{3}\].
Như vậy chỉ còn khả năng $b = 3$. Nhưng khi đó tính ra được $c = \frac{{10}}{3}$ không nguyên, loại.
Vậy bài toán có các nghiệm là $(1 ; 1 ; 1), (1 ; 1 ; 2), (1 ; 2 ; 3), (2 ; 3 ; 7)$ (và các hoán vị).

Ví dụ 3.2: (Olympic 30/4 năm 1999) Trong mặt phẳng tọa độ $Oxy$ cho $3$ đường thẳng có hệ số góc lần lượt là $\frac{1}{m}, \frac{1}{n}, \frac{1}{p}$ với $m, n, p$ là các số nguyên dương. Tìm $m, n, p$ sao cho $3$ đường thẳng đó tạo với trục hoành $3$ góc có tổng số đo là $45^0$.

Lời giải: Gọi $\alpha, \beta, \gamma$ là các góc tạo bởi ba đường thẳng đó với trục hoành. Theo đề bài ta có
\[\tan \alpha = \frac{1}{m},\;\tan \beta = \frac{1}{n},\;\tan \gamma = \frac{1}{p}\].
Ta cần tìm $m, n, p$ nguyên dương sao cho $\alpha + \beta + \gamma = 45^0$. Điều này tương đương với
$$\begin{gathered} \tan (\alpha + \beta + \gamma ) = 1 \\ \Leftrightarrow \frac{{\tan \alpha + \tan \beta + \tan \gamma - \tan \alpha \tan \beta \tan \gamma }}{{1 - \tan \alpha \tan \beta - \tan \beta \tan \gamma - \tan \gamma \tan \alpha }} = 1 \\ \Leftrightarrow \frac{1}{m} + \frac{1}{n} + \frac{1}{p} + \frac{1}{{mn}} + \frac{1}{{np}} + \frac{1}{{pm}} - \frac{1}{{mnp}} = 1\quad (1) \end{gathered}$$
Không mất tính tổng quát, giả sử $m \leq n \leq p$. Khi đó
\[\frac{1}{m} + \frac{1}{n} + \frac{1}{p} + \frac{1}{{mn}} + \frac{1}{{np}} + \frac{1}{{pm}} - \frac{1}{{mnp}} < \frac{3}{n} + \frac{3}{{{n^2}}}\]
Từ phương trình $(1)$ ta suy ra
\[\frac{3}{m} + \frac{3}{{{m^2}}} > 1\]
Từ đó $n \leq 3$. Dễ thấy $n > 1$ nên ta chỉ có 2 trường hợp cần phải xét
+) $m = 2$. Thay vào $(1)$ ta được



$$\begin{gathered} \frac{1}{2} + \frac{1}{n} + \frac{1}{p} + \frac{1}{{2n}} + \frac{1}{{np}} + \frac{1}{{2p}} - \frac{1}{{2np}} = 1\\ \Leftrightarrow np = 3(n + p) + 1 \Leftrightarrow (n - 3)(p - 3) = 10 = 1.10 = 2.5\end{gathered}$$


Suy ra $(n ; p) = (4; 13)$ hoặc $(n;p)=(5, 8)$.

+) $m = 3$. Thay vào $(1)$, ta được



$$\begin{gathered} \frac{1}{3} + \frac{1}{n} + \frac{1}{p} + \frac{1}{{3n}} + \frac{1}{{np}} + \frac{1}{{3p}} - \frac{1}{{3np}} = 1 \\ \Leftrightarrow np = 2(n + p) + 1 \Leftrightarrow (n - 2)(p - 2) = 5 = 1.5 \end{gathered}$$


Suy ra $(n; p) = (3; 7)$.

Kết hợp lại ta được các đáp số của bài toán là $(m; n; p) = (2; 4; 13), (2; 5; 8), (3; 3; 7)$ và các hoán vị.

Bài tập

3.3. Tìm tất cả các nghiệm nguyên dương của phương trình
\[\frac{1}{{{x^2}}} + \frac{1}{{{y^2}}} + \frac{1}{{{z^2}}} + \frac{1}{{{t^2}}} = 4\]

3.4. Giải phương trình trong tập hợp các số nguyên dương
\[\frac{1}{x} + \frac{1}{y} + \frac{1}{z} = 1\]

3.5. Tìm tất cả các bộ số nguyên dương $(x, y)$ sao cho $x^2 + 1$ chia hết cho $y$ và $y^3 + 1$ chia hết cho $x^2$.

3.6. (Bulgaria 2001) Tìm tất cả các bộ số nguyên dương $(a, b, c)$ sao cho $a^3 + b^3 + c^3$ chia hết cho $a^2b, b^2c, c^2a$.

3.7. (Ailen 2003) Tìm tất cả các nghiệm nguyên của phương trình $(m^2+n)(n^2+m) = (m+n)^3$.

3.8. (Moldova 2003) Chứng minh rằng phương trình:
$$\frac{1}{a} + \frac{1}{b} + \frac{1}{c} + \frac{1}{abc} = \frac{12}{a+b+c}$$
có vô số nghiệm nguyên dương.

3.9. (Rio Plate 2002) Tìm tất cả các cặp số nguyên dương $(a, b)$ sao cho
\[\frac{{{a^2}b + b}}{{a{b^2} + 9}}\]
là một số nguyên.

3.10. (Litva 2003) Tìm tất cả nghiệm nguyên của phương trình $3xy - x - 2y = 8$.

3.11. (IMO 2003) Tìm tất cả các cặp số nguyên dương $(a, b)$ sao cho số
\[\frac{{{a^2}}}{{2a{b^2} - {b^3} + 1}}\]
là số nguyên.

3.10. (Na Uy 2003) Tìm tất cả các bộ số nguyên $(x, y, z)$ sao cho $x^3 + y^3 + z^3 – 3xyz = 2003$.

3.12. (THTT 3/209) Tìm tất cả nghiệm nguyên $(x, y)$ của phương trình $(x+y^2)(y+x^2) = (x-y)^3$

4. Phương pháp xuống thang
Fermat đã dùng phương pháp này để chứng minh phương trình $x^4 + y^4 = z^4$ không có nghiệm nguyên dương. Và cũng từ đây, bằng vài dòng ngắn ngủi trên lề cuốn sách Diophantus, ông đã làm đau đầu các nhà toán học suốt 300 năm qua bằng định lý lớn nổi tiếng mang tên ông.
Cơ sở của phương pháp xuống thang là tính sắp thứ tự tốt của $\mathbb{N} $ (và $\mathbb{N}^k$, tích Đề các của $k$ phiên bản $\mathbb{N}$): Một tập con khác rỗng bất kỳ của $\mathbb{N}$ đều có phần tử nhỏ nhất.
Để chứng minh một phương trình là vô nghiệm, ta giả sử ngược lại rằng tập $R$ các nghiệm nguyên (tự nhiên, nguyên dương) của phương trình khác rỗng. Ta đưa ra một thứ tự tốt trên $R$ và giả sử $\alpha_0$ là nghiệm nhỏ nhất (theo thứ tự nêu trên). Nếu bằng cách nào đó ta xây dựng được nghiệm $\alpha_1$ nhỏ hơn $\alpha_0$ thì chúng ta sẽ đi đến mâu thuẫn. Mâu thuẫn này chứng tỏ điều giả sử là sai và như vậy phương trình đã cho vô nghiệm.

Ví dụ 4.1: Chứng minh rằng nghiệm nguyên duy nhất của phương trình $x^3 – 3y^3 – 9z^3 = 0$ là $(0, 0, 0)$.
Lời giải. Giả sử phương trình đã cho có nghiệm nguyên khác $(0, 0, 0)$. Trong số các nghiệm đó, gọi $(x_0, y_0, z_0)$ là nghiệm nguyên có $|x_0| + |y_0| + |z_0| = d$ nhỏ nhất. Rõ ràng $d > 0$
Từ đẳng thức $x_0^3 = 3(y_0^3+3z_0^3)$ suy ra $x_0$ chia hết cho $3$. Đặt $x_0 = 3x_1$, ta được
$$27x_1^3 = 3(y_0^3+3z_0^3) \Rightarrow y_0^3 = 3(3x_1^3-z_0^3)$$
Từ đây ta lại suy ra $y_0$ chia hết cho $3$. Đặt $y_0 = 3y_1$, ta được
$$27y_0^3 = 3(3x_0^3-z_0^3) \Rightarrow z_0^3 = 3(x_1^3 – 3y_1^3)$$
Từ đẳng thức cuối cùng này ta suy ra $z_0$ chia hết cho $3$ Đặt $z_0 = 3z_1$ thay vào thì ta được
$$x_1^3 – 3y_1^3 – 9z_1^3 = 0$$
tức là $x_1, y_1, z_1$ cũng là một nghiệm của phương trình đề bài.

Nhưng
$$|x_1| + |y_1| + |z_1| = \dfrac{|x_0| + |y_0| + |z_0|}{3} = \frac{d}{3}$$.
Vì $0 < \frac{d}{3} < d$ nên đều này mâu thuẫn với cách chọn $(x_0, y_0, z_0)$.

Vậy phương trình đã cho không có nghiệm nguyên khác $(0, 0, 0)$.

Ví dụ 4.2: Chứng minh rằng phương trình $x^4 + y^4 = z^2$ không có nghiệm nguyên dương.
Lời giải: Giả sử rằng phương trình đã cho có nghiệm nguyên dương. Giả sử $(x, y) = d$, tức là $x = da, y = db$, trong đó $(a, b) = 1$. Khi đó ${a^4} + {b^4} = {\left( {\frac{z}{{{d^2}}}} \right)^2}$. Giả sử $z = d^2c$, trong đó $c \in \mathbb{Q}$, khi đó
$$a^4 + b^4 = c^2, \ \ \ (1)$$

Vì $c \in \mathbb{Q}, c^2 \in \mathbb{N}$ nên $c \in \mathbb{N}^+$. Trong tất cả các nghiệm của phương trình $(1)$, chọn nghiệm có $c$ nhỏ nhất. Ta có $(a^2)^2 + (b^2)^2 = c^2$ trong đó $(a, b) = 1$, suy ra $(a^2, b^2) = 1$, tức là $(a^2, b^2, c)$ là bộ ba Pythagore nguyên thủy. Do đó, tồn tại các số nguyên dương $m, n$ sao cho $a^2 = m^2 – n^2, b^2 = 2mn, c = m^2 + n^2$ trong đó $m, n$ khác tính chẵn lẻ, $m > n$ và $(m, n) = 1$, nghĩa là $a^2 = m^2 – n^2$ lẻ. Giả sử $m$ chẵn, $n$ lẻ. Khi đó $n^2, a^2$ chia $4$ dư $1$, nghĩa là $m^2 = n^2 + a^2$ chia $4$ dư $2$, mâu thuẫn. Vậy $m$ lẻ, $n$ chẵn. Ngoài ra $(a, n, m)$ lập thành bộ Pythagore nguyên thủy, do đó tồn tại $p, q \in \mathbb{N}^+$ sao cho $a = p^2 – q^2, n = 2pq, m = p^2 + q^2$, trong đó $p, q$ khác tính chẵn lẻ, $p > q$ và $(p, q) = 1$, Ngoài ra $b^2 = 2mn$, nghĩa là $b^2 = 4pq(p^2+q^2)$, suy ra $b = 2h, h \in \mathbb{N}^+$, khi đó
$$h^2 = pq(p^2+q^2) \ \ \ (2)$$
Giả sử rằng tồn tại số nguyên tố $r$ chia hết $pq, p^2+q^2$. Vì $r$ chia hết $pq$ nên không mất tổng quát, có thể giả sử $r$ chia hết $p$, khi đó $r$ chia hết $(p^2 + q^2) – p^2 = q^2$, suy ra $r$ chia hết $q$, mâu thuẫn vì $(p, q) = 1$. Vậy $(pq, p^2+q^2) = 1$, như thế, từ $(2)$, ta có $pq = s^2$, $p^2 + q^2 = t^2$ với $s, t \in \mathbb{N}^+$. Vì $pq = s^2, (p, q) = 1$ nên $p = u^2, q = v^2$ với $u, v \in \mathbb{N}^+$, nghĩa là $(u^2)^2 + (v^2)^2 = t^2$ hay $u^4 + v^4 = t^2$, trong đó $c = m^2 + n^2 > m = p^2 + q^2 = t^2 > t$, mâu thuẫn với cách chọn $c$.
Như vậy điều giả sử ban đầu là sai và ta có đpcm.

Bài tập

4.2. Chứng minh rằng phương trình $x^2 + y^2 + z^2 = 2xyz$ không có nghiệm nguyên khác $(0, 0, 0)$

4.3. Chứng minh rằng hệ phương trình
$$\left\{\begin{matrix} {x^2} + {y^2} = {z^2} - {t^2} \\ xy = zt \end{matrix}\right.$$

không có nghiệm nguyên dương.

4.4. (Hungary 2001) Tìm tất cả các số nguyên $x, y, z$ sao cho $5x^2 – 14y^2 = 11z^2$.

4.5. Chứng minh rằng hệ phương trình $x^2 + 2y^2 = z^2, 2x^2 + y^2 = t^2$ không có nghiệm nguyên dương.

4.6. Chứng minh rằng phương trình $x^4 – y^4 = z^2$ không có nghiệm nguyên dương.



#310010 Người Việt giỏi toán ngoài nước Việt

Gửi bởi Ban Biên Tập trong 12-04-2012 - 23:25

Lời BBT: Đây là bài viết của TS Nguyễn Sỹ Phương, CHLB Đức đăng trên tạp chí Tia sáng.

Cuối năm trước, tại Thị phủ thành phố Würzburg, tỉnh Unterfranken (1 tỉnh hưởng quy chế tự trị, đông tới 1 triệu rưỡi dân số, bao trùm 3 thanh phố lớn và 9 huyện) thuộc Tiểu bang Bayern, Đức, đã diễn ra giải vô địch toán lớp 4 toàn tỉnh, vòng chung kết.



Hình đã gửi
Giải vô địch này được tỉnh tổ chức lần đầu tiên vào năm 2008, do hệ thống ngân hàng Volks-& Raiffeisenbanken bảo trợ, được chia làm 3 vòng loại, qua 3 cấp, trường, thành phố, và tỉnh. Khác với ở ta thi học sinh giỏi chỉ dành cho học sinh giỏi sẵn, đứng đầu lớp, giải này, vòng sơ khảo dành cho tất cả các em học sinh lớp 4 toàn tỉnh, không phân biệt kết qủa học tới đâu, tuân theo nguyên lý, bất cứ học sinh nào cũng có cơ hội bình đẳng ngang nhau, giành vô địch. Nhờ đó, giải còn tạo ra một sức hút phấn đấu rầm rộ như ngày hội, với hơn 7.800 em của 207 trường tiểu học toàn tỉnh tham gia. Em học sinh người Việt, Phạm Thanh Mai, 7 tuổi, trường tiểu học Dalbergschule ở vòng 1 đoạt giải nhất toàn trường, được chọn tham gia vòng loại cấp thành phố.

Thanh Mai sinh ra trong một gia đình thuần Việt, bố qua Đức du học từ năm 1987, hiện là chủ một công ty sản xuất phần mềm ứng dụng cho các trường đại học và hãng chế tạo ô tô Đức. Mẹ tốt nghiệp thạc sỹ kinh tế, mở văn phòng kế toán, tư vấn thuế, nhằm vào đông đảo doanh nghiệp người Việt. Tài năng Thanh Mai sớm bộc lộ, lên 2 đã biết ghép vần và làm tính số tới 10, ham thích chương trình tự học qua máy tính Vtech dành cho trẻ lên 5. Ở tuổi lên 3, em đã học tiếng Anh trong nhà trẻ. Lên 5 tuổi, Thanh Mai vào tiểu học. Với kết qủa xuất sắc toàn diện, em được Sở Giáo dục đặc cách luôn lên lớp 3. Tuy nhẩy cóc bỏ qua lớp 2, nhưng Thanh Mai năm nào cũng đứng đầu lớp, và đặc biệt được tín nhiệm làm lớp trưởng liên tục từ lớp 1 tới nay. Một cô bé, cả người lẫn tuổi nhỏ nhất lớp, chắc chắn phải có “bảo bối“ uy lực “ghê gớm", mới có thể “cầm đầu“ tất cả bạn đồng học người Đức lớn hơn tới hai ba tuổi, và cao hơn chừng một cái đầu.

Tại vòng loại cấp thành phố tổ chức tháng tiếp theo, có 80 em tham dự, nửa nam, nửa nữ, đứng đầu 40 trường (ngay chọn học sinh giỏi, họ cũng tính đến bình đẳng nam nữ), với 16 thành viên Ban Giám khảo chấm thi, công bố điểm tại chỗ. Thanh Mai đoạt tiếp giải nhất, được trao bằng thủ khoa và 1 vé xem phim. Đề thi gồm 12 bài toán, dài tới 4 trang, hoàn toàn lấy từ thực tế, từ toán đố đến sắp hình, ghép khối, cắt dán, trò chơi chữ số... Thời gian lại rất hạn chế, chỉ trong 60 phút, nghĩa là đòi hỏi cực nhanh, nghĩ và viết cùng lúc mới kịp, muốn quay cóp cũng khó. Các bậc phụ huynh đưa con em tới thi, ngồi đợi mà ai cũng hồi hộp, căng thẳng hơn cả chúng. Đọc đề thi, nhiều phụ huynh còn “hoảng“ hơn nữa, không thể hình dung chúng lại giải được. Bởi đề toán không hề tương tự dạng nào trong sách giáo khoa và càng không thể học trước kiểu vượt cấp, bồi dưỡng trước như các lớp chuyên toán ở Việt Nam, hay học thêm từ phụ huynh.
Chẳng hạn bài số 4:


“Có 1 chiếc xe bus chở học sinh. $\frac{1}{3}$ số ghế bạn nam ngồi. Bạn nữ ngồi nhiều hơn bạn nam 6 ghế. Ngoài ra còn dư 7 ghế trống. Hỏi xe bus có bao nhiêu ghế ?“.

Dùng phương pháp đại số lập phương trình, thì đây là bài toán giản đơn với đáp số bằng 39 ghế. Nhưng dùng các công thức, quy tắc số học, thì quả không dễ. Tuy nhiên, học sinh nào có khả năng suy luận logic, và năng lực phát hiện nhạy cảm, thì lời giải lại trở nên dễ dàng:


Giả sử số bạn nữ bằng bạn nam, thì chúng ngồi tổng cộng $\frac{2}{3}$ tổng số ghế. $\frac{1}{3}$ số ghế còn lại trống bằng $6+7=13$. Suy ra tổng số ghế là $13 \times 3 = 39$.


Cách thứ 2:


Chia tổng số ghế ra 3 phần. Phần thứ nhất do nam ngồi chiếm $\frac{1}{3}$. Phần thứ 2 do nữ ngồi chiếm $\frac{1}{3}$. Phần thứ 3 cũng chiếm $\frac{1}{3}$ do 6 nữ ngồi và 7 ghế trống. Từ phần thứ 3 suy ra tổng số ghế là $(6+7) \times 3 = 39$.

Cách suy luận bài này phần nào tương tự bài toán đố kinh điển tiểu học ở ta:


Vừa gà vừa chó
Bó lại cho tròn
36 con
100 chân chẵn.
Hỏi mấy gà, mấy chó?

Các bậc phụ huynh có thể dùng bài này thử trí thông minh con em mình ở bậc tiểu học.

Hiệu trưởng trường nơi Thanh Mai theo học đánh giá cuộc thi, “đề bài, nhiều người lớn cũng phải bó tay, bởi để giải được nó đòi hỏi một khả năng tư duy logic đặc biệt, không phải ai cũng có“. Đúng như quan điểm giáo dục của Nữ Giám đốc phòng Giáo dục Aschaffenburg: Các đề toán này được soạn theo chương trình giáo dục, kích thích học sinh tiếp cận đề toán từ thực tế bằng tư duy logic tổng quát, và sử dụng tổng thể mọi phương pháp toán học để tìm lời giải nhanh nhất. Thi học sinh giỏi ít đề cập đến các công thức, quy tắc toán được học mà chủ yếu là phát minh sáng tạo. Chính vì thế ở Đức không có trường chuyên, hay lớp chọn nhằm bồi dưỡng học sinh giỏi, kiểu “nuôi gà nòi thi đấu“ như ở ta.

Đoạt giải nhất vòng loại thành phố, Thanh Mai bước vào vòng chung kết tỉnh Unterfranken, đúng lúc em bước sang tuổi lên 8. Tham gia vòng loại này chỉ còn 24 học sinh đứng đầu 12 thành phố và huyện toàn Tỉnh. Thanh Mai nộp bài ở phút thứ 35 so với 60 phút được phép, đạt tổng số điểm 18/20 cao nhất trong tất cả các kỳ thi vô địch toán toàn tỉnh Unterfranken xưa nay, chỉ sai đáp số một bài do lỗi tính toán. Em được Chủ tịch Tỉnh trao tặng huy chương vàng, danh hiệu vô địch toán toàn tỉnh, kèm một bộ trò chơi bằng gỗ và một chú cáo tinh ranh, to lớn, cũng cao hơn em một cái đầu như bao bạn đồng học (xem ảnh). Còn Chủ tịch tập đoàn ngân hàng Volks- & Raifensenbanken trao tặng em một cuốn sổ tiết kiệm. Trả lời tại chỗ câu hỏi báo chí về tài năng toán học đáng thán phục, em hồn nhiên, thẳng thắn tính cách Đức: “Thực ra bài thi đối với tôi chẳng có gì khó. Bố mẹ tôi người Việt đều là nhà toán học giỏi mà!“. Đọc bài em làm, mới thấy trình độ giải toán thành thục của em, tìm đáp số rất nhẹ nhàng.

Chẳng hạn, đề số 4:


Cuốn sách Agnes đọc, dày chừng 160-170 trang. Ngày thứ 1, Agnes đọc được mấy trang thì thấy hay quá, sang ngày thứ 2 đọc nhiều gấp 3 ngày thứ 1, tới ngày thứ 3 đọc ngốn ngấu nhiều gấp 5. Hỏi sách có bao nhiêu trang ?

Thanh Mai giải:


Tổng số 3 ngày đọc là 9 lần có số trang bằng nhau. Vậy tổng số trang phải chia hết cho 9. Con số chia hết 9, nằm giữa 160 và 170, là 162. Đó chính là số trang sách.


Hay đề số 6:


Một gia đình chạy ô tô. Tuổi người lái nhân với số bánh xe và số người ngồi cho kết qủa 444. Hỏi lái xe bao nhiêu tuổi?

Thanh Mai giải:


Xe có 4 bánh, vậy tuổi của tài xế nhân với số người trong ô tô là $444:4 =111$. Xe chỉ chở được hoặc tối đa 4 người, hoặc 3 hoặc 2 hoặc 1. Do con số 111 phải chia hết cho số người, nên suy ra xe chỉ chở được 3 người, và số tuổi người lái xe là: $111:3 =37$ tuổi.

Bậc phụ huynh nào có con em bậc tiểu học, có thể cho chúng thử giải, để hình dung mức độ giỏi toán thành thục ở cấp vô địch do các nhà giáo Đức ra đề như thế nào ? Và con em mình muốn đạt tới, phải cố gắng phấn đấu tới đâu ?

Thanh Mai được bố mẹ dạy dỗ, hình thành khả năng tự học từ lúc lên 2, bằng cách cho chơi chương trình Toggolino tự học dành cho trẻ em 4-7 tuổi. Thấy con hứng thú ham thích, bố mẹ mua tiếp các điã CD chương trình tự học nhiều môn học khác nhau, của lớp cao hơn, để chúng tự tìm tòi, khám phá trên máy tính, mải mê như một trò chơi điện tử. Đó chính là phương pháp giáo dục hiện đại, được đề cập nhiều hiện nay, “chơi để học, học để chơi“.

Khi đài truyền hình “Bayern 1“ phỏng vấn “em muốn làm nghề gì trong tương lai?“, Thanh Mai trả lời không chút ngập ngừng: “Em muốn trở thành chuyên gia thuế hoặc luật sư kinh tế“. Chẳng rõ, mới tuổi lên 8, em đã hiểu gì về thuế, kinh tế, luật, nhưng có thể khách hàng người Việt của mẹ em, không ít người kinh doanh rủi ro, liêu điêu vì nó, gây cho em ấn tượng mạnh, ý thức phấn đấu, sau này giúp đỡ đồng hương mình, đáp hiếu thế hệ cha mẹ em sang Đức, không ai không khởi đầu sự nghiệp bằng 2 bàn tay trắng, cả đời lặn lội, tha phương, chỉ mỗi kỳ vọng, tương lai chúng phải rạng rỡ nơi xứ người ! – Cũng chính là biểu hiện của nguyên lý giáo dục trên nền tảng môi trường kinh tế xã hội và gia đình, và chính giáo dục phải quay trở lại đặt đòi hỏi cải cách nền tảng đó, nếu muốn tương lai con em "sánh vai với các cường quốc năm châu !“.

Theo Tia Sáng




#307819 "Nghịch lý" cùng ngày sinh nhật

Gửi bởi Ban Biên Tập trong 02-04-2012 - 21:10

NGHỊCH LÝ CÙNG NGÀY SINH NHẬT

Ngô Đức Minh (Cao học Toán ĐHSP TP. Hồ Chí Minh)

giới thiệu


Thật ra nếu tính đầy đủ, tức là tính cả ngày sinh “độc”: 29 tháng 2 (4 năm mới tổ chức sinh nhật được 1 lần) thì có tất cả là 366 ngày sinh nhật. Nếu các bạn có một nhóm 367 người thì chắc chắn rằng sẽ có 2 người cùng ngày sinh nhật (biến cố chắc chắn: xác suất sẽ là 1).


Hình đã gửi


Điều tôi vừa nói chẳng có gì lạ nhưng không biết các bạn có tin không: Chỉ cần nhóm của bạn có 23 người thôi là đã có hơn 50% cơ hội để có 2 bạn cùng ngày sinh rồi! ( tức là xác suất để có 2 người cùng ngày sinh trong nhóm 23 người là lớn hơn 1/2). Mới nghe thì thấy cái tỉ lệ này chẳng hợp lý tí nào nhưng đó lại là sự thật. Những phân tích sau đây có thể chỉ dành cho các bạn đã học phép đếm ở chương trình lớp 11:


Hình đã gửi
Trước tiên ta giả sử nhóm của bạn có $n$ bạn. Vì mỗi bạn đều có 366 cách chọn ngày sinh nhật, cho nên $n$ bạn sẽ có $366^n$ cách chọn ngày sinh- Tức là có tất cả $366^n$ khả năng khác nhau khi nói về ngày sinh của $n$ bạn trong nhóm.

Bây giờ ta sẽ tính xem có bao nhiêu cách chọn ngày sinh cho $n$ bạn trên để sinh nhật của $n$ bạn đó đều khác nhau. Bạn đầu tiên có 366 cách chọn ngày sinh nhật, bạn thứ 2 chỉ còn 365 cách ( vì phải khác ngày sinh nên không được chọn lại ngày sinh của bạn thứ nhất). Tương tự bạn thứ 3 có 364 cách chọn… Như vậy tổng số cách chọn để tất cả n bạn có ngày sinh khác nhau là:
$$366.365.364...(366-n+1)=\dfrac{366!}{(366-n)!}$$
(Một cách khác đây chính là số cách chọn ra $n$ ngày sinh khác nhau từ 366 ngày và có tính đến thứ tự khi chọn, sẽ là chỉnh hợp chập $n$ của 366 phần tử)

Xác suất để ngày sinh của cả $n$ bạn trong nhóm đều khác nhau là:
$$P(A)=\dfrac{366!}{366^n(366-n)!}$$

Suy ra xác suất để trong nhóm n bạn có ít nhất 2 bạn cùng ngày sinh là:
$$P= P(\overline{A})=1- \dfrac{366!}{366^n(366-n)!}$$

Đến đây bạn dùng máy tính (có lẽ phải dùng maple, chứ dùng CASIO nó không tính nổi!) thì thấy rằng với $n=23$ thì $ P \approx 0.506$. Nói khác đi, có đến hơn 50% khả năng để trong nhóm 23 bạn bất kì có 2 bạn cùng ngày sinh nhật.

Thậm chí với $n=57$ thì xác suất đó là 99%! Sau đây là đồ thị thể hiện sự phụ thuộc của $P$ theo $n$:

Hình đã gửi
Nếu quả thật các bạn đọc xong bài này vẫn chưa thật tin tưởng, điều thú vị nên làm là hãy kiểm tra với chính lớp học của bạn và các lớp học bên cạnh nhé…


#303530 Đề kiểm tra nhận thức GV THPT năm học 2011 - 2012 Sở GD&ĐT Lạng Sơn

Gửi bởi Ban Biên Tập trong 11-03-2012 - 11:26

Câu 1 (3 điểm)
Đồng chí hãy cho biết những phẩm chất được thể hiện bằng hành động thiết thực của người giáo viên chủ nhiệm trong trường THPT gắn với phong trào thi đua "Xây dựng trường học thân thiện, học sinh tích cực"?
Đồng chí có kiến nghị, đề xuất gì với các cấp lãnh đạo về công tác chủ nhiệm?

Câu 2 (2 điểm)
Cho hàm số $y = \dfrac{2x-4}{x+1}, \ \ \ \ (1)$.
Tìm trên đồ thị $(1)$ hai điểm đối xứng với nhau qua đường thẳng $MN$ biết $M(-3;0), N(-1;-1)$.

Câu 3 (2 điểm)
Tính diện tích hình phẳng giới hạn bởi parabol $(P): y=4x-x^2$ và các tiếp tuyến được kẻ từ $\left ( \frac{1}{2};2 \right )$ đến $(P)$.

Câu 4 (2 điểm)
Cho hai hình chóp $S.ABCD$ và $S'.ABCD$ có chung đáy là hình vuông $ABCD$ cạnh $a$. Hai đỉnh $S,S'$ nằm về cùng một phía đối với mặt phẳng $(ABCD)$, có hình chiếu vuông góc lên mặt phẳng đáy lần lượt là trung điểm $H$ của $AD$ và trung điểm $K$ của $BC$. Tính thể tích phần chung của hai hình chóp, biết rằng $SH=S'K=h$.

Câu 5 (1 điểm)
Cho dãy số $(u_n)$ thỏa mãn đồng thời hai điều kiện:
1) $u_1 = 2$
2) $u_n=\dfrac{u_1+2u_2+...+(n-1)u_{n-1}}{n(n^2-1)},\forall n>1$
Tìm $\lim v_n$ biết $v_n = (n+1)^3.u_n$

--- Hết---

BBT xin cảm ơn thầy Hoàng Ngọc Thế đã cung cấp cho chúng tôi đề thi này.